The CLAT Post - September, 2024
The CLAT Post - September, 2024
TABLE OF CONTENTS
1. Polity and Governance 03 26. Third Japan-India 2+2 Foreign and Defence
1. Unified Pension Scheme ...................................................... 03 Ministerial Meeting ............................................................... 82
2. Wayanad Tragedy and Loss and Damage Funds in 27. Action Plan (2024-2028) for the Implementation of
UNFCCC (Polity) ...................................................................... 09 the India Poland - Strategic Partnership ..................... 87
3. New Guidelines for Preventive Detention: Supreme 28. 19th India-Africa Business Conclave .............................. 91
Court ............................................................................................ 13 29. Turkmenistan-Afghanistan-Pakistan-India (TAPI)
4. Anti-corruption ombudsman Lokpal Constitutes New pipeline ....................................................................................... 94
Inquiry Wing ............................................................................ 17 30. First Legally Binding International AI Treaty ........... 97
5. Public Accounts Committee to hold ‘performance 31. China-hosted FOCAC summit .......................................... 100
review’ of regulatory bodies ............................................. 20 32. India Signs two Defence agreements with USA ...... 103
6. ‘Effective consultation’: SC ruling on process of HC
5. Environment & Biodiversity 106
judges’ appointment ............................................................. 23
33. Uttarakhand: Now the first state to Launch Gross
7. Paper on Consumption Pattern: Economic Advisory
Environment Product ......................................................... 106
Council to the Prime Minister (EAC-PM)..................... 26
34. Air Quality Life Index (AQLI) 2024 .............................. 109
8. Recent auctions under the Enemy Property Act ...... 29
35. BioE3 (Biotechnology for Economy, Environment and
9. Swachh Bharat Mission decreased infant mortality in
Employment): Centre launches new policy ............ 112
country: Nature Journal ...................................................... 31
36. Launch of Samudra Pratap: Firstindigenously built
10. Centre and Tripura sign Peace pact with 2 insurgent
Pollution Control Vessel .................................................... 115
outfits........................................................................................... 34
37. India Contributes highest to the world Plastic
11. ‘Aparajita Woman and Child Bill (West Bengal
Pollution: Nature Study ..................................................... 117
Criminal Laws and Amendment) Bill 2024’.. ............. 37
38. Swachh Vayu Survekshan Award 2024 ...................... 120
12. Himachal Pradesh's Billto Raise Women's Minimum
Marriage Age to 21 ................................................................ 39 6. What's happening in FamiLE
13. New Flag and insignia ofthe Supreme Court ............. 42 7. Science & Technology 123
14. Justice Hema Committee Report on Malayalam Film 39. Completion of SSLV Development Project ................ 123
Industry ...................................................................................... 44 40. National Space Day 2024 .................................................. 126
15. NALSA’s third National Lok Adalat ................................ 47 41. India launches its first reusable hybrid rocket
16. Formation of 5 New Districts in Ladakh ...................... 50 'RHUMI-1' ................................................................................ 129
8. Sports 132
2. Economy & Governance 52
17. RBI’s Five Strategic Priorities for India's Financial 42. National Sports Day 2024 ................................................ 132
Future .......................................................................................... 52 43. ICC Appoints New Chairman ........................................... 135
18. Overhauling of Income Tax Act ........................................ 55 44. Paralympic Games Paris 2024 ........................................ 138
19. RBI Governor Shaktikanta Das for receiving 'A+' 9. Miscellaneous 144
global rating .............................................................................. 61 45. National Geoscience Awards 2023 ............................... 144
20. National Bank for Financing Infrastructure and 46. India’s second nuclear-powered submarine:
Development ............................................................................ 63 INS Arighat .............................................................................. 147
21. RBI to Launch Unified Lending Interface .................... 65 47. Ramon Magsaysay Award 2024 ..................................... 149
3. How to revise current affairs in the last 60 days for CLAT 48. Major Military Exercises in the Past Month ............. 153
2025 49. India leads as top origin country and destination
for Hindu migrants: Pew Research Centre ............... 155
4. International Affairs 67 50. World Suicide Prevention Day ........................................ 158
22. Comprehensive Strategic Partnership between India
and Malaysia ............................................................................. 67 10. Important One Liners. 160
23. UN General Assembly launched the Multidimensional 11. CLAT POST Mock Test 199
Vulnerability Index (MVI)................................................... 72 51. Question Paper ...................................................................... 199
24. India hosts Voice of Global South summit .................. 76 52. Answer Key and Explanation .......................................... 221
25. Ukraine Visit of Prime Minister Modi ........................... 79
2
Polity & Governance
Unified Pension Scheme
News Highlights:
The Unified Pension Scheme (UPS) was approved by the Union Cabinet on August 24, 2024,
marking a significant shift in India's pension system. The scheme offers government
employees 50% of their last drawn salary as an assured lifelong pension, along with
inflation-adjusted benefits like Dearness Relief (DR).
Additionally, the scheme is contributory in nature, similar to the National Pension Scheme
(NPS), but offers a fixed payout like the Old Pension Scheme (OPS). The scheme is expected to
benefit 23 lakh Central Government employees and will be implemented from April 1,
2025.
Family Pension:
In case of the employee’s death, 60% of the last
pension will be provided to their family.
Introduction: Introduction
The Old Pension Scheme (OPS) was The old pension scheme was done away
introduced in 1871 during British rule under with by the Bharatiya Janata Party (BJP)-led
the Pensions Act. It was designed to offer government during Atal Bihari Vajpayee's
financial security to government employees regime in December 2003. The national
after retirement. pension scheme (NPS) took effect from April
Defined Benefit Plan: OPS is a 'Defined 1, 2004. It was created to reduce the
Benefit' pension system where the pension is financial burden of pensions on the
calculated based on the last drawn salary, government and to provide market-
ensuring a fixed payout. linked retirement savings for
government employees.
Applicability: Initially, the NPS was
applicable only to Central Government
Pension Benefits employees, excluding the Armed Forces.
50% of Last Salary: The OPS assured Later, it was extended to all Indian citizens
government employees 50% of their last drawn (voluntarily) and also became mandatory for
basic salary as a lifelong monthly pension. This most state government employees who
benefit was seen as highly favourable, joined after its implementation.
providing financial security to retirees.
Dearness Allowance (DA): Along with the
pension, retirees received a Dearness Defined Contribution Plan
Allowance (DA), which was adjusted Employee and Employer Contributions:
periodically based on the inflation rate to Unlike the Old Pension Scheme (OPS),
mitigate the cost of living. which was based on a defined benefit, the
Family Pension: After the employee's death, NPS is a defined contribution system.
50-60% of the pension amount was passed This means both the employee and
on to the spouse or family, ensuring employer contribute towards the pension
continued financial support. fund. The employee contributes 10% of
their salary and Dearness Allowance
(DA), while the government contributes
Unfunded Nature 14% for central government employees.
No Contribution from Employees: Under OPS, Voluntary Contributions: Private
employees were not required to contribute any individuals can also join the NPS and
amount towards their pension fund during their contribute voluntarily.
working years.
Pay-As-You-Go: The scheme operated under a
'Pay-As-You-Go' system, meaning that pensions Permanent Pension Account Number
were directly paid out from government Every NPS subscriber is issued a Permanent
revenue, primarily funded by taxes collected Pension Account Number (PPAN), which is
from citizens. This led to increased pension unique and allows the portability of the
liabilities for the government over time. account across jobs and locations.
Dearness Inflation-indexed
Linked to inflation,
Allowance (DA) Not applicable pension with periodic
adjusted periodically
DA adjustments
LEGALEDGE
LEGAL EDGE 7676564400 support@toprankers.com www.toprankers.com 3
7
PRACTICE QUESTIONS
1. What year was the National Pension System (NPS) (c) Arvind Subramanian Committee
implemented for new employees? (d) Vijay Kelkar Committee
(a) 2000 (b) 2002
(c) 2004 (d) 2006 4. What percentage of the average basic pay is guaranteed
as a pension under the Unified Pension Scheme (UPS) for
2. What is the full form of "PFRDA," the regulatory eligible employees?
authority established to promote old-age income (a) 60%
security? (b) 50%
(a) Pension Fund Regulatory & Development Authority (c) 40%
(b) Public Financial Regulatory & Development Agency (d) 30%
(c) Personal Fund Regulatory & Development Authority
(d) Pension Financial Regulation & Distribution 5. What is the minimum qualifying service required for an
Association employee to receive the full assured pension under the
Unified Pension Scheme (UPS)?
3. Which committee's recommendations led to the (a) 20 years
establishment of the Unified Pension Scheme (UPS) that (b) 15 years
will replace the National Pension System (NPS)? (c) 25 years
(a) Raghuram Rajan Committee (d) 30 years
(b) T. V. Somanathan Committee
Disbursement Mechanisms:
The LDF Governing Board is
currently developing mechanisms
for direct access, small grants, &
rapid disbursement options.
Principle of Operation: The fund is designed to offer
The LDF operates on the "Polluter Pays Principle", which mandates that financial relief to vulnerable
historically high-emitting countries, such as developed nations, countries and communities,
compensate vulnerable nations and regions impacted by climate change. particularly those with minimal
contribution to global emissions.
Pledged Amount:
At COP28, a pledge of $475 million was made towards the LDF to strengthen financial assistance to those impacted
by climate-related disasters.
Initiation:
The movement for creating a Loss and Damage Fund was initiated in the early 1990s by the
island nation of Vanuatu and the Alliance of Small Island States (AOSIS). They sought
compensation for damages caused by rising sea levels and other climate impacts.
1 Introduction
The primary objective of the UNFCCC is to stabilize greenhouse gas concentrations in the atmosphere at
a level that would prevent dangerous anthropogenic interference with the climate system.
The treaty aims to achieve this stabilization in a time frame sufficient to allow ecosystems to adapt
naturally to climate change, ensure that food production is not threatened, and to enable economic
development to proceed in a sustainable manner.
Conference of the Parties (COP): The COP is the supreme decision-making body of the convention,
which meets annually to review the implementation of the Convention and any other legal instruments
that the COP adopts.
The first COP meeting was held in Berlin, Germany in March 1995.
Secretariat: Based in Bonn, Germany, the UNFCCC secretariat provides organizational and logistical
support for the COP meetings and assists in the implementation of the convention.
Case Details
Key Provisions of the Guidelines
Fair and Effective Opportunity
Case Name Jaseela Shaji vs. Union of
India Obligates detaining authorities to provide all
Bench Justice Bhushan R Gavai, relevant documents used in the detention decision.
Composition with Justices Prashant
Kumar Mishra and KV
Viswanathan Non-Arbitrary Actions
PRACTICE QUESTIONS
1. Under which act was the preventive detention order 4. Which Amendment Act reduced the period of detention
upheld by the Kerala High Court in the case of Jaseela without obtaining the opinion of an advisory board from
Shaji vs the Union of India? three to two months, although this provision has not yet
(a) IPC (b) COFEPOSA been enforced?
(c) POTA (d) TADA (a) 42nd Amendment (b) 44th Amendment
(c) 45th Amendment (d) 46th Amendment
2. Which article of the Constitution was highlighted by the
Supreme Court in 2024 as protecting the fundamental 5. In which case did the Allahabad High Court rule that
right against arbitrary preventive detention? preventive detention should not be used when a person is
(a) Article 14(5) (b) Article 19(5) already detained, unless their actions cause widespread
(c) Article 21(5) (d) Article 22(5) public disorder or disrupt societal peace?
(a) Abhayraj Gupta v. Superintendent, Central Jail,
3. Which type of detention punishes a person after a trial Bareilly
and conviction in court for an offense they committed? (b) Maneka Gandhi v. Union of India
(a) Preventive Detention (c) Gopalan v. State of Tamil Nadu
(b) Administrative Detention (d) Shreya Singhal v. Union of India
(c) Punitive Detention
(d) Indefinite Detention
Lokpal: About
The Lokpal and Lokayuktas Act, 2013 was enacted to
establish the institution of Lokpal at the central level
and Lokayuktas at the state level to inquire into
allegations of corruption against public
functionaries.
Ministers
Inquiry & Prosecution Wings
Section 11: Lokpal must constitute an Inquiry Members of Parliament
Wing to conduct preliminary investigations.
Section 15: Lokpal must also constitute a
Group A, B, C, and D
Prosecution Wing to prosecute public servants
officers and officials of the
once sufficient grounds for action are found.
Central Government
Role of Lokpal
Functionaries under any
Lokpal can order a Central Bureau of corporation, board,
Investigation (CBI) or Central society, or body financed
Vigilance Commission (CVC) probe into or controlled by the
allegations of corruption, and it can central government.
directly supervise the investigations.
Time Limit for Investigations
Article 32 and 226 of the Constitution The preliminary inquiry must be completed
empower individuals to approach the within 90 days, and the investigation must be
Supreme Court and High Courts if they completed within 6 months, extendable by a
are not satisfied with the decisions of maximum of 6 months with proper justification.
Lokpal.
2. Correct Answer: (c) Three 5. Correct Answer: (a) Pradip Kumar Tripathi
Explanation:The organisational structure of the Lokpal Explanation: Pradip Kumar Tripathi is the current
includes a Director of Inquiry who is assisted by three Secretary heading the Administrative Branch of the
Superintendents of Police (SPs): SP (general), SP Lokpal. This position involves significant responsibilities,
(economic and banking), and SP (cyber). Each of these including overseeing the administrative aspects of the
SPs is further supported by inquiry officers and other Lokpal's operations to ensure efficiency and compliance
staff. with the law.
Examination of Public Accounts: The PAC audits the annual accounts of the
Key Functions government and scrutinises the expenditures and revenues, ensuring they align with
the decisions of Parliament.
of the PAC Scrutiny of Government Audits: The Comptroller and Auditor General (CAG)
submits reports to the President, and these are later reviewed by the PAC to ensure
that public money has been spent appropriately.
Review of Expenditure: The committee reviews the utilisation of funds in various
government programs and projects, ensuring that expenditures are justified, and no
financial irregularities have occurred.
Examination of Regulatory Bodies: The PAC can examine the performance of
regulatory bodies like the Securities and Exchange Board of India (SEBI) and
Telecom Regulatory Authority of India (TRAI), assessing their effectiveness in
managing and regulating their respective sectors.
Investigation of Public Sector Undertakings (PSUs): The PAC also reviews the
financial operations of government-owned corporations and Public Sector
Undertakings (PSUs), ensuring efficient management of public funds.
Calling Witnesses: The PAC has the authority to call ministers, officials, and other
individuals for questioning related to its investigations.
Recommendations not Binding on Government: While the PAC cannot enforce its
recommendations, they carry significant weight, and the government is expected to act
Powers of the
upon them.
PAC Broad Ambit: The PAC has a wider scope of investigation compared to other
parliamentary committees. It is empowered to review the revenue and expenditure of
all government departments and even public infrastructure and utilities like airports
and highways.
Collegium System
The term "collegium" originated from Latin, meaning a body
of colleagues bound together by collective responsibility.
The system was crystallized through three key Supreme
Court judgments collectively known as the Three Judges
Cases (1982, 1993, 1998).
It was introduced in 1993 in the Second Judges Case.
Initially it formed in the consultation with the 2 senior-most
judges in the SC. However, in 1998 – Supreme Court
expanded the Collegium into 5 member body.
Supreme Court Collegium: headed by CJI & 4 other senior
most judges of Apex Court
High Court Collegium: Chief Justice of HC & 4 other Senior
most judges of that court.
In the matter of Transfer – the opinion of the CJI is deemed
“determinative”.
PRACTICE QUESTIONS
1. What is the First Judges Case also known by? 4. Which article of the Constitution was referenced in the
(a) Kesavananda Bharati v. State of Kerala Third Judges Case to expand the Collegium to a five-
(b) Indira Nehru Gandhi v. Raj Narain member body?
(c) Maneka Gandhi v. Union of India (a) Article 124
(d) S.P. Gupta v. Union of India (b) Article 136
(c) Article 140
2. How many judges, including the Chief Justice, are part of (d) Article 143
a High Court collegium?
(a) Three (b) Four 5. In which year did the Supreme Court introduce the
(c) Five (d) Six Collegium system, specifying that "consultation" with the
Chief Justice of India implies "concurrence"?
3. Who heads the Supreme Court collegium responsible (a) 1987
for the appointment of judges? (b) 1993
(a) Prime Minister (b) Chief Justice of India (c) 1998
(c) Law Minister (d) President of India (d) 2001
Broader Trends:
Decline in Household Expenditure on Food:
The paper also notes a significant socio-economic shift: for the first time since 1947, the average household
expenditure on food in India has dropped to less than half of total expenditure.
This reflects changes in consumption patterns, improved economic conditions, and the availability of non-food-
related goods and services.
For the first time since 1947, India’s average household expenditure on food has dropped to less than 50% of
total expenditure.
In previous decades, food constituted a larger portion of household budgets. For instance:
PRACTICE QUESTIONS
1. Which agency serves as the Nodal Agency for the EAC- 4. Who has the prime responsibility of procuring food
PM? grains at the Minimum Support Price (MSP)?
(a) RBI (b) Ministry of Finance (a) RBI
(c) NITI Aayog (d) SEBI (b) Ministry of Agriculture
(c) State Governments
2. How many countries are included in the EAC-PM's (d) FCI
working paper on the share of religious minorities?
(a) 165 (b) 167 (c) 170 (d) 175 5. Who heads the Economic Advisory Council to the Prime
Minister (EAC-PM)?
3. In which year was the Economic Advisory Council to the (a) Prime Minister
Prime Minister (EAC-PM) established? (b) Chairman
(a) 2015 (b) 2016 (c) Finance Minister
(c) 2017 (d) 2018 (d) President
2. In which year was the Enemy Property Act enacted? 5. In which 2005 case did the son of a former Raja, who had
(a) 1947 (b) 1956 acquired Pakistani citizenship, challenge the classification
(c) 1965 (d) 1968 of his inherited properties in Uttar Pradesh as enemy
property after his father's death?
3. In which year was the Enemy Property (Amendment (a) Union of India vs. Raja Mohammad Amir Mohammad
and Validation) Bill, originally modifying the 1968 Act Khan
and the 1971 Act, passed by Parliament? (b) Government of India vs. Nawab of Bhopal
(a) 2015 (b) 2016 (c) Republic of India vs. Maharaja of Gwalior
(c) 2017 (d) 2018 (d) State of India vs. Nawab of Pataudi
5 Despite the overall success, the study noted that disparities in toilet adoption persist due to caste
and religion-based discrimination. Some communities, especially marginalized groups, face
challenges in accessing and using toilets.
Varanasi, the Best Ganga Town: For its efforts Surge in Participation:
in river cleanliness and waste management along Starting from just 73 cities in
the Ganga, Varanasi was recognized as the best 2016, Swachh Survekshan has
Ganga town under Swachh Survekshan 2024. grown to assess 4,477 cities in
2024, reflecting the vast scale Manohar Lal Khattar
Surat and Navi Mumbai Shine: Surat (Gujarat) of urban India’s push towards
and Navi Mumbai (Maharashtra) claimed the cleanliness.
second and third spots, respectively, in the list
of cleanest cities. These cities have been
consistent performers in recent years.
2. In which year was the Swachh Bharat Mission (SBM), a 5. Who recruits the volunteers known as Swachh Bharat
flagship sanitation program aimed at achieving Preraks to monitor the progress of sanitation activities
universal sanitation coverage, launched by Prime under the Swachh Bharat Mission?
Minister Narendra Modi? (a) Government of India
(a) 2012 (b) 2013 (b) Tata Trusts
(c) 2014 (d) 2015 (c) Non-Governmental Organizations
(d) Local Municipalities
3. How many infant deaths annually did the Swachh
Ethnic Conflict:
Manipur is home to a complex ethnic composition, with major
communities including the Meiteis, Nagais, and Kukis.
The Meitei community, which comprises around 53% of the
population, primarily inhabits the Imphal Valley, while the
Nagas and Kukis are dominant in the hill districts.
Tensions have long existed between the Meiteis and the tribal
communities (Nagas and Kukis) due to historical grievances,
land disputes, and political representation issues.
A key factor in the recent tensions is the demand for Scheduled
Tribe (ST) status by the Meitei community. The Meiteis, despite
being the majority, are not recognized as an ST.
The tribal groups, particularly the Kukis and Nagas, oppose the
Meiteis' ST demand.
Armed Insurgency:
Manipur has witnessed armed insurgency for decades, with
multiple insurgent groups operating in the state. Some of the
major insurgent outfits include the United National Liberation
Front (UNLF), People’s Liberation Army (PLA), and Kuki
National Organization (KNO).
These groups primarily fight for greater autonomy or
independence from India, with the UNLF advocating for an
independent Manipur, while Kuki groups demand a separate
Kuki homeland.
Insurgent activities have further complicated the situation,
leading to ongoing security operations by the Indian Army and
paramilitary forces.
The Meiteis dominate politics in the Imphal Valley, while Kuki
and Naga communities have more influence in the hill areas.
The unequal representation has led to persistent demands for
autonomy by the hill tribes.
Key Amendments
Speedy Investigation:
The Bill mandates that investigations into rape cases must be completed
within 21 days from the filing of the First Information Report (FIR).
A special Aparajita Task Force will be formed from the West Bengal police
to ensure time-bound completion of investigations into sexual assault cases.
The establishment of 52 special fast-track courts is aimed not only at handling
sexual violence cases but also cases related to women and children
specifically. These courts are designed to ensure timely justice and prevent
delays.
PRACTICE QUESTIONS
1. Which of the following Acts is not proposed to be punishment and the harshest penalties for rape and
amended by the Aprajita Bill 2024? sexual harassment?
(a) Bharatiya Nyaya Sanhita (BNS) 2023 (a) Tamil Nadu (b) West Bengal
(b) Bhartiya Nagrik Suraksha Sanhita (BNSS) 2023 (c) Maharashtra (d) Karnataka
(c) Protection of Children from Sexual Offences Act
(POCSO) 2012 4. Under the Aparajita Women and Child Bill, 2024, what is
(d) Indian Penal Code (IPC) the mandated time frame for completing investigations
into rape cases?
2. Which article of the Indian Constitution provides states (a) 21 days (b) 30 days
with the authority to legislate on issues listed in the (c) 2 months. (d) 3 months
State List but introduces complexity due to concurrent
jurisdiction over criminal laws? 5. Under which section of the Bharatiya Nagarik Suraksha
(a) Article 246 (b) Article 245 Sanhita (BNSS) is it mandated that no private or
(c) Article 248. (d) Article 249 government hospitals can charge fees for the treatment of
rape victims?
3. Which state's assembly passed the Aparajita Women (a) Section 350D. (b) Section 357C
and Child Bill, 2024, that includes provisions for capital (c) Section 365B. (d) Section 375A
Legal Implication:
Concurrent Jurisdiction:
Marriage and divorce fall under the Concurrent List in the Seventh Schedule (Entry 5) of the Indian
Constitution, allowing both the Centre and states to legislate on the matter.
However, since this Bill introduces provisions that differ from those in the 2006 Act (central law), Article 254 of
the Constitution becomes relevant. This article states that if a state law is inconsistent with an existing central law,
the inconsistent provisions may be void unless the Bill is reserved for the President’s assent.
Need for President’s Assent
When the President grants assent to a state law that is inconsistent with a central law, the state law prevails within
that state, according to Article 254(2).
For this Bill to become law, the Governor of Himachal Pradesh will need to reserve it for the President’s
consideration. If the President grants assent, the state law will override the provisions of the central law
within Himachal Pradesh.
PRACTICE QUESTIONS
1. Which Act does the Himachal Pradesh (HP) Assembly marriage and divorce, including child marriages, fall that
aim to amend by raising the minimum marriageable age allows both central and state governments to legislate?
for women from 18 to 21 years? (a) Entry 3 (b) Entry 5 (c) Entry 7 (d) Entry 9
(a) Child Marriage Restraint Act, 1929
(b) Prohibition of Child Marriage (Amendment) Act, 4. When was The Juvenile Justice (Care and Protection of
2010 Children) Act, enforced?
(c) Prohibition of Child Marriage Act, 2006 (a) 2014 (b) 2015 (c) 2016 (d) 2017
(d) Child Marriage Prevention Act, 1995
5. Which committee, established by the Ministry for Women
2. Under which article of the Indian Constitution can a and Child Development in June 2020, recommended
state governor reserve a bill for the President's increasing the marriage age to 21 and enhancing access to
consideration if it might undermine the authority of the education, skill training, and sex education?
High Court or conflict with central laws? (a) The Verma Committee
(a) Article 200 (b) Article 201 (b) The Malhotra Committee
(c) Article 202 (d) Article 203 (c) The Jaya Jaitly Committee
(d) The Sharma Committee
3. According to the Seventh Schedule of the Indian
Constitution, under which entry does the regulation of
PRACTICE QUESTIONS
1. Which of the following is not featured on the new flag of (a) Article 104 (b) Article 114
the Supreme Court, unveiled by President Droupadi (c) Article 124 (d) Article 134
Murmu to mark the 75th year of its establishment?
(a) The Ashoka Chakra 4. Which institution was responsible for conceptualizing
(b) The Supreme Court building and designing the new flag and insignia of the Supreme
(c) The book of the Constitution of India Court?
(d) The national emblem (a) Indian Institute of Technology (IIT), Delhi
(b) National Institute of Fashion Technology (NIFT), Delhi
2. What is inscribed in Devanagari script on the new (c) National Institute of Design (NID), Ahmedabad
insignia of the Supreme Court of India? (d) School of Planning and Architecture (SPA), Delhi
(a) Satyameva Jayate (b) Yato Dharmastato Jayah
(c) Bharat Mata Ki Jai (d) Jai Hind 5. In what year was a black bronze sculpture of Mother
India, portrayed as a lady, installed on the lawn of the
3. According to the Indian Constitution, which article Supreme Court?
states that "There shall be a Supreme Court of India"? (a) 1968 (b) 1978 (c) 1988 (d) 1998
2. Correct Answer: (b) Yato Dharmastato Jayah 5. Correct Answer: (b) 1978
Explanation: The new insignia of the Supreme Court of Explanation: In 1978, a black bronze sculpture depicting
India features the inscription ‘Yato Dharmastato Jayah’ in Mother India in the form of a lady figure was installed on
Devanagari script. This phrase, which translates to the lawn of the Supreme Court. This sculpture stands at a
"Where there is righteousness (Dharma), there is height of 210 centimeters and represents the
victory," underscores the Supreme Court's commitment embodiment of the nation, symbolically overseeing the
to justice and righteousness in its proceedings and ethos. proceedings of the highest judicial body in India.
The Hema Committee, comprising retired High Court Justice K. Hema, former
actor Sharada, and retired IAS officer K.B. Valsala Kumari was formed in 2017
after Kerala-based Women in Cinema Collective’s (WCC) petition, to study
issues of sexual harassment and gender inequality in Malayalam film industry.
The WCC itself came into existence after a Malayalam woman actor came forward
alleging abduction and sexual assault on her in Kochi. The investigation done by a
team of Kerala Police converged on Malayalam actor Dileep.
Retired Justice K. Hema
PRACTICE QUESTIONS
1. Which of the following individuals was not a member of (c) Women's Rights Advocacy Committee
the Hema committee that investigated the Malayalam (d) Workplace Conduct Review Committee
film industry?
(a) Sharada (b) K B Valsala Kumari 4. Which section of the Bharatiya Nyaya Sanhita, formerly
(c) Anjali Menon (d) K Hema the Indian Penal Code, deals specifically with assault or
criminal force to a woman with intent to outrage her
2. In which year was the POSH (Prevention of Sexual modesty?
Harassment) Act, aimed at addressing sexual (a) Section 352 (b) Section 354
harassment faced by women in the workplace, enacted (c) Section 356 (d) Section 358
by the Government of India?
(a) 2010 (b) 2011 (c) 2012 (d) 2013 5. Which act is specifically aimed at preventing trafficking
for commercial sexual exploitation?
3. Which committee is mandated to be established under (a) Immoral Traffic (Prevention) Act, 1956 (ITPA)
the Sexual Harassment of Women at Workplace (b) Protection of Children from Sexual Offenses Act
(Prevention, Prohibition, and Redressal) Act to address (c) Human Trafficking and Protection Act
issues of sexual harassment? (d) Commercial Sexual Exploitation Prevention Act
(a) Equal Employment Opportunity Committee
(b) Internal Complaints Committee (ICC)
Event Overview:
The Lok Adalat achieved a Resolutions covered a wide range The settlements reached an
resolution of 1,14,56,529 cases, of disputes including criminal approximate value of ₹8,482.08
with 94,60,864 pre-litigation compoundable offences, traffic crore, highlighting the economic
and 19,95,665 pending court challans, bank recoveries, motor implications of the resolutions.
cases being settled. accident claims, labour disputes,
and matrimonial disputes..
Operational Details
Geographical Coverage: Participation and Trust
PRACTICE QUESTIONS
1. Who presided over the 3rd National Lok Adalat of the (a) Civil Procedure Code
year 2024, organized by the National Legal Services (b) Legal Services Authorities Act, 1987
Authority (NALSA)? (c) Indian Judicial Act
(a) Justice Sanjiv Khanna (b) Justice Rohinton Nariman (d) Arbitration and Conciliation Act
(c) Justice U.U. Lalit (d) Justice Indu Malhotra
2. 4. What is the full form of NALSA, the authority established
In how many States and Union Territories was the 3rd to provide free legal services to the weaker sections of
National Lok Adalat of 2024 organized? society?
(a) 15 (b) 21 (c) 27 (d) 29 (a) National Arbitration and Legal Services Authority
(b) National Association for Legal Services and Aid
3. Under which act are Lok Adalats given statutory status, (c) National Legal Services Authority
making their awards final and binding like a decree of a (d) National Alliance for Law and Social Aid
civil court, with no appeal permissible?
PRACTICE QUESTIONS
1. How many new districts were granted "in-principle 4. What is the approximate average elevation of Ladakh,
approval" for formation in Ladakh by the Ministry of making it the highest plateau in India?
Home Affairs? (a) 2,000 m (6,500 ft)
(a) Three (b) Four (c) Five (d) Six (b) 3,000 m (9,800 ft)
(c) 4,500 m (14,800 ft)
2. Which of the following is not one of the five newly (d) 1,500 m (4,900 ft)
approved districts of Ladakh?
(a) Zanskar (b) Drass (c) Leh (d) Nubra 5. What is the capital city of Ladakh as of 2024?
(a) Kargil
3. In which year did Ladakh become a Union Territory (b) Drass
under the Union Home Ministry's direct administration? (c) Leh
(a) 2017 (b) 2018 (c) 2019 (d) 2020 (d) Changthang
Key Aspects of the Income Tax Act Review: Simplification and Modernization
Objective:
1 The primary goal is to make the Income Tax Act more concise, lucid, and easier to comprehend, thereby
minimising disputes and ensuring tax certainty for taxpayers.
Methodology:
2 The review involves eliminating outdated and redundant sections, with a panel led by V.K. Gupta focused on
modernising tax computation methods and simplifying the appeals system.
Expected Completion:
3 The comprehensive review is scheduled to be completed within a six-month timeframe, as stated in the
2024-25 Union Budget announcement by Finance Minister Nirmala Sitharaman.
Historical Context:
Income Tax Act, 1961: The Act was originally framed in 1922 and underwent significant changes in 1961. However,
over the years, many of its provisions have become outdated.
Efforts at Simplification: Previous efforts to simplify the tax law include the 1958 Law Commission of India report,
which sought to modernize the Income Tax Act of 1922. This current review follows a similar path toward
modernization.
Reforms in Corporate Tax: In recent years, corporate tax rates have been reduced, incentives have been phased out,
and the effective tax rate for corporations fell from 29.49% in 2017-18 to 23.26% in 2021-22.
Income Tax Act, 1961:
The Act was originally framed in 1922 and underwent significant changes in 1961. However, over the years, many of
its provisions have become outdated.
Efforts at Simplification: Previous efforts to simplify the tax law include the 1958 Law Commission of India report,
which sought to modernize the Income Tax Act of 1922. This current review follows a similar path toward
modernization.
Reforms in Corporate Tax: In recent years, corporate tax rates have been reduced, incentives have been phased out,
and the effective tax rate for corporations fell from 29.49% in 2017-18 to 23.26% in 2021-22.
Importance of Reviewing the Income Tax Act:
Complexity of Existing Law: The Income Tax Act, 1961, contains over 298 sections and 23 chapters, making it
complex for taxpayers to navigate. Simplification will reduce the burden and enhance compliance.
Global Alignment: The review seeks to align India’s tax laws with global standards, including addressing issues like
tax avoidance and applying global practices like those recommended by the OECD’s BEPS project.
1 2 3
.5%
the financial losses after
the 1857 Revolt.
97
5
Income Tax Act, 1961:
3 The Income Tax Act of 1961 is the Highest Income Tax Rate:
primary legislation that governs the Prime Minister and Congress
taxation of incomes in India. It was leader Indira Gandhi saw
taxation as "a major
2 1961 introduced with 298 sections
instrument to achieve greater
Income Tax Act, 1922: and 23 chapters, detailing various
The Income Tax Act of rules, exemptions, and procedures equality of incomes and
1922 was the first law related to income tax collection. wealth". Her government in
that governed income 1973-74 raised income tax
tax in independent rates to 97.5%, but soon had
India. It served as the Corporate Tax Rates: to withdraw the policy..
backbone for taxation India has been reducing corporate 9
until the Income Tax Act tax rates over time. The effective tax rate
for domestic companies dropped from Income Tax Day:
of 1961 replaced it.
29.49% in 2017-18 to 25% in 2024-25 to Income Tax Day is
encourage business growth and celebrated in India on
8 July 24th every year
investment.
First Income Tax Slab: to commemorate the
establishment of the
In the Income Tax Act of 10 Income Tax Exemptions: 6
1961, the first tax slab for Under the EEE (Exempt-Exempt- income tax system in
individuals was 10% for Exempt) regime, certain investments India in 1860.
incomes above ₹15,001 such as the Public Provident Fund
and up to ₹30,000. (PPF) and Employees' Provident
Fund (EPF) are exempt at all three
stages: contribution, accumulation,
and withdrawal.
Section 80C
11 of the Income Tax Act: 12 7
Section 80C is the most commonly Largest Taxpayer: Number of Taxpayers:
used section by taxpayers to claim Reliance Industries is often As of 2023, India has 93.7
deductions on various investments, regarded as one of India’s million income taxpayers,
such as Life Insurance Premiums, largest corporate taxpayers, a significant increase from
Public Provident Fund, and contributing significant 89.8 million in 2019-2020
National Savings Certificates, with amounts to the exchequer due to tax reforms and
an upper limit of ₹1.5 lakh/year. annually. simplified tax regimes.
PRACTICE QUESTIONS
1. How many sections are contained in the Income Tax Act 3. Which scheme aims to settle direct tax disputes by
of 1961, which serves as the foundational statute for providing taxpayers with an opportunity to resolve
income taxation in India? pending appeals and avoid prolonged litigation?
(a) 305 (b) 298 (a) Aay Samaadhaan Yojana
(c) 275 (d) 250 (b) Bharat Settle Tax Initiative
(c) Pratyaksh kar Sauhaardapoorn Samaadhaan
2. In which year was the Central Board of Direct Taxes (d) Vivad Se Vishwas Scheme
(CBDT) established?
(a) 1924 (b) 1937 4. Within which department of the Ministry of Finance does
(c) 1963 (d) 1971 the Central Board of Direct Taxes (CBDT) operate?
3.
LEGALEDGE 59
7676564400 support@toprankers.com www.toprankers.com
(a) Department of Economic Affairs 5. How many members, each holding the rank of ex-officio
(b) Department of Financial Services Special Secretary to the Government of India, comprise
(c) Department of Revenue the Central Board of Direct Taxes (CBDT)?
(d) Department of Expenditure (a) Four (b) Five
(c) Six (d) Seven
Key Contributions:
Economic Recovery: Das spearheaded RBI’s response to major economic
challenges, particularly during the COVID-19 pandemic, by implementing
Shaktikanta Das monetary policies aimed at inflation control, financial stability, and
mitigating the impacts of global economic uncertainties.
Global Finance Magazine:
Founded: The magazine was established in 1987 and is headquartered in New York.
Readership: Global Finance has a circulation of 50,050 and reaches readers in 163 countries. It primarily caters to
corporate leaders, bankers, and investors, helping them navigate global business and finance trends.
Central Banker Report Cards:
The Central Banker Report Cards,
published since 1994, evaluate central
bank governors from nearly 100
Reserve Bank of India
countries, including regional central
banks like the European Union and Established: April 1, 1935
Central Bank of West African States. Statutory Background: Reserve Bank of India Act,
1934
Grading System: Central bankers are
Nationalized on: January 1, 1949
graded on a scale from "A+" to "F" based Headquarters: Mumbai, Maharashtra
on their success in managing inflation, Incumbent Governor: Shaktikanta Das
economic growth, currency stability, and Functions: Regulates the issuance of banknotes.
interest rates. Maintains reserves to ensure monetary stability.
The report highlights leaders whose Operates and oversees the currency and credit
strategies demonstrate originality, system.
Formulates and executes India's monetary policy.
creativity, and tenacity in managing Controls the supply and cost of credit.
their respective economies.
NaBFID was set up in April 2021 by the National Bank for Financing
Infrastructure and Development Act, 2021, as the fifth All India
Financial Institution (AIFI) to support long-term, non-recourse
infrastructure financing in India.
The institution has both developmental and financial objectives,
focusing on closing the financing gap in the infrastructure sector by
providing innovative financial instruments.
Financial Structure:
Key Initiatives:
First Loan: NaBFID disbursed its first loan in December 2022, to a tunnel project in Jammu and
Kashmir, run by Quazigund Expressway Pvt Ltd
Total Sanctions: By 2024, NaBFID has sanctioned loans worth more than ₹86,804 crore across diverse
infrastructure sectors such as roads, renewable energy, ports, railways, and city gas distribution.
Long-Term Loans: Around 50% of the loans have been sanctioned for a tenure of 15-20 years.
Future Plans: NaBFID plans to sanction loans worth ₹3 lakh crore by March 2026.
Partnerships:
NaBFID collaborates with multilateral institutions for long-term credit lines, blended finance, and
technical assistance.
It has partnered with the International Finance Corporation (IFC) to offer Transaction Advisory
Services aimed at developing a pipeline of investment-ready Public-Private Partnership (PPP) projects
in India's infrastructure sector.
Digital Technologies:
Strengthening cooperation in digital infrastructure, cybersecurity, and emerging
technologies.
Tourism:
Facilitating travel and promoting tourism initiatives between the two nations. India
noted Malaysia’s designation of 2026 as Visit Malaysia Year.
Financial Services:
Public Administration and
Including an agreement between
Governance Reforms:
Labuan Financial Services Authority
Sharing best practices and
(LFSA) and International Financial
enhancing governance.
Services Centres Authority, India
01 03
Defence Collaboration
Both countries agreed to intensify
Multilateral Cooperation defence cooperation through Sustainable Development
and BRICS regular military exchanges and and Climate Action
joint exercises, along with
Malaysia expressed appreciation expanding the defence industry Malaysia agreed to join India’s
for India’s support for ASEAN and R&D collaborations. International Big Cat Alliance
centrality and India's collaboration (IBCA), furthering collaboration in
in Malaysia’s 2025 ASEAN wildlife conservation. Both
Chairmanship. India also agreed to 05 countries also emphasised
cooperation on climate change
support Malaysia’s request to join
BRICS. mitigation and sustainable energy.
The two nations reaffirmed their Malaysia acknowledged India’s
commitment to UN reforms, with global climate initiatives, including
Special Rice Allocation
India receiving Malaysia’s support the International Solar Alliance
for its bid for permanent India agreed to a one-time export (ISA) and the Coalition for Disaster
membership in the UN Security of 2,00,000 metric tonnes of Resilient Infrastructure (CDRI),
Council (UNSC) white rice to Malaysia, easing reflecting shared goals in
concerns over food security. sustainability.
04 06 07
Defense Minister Rajnath Singh's visit in July 2023 resulted in amendments to the 1993 MoU,
along with the establishment of Hindustan Aeronautics Limited's regional office in Kuala Lumpur.
Joint military exercises such as Harimau Shakti (Army), Samudra Lakshmana (Navy), and Udara
Shakti (Air Force) have strengthened inter-service cooperation.
The Indian Navy and the Royal Malaysian Navy regularly engage in maritime cooperation,
promoting regional security.
Malaysia is home to 2.95 million Indians, forming the second-largest Persons of Indian Origin
(PIO) community globally. The community is primarily Tamil-speaking, with significant numbers
also speaking Telugu, Malayalam, and Punjabi.
The Netaji Subhash Chandra Bose Indian Cultural Center (NSCBICC) in Kuala Lumpur, established
in 2010, promotes cultural exchanges, offering classes in Carnatic vocal music, Kathak dance,
Yoga, and Hindi.
India's influence on Malaysian culture is evident in literary works such as the Hikayat Seri Rama,
a Malay adaptation of the Hindu Ramayana, showcasing how Indian epics have transcended
borders.
Shared cultural heritage is also visible in religious sites like the Sri Veera Hanuman Temple, with
architecture rooted in Indian traditions.
PRACTICE QUESTIONS
1. Where will India establish an Ayurveda Chair to ASEAN countries?
promote traditional medicine education and research in (a) ASEAN-India Trade in Goods Agreement
Malaysia? (b) Asia-India Trade and Investment Growth Agreement
(a) Universiti Malaya (c) ASEAN-India Technological Integration Growth
(b) Universiti Tunku Abdul Rahman Agreement
(c) Universiti Sains Malaysia (d) Asia-India Transport Infrastructure Growth
(d) Universiti Teknologi Malaysia Agreement
2. What is the full form of AITIGA, which is being reviewed 3. In which year did Malaysia gain its independence from
to enhance supply chain connections between India and the United Kingdom?
STRUCTURE OF MVI
The MVI produces an overall score for each country This offers a more detailed and tailored analysis of
based on their structural vulnerability and resilience each country’s vulnerability and resilience factors,
across various dimensions of sustainable development. allowing for country-specific insights.
Replaced: 2 3 Founding
League of Members:
Primary Goal:
Nations, which 51 countries, As
To maintain international peace and security, promote
had failed to of 2024, there
human rights, foster social and economic development,
prevent WWII. are 193 member
provide humanitarian aid, and uphold international law.
states in the UN.
The summit reaffirmed the Global South's dedication to achieving the SDGs by 2030, addressing challenges in
development finance, healthcare, climate change, youth empowerment, digital transformation, and trade.
2. Correct Answer: (b) An Empowered Global South for a 5. Correct Answer: (b) Europe
Sustainable Future Explanation: Europe is not considered part of the Global
Explanation: The theme of the 3rd Voice of Global South South. The Global South generally refers to regions outside
Summit hosted by India in 2024 was "An Empowered Europe and North America, including Latin America, Asia,
Global South for a Sustainable Future." This theme reflects Africa, and Oceania. These regions are typically low-income
the focus on fostering empowerment and sustainable and politically or culturally marginalized in the global
development for nations in the Global South. context.
BHISHM Cubes
1 PM Modi presented four BHISHM (Bharat Health Initiative for Sahyog Hita and Maitri) Cubes to the
Ukrainian government to assist with healthcare and humanitarian efforts amid the ongoing war.
These cubes are part of Project Aarogya Maitri.
2 pay respects to the victims of the war, and also paid tribute to Mahatma Gandhi at the ‘Oasis of Peace’
park in Kyiv.
He also interacted with Ukrainian students learning Hindi at the School of Oriental Studies,
acknowledging their contribution to promoting Indian culture.
4
to play an active role in restoring peace in Ukraine.
Modi’s visit is part of a broader diplomatic effort to engage with Europe, especially Eastern and Central
Europe, beyond India’s traditional focus on Russia, Germany, France, and Britain.
India’s Non-Alignment policy is evolving toward maintaining close ties with all countries rather than
keeping equal distance, reflecting a multi-aligned approach to diplomacy.
5
Despite increased engagement with Ukraine, India-Russia relations remain strong, with India playing a
critical role in helping Russia manage sanctions and maintaining military cooperation.
Modi’s visit to Ukraine is not seen as a move to abandon Russia but rather as an attempt to expand
India’s global influence while maintaining strategic balance.
Official Language: Ukrainian, though Russian is widely spoken, especially in the eastern and
southern parts of the country.
Independence Day: August 24, 1991, following the collapse of the Soviet Union.
Historical Ties: Ukraine has been part of various empires and states, including the Kievan Rus,
Polish-Lithuanian Commonwealth, and the Soviet Union.
Culture: Ukrainian folk traditions, especially in music and dance, play a central role
in the country's identity, and Ukraine is home to UNESCO World Heritage
sites like Kyiv's Saint Sophia Cathedral and Lviv’s Old Town.
Chernobyl Disaster: Ukraine was the site of the Chernobyl nuclear disaster in 1986, one of the
worst nuclear accidents in history. The Chernobyl Exclusion Zone remains a
tourist destination and a reminder of the risks associated with nuclear energy.
Western Alignment Ukraine has expressed a strong desire to align with Western institutions,
& War with Russia: seeking membership in the European Union (EU) and NATO.
Russia has long viewed NATO's expansion towards its borders, particularly
Ukraine’s aspirations to join NATO, as a threat to its security. Russia
opposed Ukraine's potential NATO membership, fearing it would
undermine its strategic buffer zone.
The ongoing war with Russia has caused severe economic and
humanitarian crises, with millions displaced and thousands of casualties
since 2022.
Key Highlights from the Third 2+2 Foreign & Defence Ministerial Meeting
PRACTICE QUESTIONS
1. Where was the third 2+2 Foreign and Defence 4. In which century did the historical connection between
Ministerial Meeting between India and Japan held on 20 Japan and India begin with the introduction of
August 2024? Buddhism?
(a) Tokyo (a) 4th century
(b) Mumbai (b) 5th century
(c) Osaka (c) 6th century
(d) New Delhi (d) 7th century
2. In which year did the first 2+2 dialogue between India 5. During the 2019 G20 Osaka summit, Japan and India
and the US take place? signed a Memorandum of Understanding (MoU)
(a) 2016 establishing a sister-state relationship between Gujarat
(b) 2017 and which Japanese prefecture?
(c) 2018 (a) Osaka
(d) 2019 (b) Hyogo
(c) Kyoto
3. What did Indian Prime Minister Jawaharlal Nehru (d) Hokkaido
donate to Ueno Zoo in Tokyo in 1949, symbolizing the
beginning of a renewed relationship between India and
Japan after World War II?
(a) An elephant
(b) A tiger
(c) A lion
(d) A panda
1
Five-Year Action Plan (2024-2028): The two nations agreed on a five-year action plan to strengthen
cooperation in multiple areas. Key focus areas include political dialogue, security cooperation,
economic ties, and technology partnerships.
Strategic Partnership Elevation: India and Poland elevated their bilateral relations to the level of a
3 all its forms. They emphasised the need to implement UN Security Council resolutions and pressed for the
early adoption of the Comprehensive Convention on International Terrorism (CCIT).
India-EU Partnership: The discussions focused on advancing the India-EU Strategic Partnership, with
4 support for concluding the India-EU trade and investment negotiations and operationalizing the India-
EU Trade and Technology Council (TTC). Both nations will collaborate on the India-EU Connectivity
Partnership to promote economic and security cooperation.
Cultural and Educational Exchanges: The two sides agreed to bolster cultural ties, educational
5 partnerships, and promote tourism between India and Poland. People-to-people exchanges were also
emphasized to enhance mutual understanding.
6
potential space exploration initiatives. Poland expressed support for India's International Energy
Agency ambition, while India encouraged Poland to join the International Solar Alliance (ISA) and
the Coalition for Disaster Resilient Infrastructure (CDRI).
Both sides committed to cybersecurity cooperation, emphasising the role of Information and
Communication Technology (ICT) in driving economic and social development.
7
Transport and Connectivity: Enhancing transport infrastructure and increasing direct flight
connections between the two nations were identified as priorities for boosting people-to-people
exchanges and promoting trade.
Political and Security Cooperation: Both sides emphasised regular high-level political dialogues and
8 security consultations to enhance defence cooperation. A Joint Working Group for Defence
Cooperation will convene in 2024 to explore potential areas for collaboration in defence production and
modernization of armed forces.
Trade and Investment: The visit prioritised economic security and trade expansion with a focus on
9
balancing trade and exploring new sectors like high-tech and green technologies. The Joint
Commission for Economic Cooperation (JCEC), led by commerce ministers from both countries, was
identified as a key mechanism to address trade imbalances and boost sectors like food processing, IT,
agriculture, and renewable energy.
India-Poland Relations
PRACTICE QUESTIONS
1. In which year were diplomatic relations between India 4. At which memorial did the Prime Minister of India pay
and Poland established? tribute in Warsaw, commemorating the Jamsaheb of
(a) 1947 (b) 1954 Nawanagar, who provided shelter to over a thousand
(c) 1957 (d) 1960 Polish children during World War II?
(a) Kolhapur Memorial
2. What is the capital city of Poland? (b) Gandhi Memorial
(a) Warsaw (b) Krakow (c) Monte Cassino Memorial
(c) Gdansk (d) Poznan (d) Dobry Maharaja Memorial
3. The Monument to the Battle of Monte Cassino 5. As part of the India-Poland relationship, cooperation has
commemorates soldiers from which military unit who been strengthened in designating individuals affiliated
fought in one of the bloodiest battles of World War II? with groups listed by which UNSC sanctions committee?
(a) First Polish Army (a) UNSC 1373 Sanctions Committee
(b) Indian National Army (b) UNSC 1540 Sanctions Committee
(c) British Expeditionary Force (c) UNSC 1267 Sanctions Committee
(d) Second Polish Corps (d) UNSC 2231 Sanctions Committee
3. Correct Answer: (d) Second Polish Corps 5. Correct Answer: (c) UNSC 1267 Sanctions Committee
Explanation: The Monument to the Battle of Monte Explanation: India and Poland have strengthened
Cassino commemorates the soldiers of the Second Polish cooperation in designating individuals affiliated with
Corps who fought in the Battle of Monte Cassino during terrorist groups listed by the United Nations Security
World War II. Council (UNSC) 1267 Sanctions Committee.
Pharmaceuticals:
Indian pharmaceutical exports to Africa reached Automobiles:
$3.8 billion in 2023, with room for further growth India sees the automobile industry as a major
as Indian firms can provide affordable medicines to area of collaboration. With the rising demand
meet Africa’s healthcare needs. for affordable vehicles, India is aiming to
India is known for its ability to produce low-cost, strengthen its exports of automobiles and
high-quality generic drugs, which can significantly components to African markets.
benefit African healthcare systems.
Key Areas of Focus
for Indian Exports
to Africa
PRACTICE QUESTIONS
1. Who inaugurated the 19th CII India Africa Business (d) Together Towards Tomorrow
Conclave on August 21, 2024?
(a) Prime Minister Narendra Modi 3. In which year was the Confederation of Indian Industry
(b) President Droupadi Murmu (CII) founded?
(c) Vice President Jagdeep Dhankhar (a) 1885 (b) 1895
(d) Finance Minister Nirmala Sitharaman (c) 1905 (d) 1915
2. What was the theme of the 19th CII India Africa 4. Where is the headquarters of the Confederation of Indian
Business Conclave held in 2024? Industry (CII) located?
(a) Creating One Future (a) Mumbai (b) Kolkata
(b) One World, One Market (c) Bengaluru (d) New Delhi
(c) Strengthening Global Partnerships
2. Correct Answer: (a) Creating One Future 5. Correct Answer: (b) Sanjiv Puri
Explanation: The theme of the 19th CII India Africa Explanation: Sanjiv Puri, the Chairman and Managing
Business Conclave in 2024 was "Creating One Future." Director of ITC Limited, has been elected as the President
This theme reflects India's civilizational ethos and aligns of the Confederation of Indian Industry (CII) for the year
with the motto of India's G20 Presidentship, "One Earth, 2024-25.
One Family, One Future," highlighting unity and
collaboration in shaping a shared future.
News Highlights:
Overview:
The TAPI Pipeline, also known as the
"Peace Pipeline", is a 1,814-kilometer
project designed to transport natural gas
from Turkmenistan’s Galkynysh gas field
to India, passing through Afghanistan and
Pakistan.
At full capacity, the pipeline will transport
33 billion cubic meters (bcm) of gas
annually to the respective countries:
Afghanistan (5%), Pakistan (47.5%), and
India (47.5%) during the 30-year
commercial operations period.
Route:
The pipeline will originate from the
Galkynysh gas field in southeast
Turkmenistan.
It will pass through Afghanistan, including Herat and Kandahar, then traverse Balochistan province in Pakistan,
and finally reach Fazilka in the Indian state of Punjab, near the Indo-Pak border.
Timeline and Delays:
The TAPI project was conceived in the 1990s as a strategic energy project for the region.
An inter-governmental agreement was signed in 2010 by Turkmenistan, Afghanistan, Pakistan, and India.
A Gas Pipeline Framework Agreement was signed in December 2010, followed by a bilateral gas sale
agreement in May 2013.
Work on Turkmenistan’s section of the pipeline began in 2015, while a groundbreaking ceremony for the Afghan
section was held in February 2018 in Herat. However, due to security concerns, work was delayed multiple times,
particularly in Afghanistan.
Afghanistan has now announced the start of operations on its side, though experts expect the pipeline to become
fully operational only in the next decade due to ongoing challenges, including financing and security issues.
Financing:
The Asian Development Bank (ADB) is the primary financier of the TAPI project and also serves as the
transaction adviser for the development.
Turkmenistan secured a $700 million loan from the Islamic Development Bank in 2016 for financing its
portion of the pipeline.
The remaining three countries—Afghanistan, Pakistan, and India—have made an initial investment of $200 million
for the project’s development.
Economic Impact and Benefits:
The TAPI pipeline is expected to deliver cheaper natural gas compared to liquefied natural gas (LNG), ensuring a
consistent supply to energy-deficit regions in Afghanistan, Pakistan, and India.
2. Correct Answer: (c) 1,814 kilometres 5. Correct Answer: (b) Afghanistan 5%, Pakistan 47.5%,
Explanation: The TAPI pipeline is expected to span India 47.5%
approximately 1,814 kilometres. This extensive Explanation: During the 30-year commercial operations
infrastructure project is designed to transport a period, the natural gas transported by the TAPI pipeline
significant volume of natural gas from Turkmenistan will be distributed among the buyer countries as follows:
through Afghanistan and Pakistan to India, promoting Afghanistan will receive 5% of the annual volume, while
energy security and regional cooperation. Pakistan and India will each receive 47.5%. This
distribution reflects the agreements made to meet the
3. Correct Answer: (c) 2008 energy demands of each nation involved in the project.
Explanation: India joined the TAPI pipeline initiative in
2008, marking a significant milestone in the
development of this regional project. India's
involvement helped to propel the project forward,
aiming to enhance energy security and economic ties
among the participating countries.
AI Treaty: Features
The AI convention is the result of 24 months of negotiations among 57 countries, including
1
the US, UK, EU, Japan, Canada, and Australia.
It adopts a risk-based approach to the design, development, use, and decommissioning of AI
systems. The treaty applies to AI systems used in both the public sector and the private sector,
covering their entire lifecycle.
The treaty was developed under the leadership of the Council of Europe, an international
2
organisation that promotes human rights, democracy, and the rule of law.
Besides the US, UK, and EU, several other countries including Andorra, Georgia, Iceland,
Norway, Moldova, San Marino, and Israel have also signed the treaty.
Non-member states such as Argentina, Australia, Japan, and Mexico were involved in
drafting the treaty and are expected to sign soon.
The treaty holds signatory countries accountable for any harmful or discriminatory outcomes
resulting from the use of AI systems.
3 Signatories must ensure that AI systems respect privacy rights and equality, and victims of AI-
related human rights violations must have legal recourse.
Countries must protect democratic processes by ensuring that AI systems do not undermine the
separation of powers or interfere with judicial independence.
The treaty covers AI systems used in a wide range of sectors but includes exemptions for
This treaty is the first global, legally binding agreement that addresses the need to regulate AI in a
5
way that respects human rights, democratic principles, and the rule of law.
The treaty holds nations accountable for AI-related violations such as privacy breaches,
discrimination, and human rights abuses. It provides legal mechanisms for individuals harmed by
AI systems to seek justice.
The diverse participation of countries from different continents underscores the global
6
importance of the treaty. It complements existing AI frameworks like the G7 Pact on AI (October
2023), Europe’s AI Act, and the Bletchley Declaration (November 2023), which focus on
regional or multilateral cooperation.
PRACTICE QUESTIONS
1. In which year was the Council of Europe (COE) (b) Only Asian countries
founded? (c) African Union members
(a) 1945 (b) 1949 (c) 1955 (d) 1960 (d) European Union members, the United States, and the
United Kingdom
2. Where is the headquarters of the Council of Europe
(COE) located? 4. How many member states does the Council of Europe
(a) Paris, France (b) Brussels, Belgium (COE) currently have?
(c) Strasbourg, France (d) Geneva, Switzerland (a) 27 (b) 33 (c) 46 (d) 50
3. According to the Council of Europe, the first legally 5. In which year was the Bletchley Declaration signed by 28
binding international treaty on Artificial Intelligence countries?
(AI) will be open for signing by which of the following (a) 2020 (b) 2021
groups? (c) 2022 (d) 2023
(a) Only European Union members
FOCAC includes 53 African nations as members, representing nearly the entire continent except
Eswatini, which maintains diplomatic ties with Taiwan, contrary to Beijing’s "One China" Policy.
The African Union Commission (AUC), the leading organization responsible for promoting economic
integration and cooperation across Africa, is also a member of FOCAC.
The theme for the 2024 FOCAC summit is “Joining Hands to Advance Modernization and Build a
High-Level China-Africa Community with a Shared Future”. This theme reflects China and Africa’s
shared goal of modernization, mutual development, and closer integration in global economic and
political affairs.
PRACTICE QUESTIONS
1. In which city was the Forum on China-Africa 4. Which African nation is not a member of the Forum on
Cooperation (FOCAC) summit, attended by 53 African China-Africa Cooperation (FOCAC) due to its diplomatic
nations, hosted? ties with Taiwan?
(a) Shangha (b) Beijing (a) Eswatini
(c) Guangzhou (d) Shenzhen (b) Botswana
(c) Kenya
2. When was the Forum on China-Africa Cooperation (d) Nigeria
(FOCAC) established to formalize the strategic
partnership between China and African nations? 5. What is the full form of AfCFTA, the agreement that opens
(a) 1990 (b) 1995 up prospects for deeper economic integration among
(c) 2000 (d) 2005 African nations?
(a) African Continental Free Trade Agreement
3. How frequently is the Forum on China-Africa (b) Africa Comprehensive Free Trade Area
Cooperation (FOCAC) summit conducted? (c) African Continental Financial Trade Association
(a) Every two years (b) Every three years (d) Africa Continental Free Trade Alliance
(c) Every four years (d) Every five years
2. Correct Answer: (c) 2000 5. Correct Answer: (a) African Continental Free Trade
Explanation: The Forum on China-Africa Cooperation Agreement
(FOCAC) was established in 2000 to formalize and Explanation: AfCFTA stands for the African Continental
enhance the strategic partnership between China and Free Trade Agreement. This agreement is a significant
African nations. It facilitates cooperative agreements and step towards economic integration across the African
economic partnerships between China and the continent. continent, aiming to create a single market for goods and
services, facilitated by movement of persons to deepen
3. Correct Answer: (b) Every three years the economic integration and prosperity of African
Explanation: The Forum on China-Africa Cooperation countries.
(FOCAC) summit is conducted every three years, with the
hosting duties alternating between China and an African
member nation. This regular engagement helps to
continually strengthen and renew the strategic and
economic ties between the participants.
SOSA ensures that both India and the US will This agreement enables the deployment of
provide reciprocal priority support for goods Indian armed forces officers in key US
and services that promote national defence. This Strategic Commands. The first Indian liaison
arrangement will allow both countries to resolve officer will be posted to the US Special
supply chain disruptions and meet urgent Operations Command headquarters in
national security needs. Florida.
India became the 18th SOSA partner of the US, The goal is to enhance information-sharing
joining a group of nations that collaborate on and improve coordination between the Indian
defence supply chains. and US militaries, strengthening overall
While SOSA is legally non-binding, it is considered defence collaboration.
an important mechanism to improve
interoperability with US defence trade partners.
The US and India are also working towards concluding a Reciprocal Defence Procurement (RDP) Agreement,
which will be legally binding and facilitate better defence trade by promoting interoperability and
standardisation of conventional defence equipment.
Some recent Agreements between India and USA
The 2013 Joint US-India Declaration on Defence Cooperation and the 2015 US-India Defence Framework
outlined a long-term vision for bilateral defence collaboration, covering areas such as joint training, technology
transfer, and industrial cooperation.
2023 Roadmap for Defence Industrial Cooperation: The 2023 roadmap emphasizes co-production in areas such
as jet engines, unmanned platforms, and ground mobility systems. The US has endorsed India’s integration into
global defence supply chains and is supporting the development of naval and maritime infrastructure in India.
Initiative on Critical and Emerging Technologies (iCET): In January 2023, the iCET was established as a
framework to expand cooperation in critical and emerging technologies between India and the US, covering both
commercial and defence technologies. This initiative seeks to co-develop and co-produce cutting-edge defence
systems.
India-US Defence Acceleration Ecosystem (INDUS-X): The INDUS-X was launched in June 2023 during Prime
Minister Narendra Modi’s state visit to the US. It is an initiative to foster defence innovation by building a defence
innovation bridge between the two nations, supporting collaboration in technology, systems, and products.
General Security of Military Information Agreement (GSOMIA) – 2002: This agreement established
the framework for sharing military information between India and the US.
Logistics Exchange Memorandum of Agreement (LEMOA) – 2016: LEMOA enables reciprocal use of
each other's military bases for logistics support, facilitating joint operations and cooperation in
global missions.
Communications Compatibility and Security Agreement (COMCASA) – 2018: COMCASA is an India-
specific version of a US defence agreement that enhances interoperability between the two
militaries, allowing India access to high-end communication technology for its US-origin platforms.
Basic Exchange and Cooperation Agreement (BECA) – 2020: BECA facilitates the exchange of
geospatial intelligence, including high-end military technology, maps, and other sensitive data,
enhancing India’s precision-strike capabilities and intelligence gathering.
Industrial Security Annex (ISA) – 2019: The ISA under the GSOMIA allows for the exchange of
classified information between the defence industries of India and the US, promoting industrial
cooperation in defence projects.
Major Defence Partner Designation – 2016: In 2016, the US designated India as a Major Defence
Partner, providing India with preferential access to defence technologies and military platforms.
Strategic Trade Authorisation (STA) Tier 1 Status – 2018: India was elevated to Strategic Trade
Authorisation (STA) Tier 1 status, allowing licence-free access to a range of military and dual-use
technologies regulated by the US Department of Commerce.
Defence Trade and Technology Initiative (DTTI) – 2012: The DTTI was established to promote
industrial cooperation and defence trade between India and the US. It encourages joint research
and development, and facilitates technology transfer.
PRACTICE QUESTIONS
1. India Signs two Defence agreements with USA (d) Fort Bragg, North Carolina
1. What is the rank of India among the Security of
Supplies Arrangement (SOSA) partners of the US?
(a) 10th (b) 15th (c) 18th (d) 20th 4. What is the full form of GSOMIA, the agreement that laid
the foundation for India-US defense cooperation in 2002?
2. Which of the following countries is not a Security of (a) General Security of Military Information Agreement
Supplies Arrangement (SOSA) partner of the US? (b) Global Security of Military Intelligence Agreement
(a) Canada (b) Germany (c) General Strategy of Military Integration Agreement
(c) Japan (d) Italy (d) Global Strategy of Military Information Association
3. Where will India deploy its first Liaison Officer under 5. In which year was the Logistics Exchange Memorandum
the Memorandum of Understanding (MoU) aimed at of Agreement (LEMOA) signed, establishing a framework
enhancing information-sharing with the US? for reciprocal logistical support between the militaries of
(a) Pentagon, Washington D.C. India and the US?
(b) US Special Operations Command, Florida (a) 2014 (b) 2015 (c) 2016 (d) 2017
(c) Naval Station Norfolk, Virginia
Facts on Uttarakhand!
Land of the Gods: Uttarakhand is often referred to as "Devbhumi" or the Land of the Gods because of its
numerous temples and pilgrimage sites like Kedarnath, Badrinath, Gangotri, and Yamunotri. These four
sites together form the famous Char Dham pilgrimage circuit.
Birthplace of River Ganga: The Ganga, one of the holiest rivers in India, originates from Gaumukh, located
at the Gangotri Glacier in Uttarakhand.
Yoga Capital of the World: The town of Rishikesh is globally recognized as the Yoga Capital of the World.
It attracts thousands of spiritual seekers and yoga enthusiasts from around the world, and many
international yoga festivals are held here annually.
World’s Highest Shiva Temple: The Tungnath Temple, located at an altitude of 12,073 feet in the Garhwal
region, is the highest Shiva temple in the world. It’s a popular trek destination and part of the Panch Kedar
(five temples dedicated to Lord Shiva).
Jim Corbett National Park: Jim Corbett National Park is the oldest national park in India, established in
1936 as Hailey National Park. It was renamed after the famous British hunter and conservationist Jim
Corbett, and is home to a large population of Bengal tigers.
Auli - The Skiing Paradise: Auli, in Uttarakhand, is known as the Skiing Capital of India and is one of the
top skiing destinations in the world. It also offers stunning views of the Nanda Devi and other snow-capped
Himalayan peaks.
Valley of Flowers: The Valley of Flowers is a UNESCO World Heritage site and one of the most beautiful
places in India. Located in the Chamoli district, this high-altitude valley is known for its vibrant floral
meadows and rare medicinal plants.
Birthplace of Chipko Movement: Uttarakhand was the birthplace of the Chipko Movement, a pioneering
environmental conservation movement in the 1970s where villagers, especially women, hugged trees to
prevent their cutting. This movement gained global recognition for environmental protection.
US: Since 1970, the US has seen a 67.2% reduction in pollution, adding 1.5 years to life expectancy.
China: After implementing strict air quality measures in 2014, China saw a 41% reduction in pollution,
increasing life expectancy by 2 years.
South and Southeast Asia: The region remains the most polluted, contributing to 45% of global life
years lost. In Myanmar, pollution reduces life expectancy by 2.9 years.
Africa: In Central and West Africa, PM2.5 concentration averages 22.2 µg/m³, reducing life expectancy
by 1.7 years.
PRACTICE QUESTIONS
1. Which institution released the Air Quality Life Index 4. In which year was the National Air Quality Index
(AQLI) 2024? launched to effectively disseminate air quality
(a) World Health Organization information to the public?
(b) Energy Policy Institute at the University of Chicago (a) 2012
(c) United Nations Environment Programme (b) 2014
(d) Environmental Protection Agency (c) 2016
(d) 2018
2. According to the AQLI 2024 report, what percentage of
India's population breathes air that exceeds the national 5. Which program was revamped by the Government of
PM2.5 standard? India in 2022 to achieve a 40% reduction in particulate
(a) 20% (b) 30% pollution levels by 2026 in 131 non-attainment cities?
(c) 40% (d) 50% (a) National Solar Mission
(b) National Green Corps
3. What is the World Health Organization's recommended (c) National Clean Air Programme
annual average concentration limit for PM2.5? (d) National Environmental Policy
(a) 5 µg/m³ (b) 10 µg/m³
(c) 15 µg/m³ (d) 20 µg/m³
04 03
carbon capture and transformation of
CO2 into useful byproducts like biofuels,
eliminating the need for costly and
unsustainable carbon storage.
Synthetic Biology: Synthetic biology allows for the creation of novel organisms or biochemicals designed to
perform specific functions, such as the production of enzymes for industrial applications or lab-grown organs for
medical transplants.
India's Biotechnology Sector: India’s biotechnology industry is valued at around $80 billion and is projected to
reach $150 billion by 2025. The country is a global leader in biopharmaceuticals, biosimilars, and vaccine
production.
Notable Biotech Innovations: Insulin production through genetically modified bacteria, Golden Rice, a GM crop
designed to combat vitamin A deficiency., Lab-grown meat and animal-free dairy as sustainable food sources.
PRACTICE QUESTIONS
1. Which body approved the 'BioE3 Policy for Fostering (c) Technology Development and Deployment
High-Performance Biomanufacturing'? (d) Environmental Conservation
(a) Supreme Court (b) Parliament
(c) Union Cabinet. (d) State Governments 4. What does 'BioE3' stand for in the context of the new
biotechnology policy?
2. Alongside the BioE3 Policy, the Union Cabinet merged (a) Biotechnology for Economy, Environment, and
three schemes of the Science & Technology Ministry Employment
into a single scheme named: (b) Biotechnology for Ecology, Economy, and
(a) Bharat Shakti (b) Vigyan Jyoti Employment
(c) Vigyan Dhara (d) Tech Pragati (c) Biotechnology for Economy, Energy, and Efficiency
(d) Biotechnology for Education, Environment, and
3. Which of the following is NOT one of the three central Energy
sector umbrella schemes implemented by the
Department of Science & Technology (DST) before their 5. By which year does the BioE3 Policy aim to achieve a US
merger? $300 billion bioeconomy?
(a) S&T Institutional and Human Capacity Building (a) 2025 (b) 2030
(b) R&D and Innovation (c) 2040 (d) 2050
SPECIFICATIONS:
2 The ship has a length of 114.5 metres, a breadth of 16.5 metres, and
displaces 4,170 tonnes.
Its keel-laying ceremony was held on November 21, 2022, marking the
start of the construction phase.
NAME SIGNIFICANCE:
5 The name ‘Samudra Pratap’ translates to ‘Ocean Majesty’, highlighting its
role in preserving the majesty of India's marine environments.
PRACTICE QUESTIONS
1. What is the name of the first indigenously built 4. Who is the current Raksha Rajya Mantri who urged
Pollution Control Vessel launched on 29th August 2024? industry partners to aim for self-sufficiency and net
(a) Samudra Raksha (b) Samudra Prahari export status in defense production?
(c) Samudra Shakti. (d) Samudra Pratap (a) Rajnath Singh
(b) Sanjay Seth
2. Where was the first indigenously built Pollution Control (c) Ajay Bhatt
Vessel launched? (d) Nirmala Sitharaman
(a) Mumbai (b) Kochi
(c) Goa (d) Chennai 5. What is the crew complement of the Pollution Control
Vessel?
3. Which oil spill is considered the largest and most (a) 24 officers and 90 sailors
famous in history? (b) 24 officers and 115 sailors
(a) Deepwater Horizon (b) Prestige (c) 30 officers and 100 sailors
(c) Exxon Valdez. (d) Amoco Cadiz (d) 20 officers and 120 sailors
1. Correct Answer: (d) Samudra Pratap 4. Correct Answer: (b) Sanjay Seth
Explanation: The name of the first indigenously built Explanation: As of June 11, 2024, the current Raksha
Pollution Control Vessel launched is Samudra Pratap. Rajya Mantri (Minister of State for Defence) of India is
Shri Sanjay Seth.
2. Correct Answer: (c) Goa
Explanation: The Pollution Control Vessel was launched 5. Correct Answer: (b) 24 officers and 115 sailors
in Goa. Explanation: The crew complement of the Samudra
Pratap includes 24 officers and 115 sailors, designed to
3. Correct Answer: (a) Deepwater Horizon efficiently manage the operations aboard this large and
Explanation: The Deepwater Horizon oil spill, which technologically advanced vessel.
occurred in 2010 in the Gulf of Mexico, is recognized as
the largest and most famous oil spill in history.
Global Initiatives
UN Plastic Pollution Treaty: In 2022, 175 nations agreed
to negotiate a legally binding agreement to tackle plastic
pollution by 2024.
Basel Convention (2019 Amendment): This agreement
regulated the transboundary movements of hazardous
plastic waste and placed stricter controls on plastic waste
exports.
Indian Initiatives
Extended Producer Responsibility (EPR): India’s EPR policy makes producers responsible
for the entire lifecycle of plastic products, including disposal and recycling.
Plastic Waste Management (Amendment) Rules, 2022: These rules prohibit the use of
plastic carry bags with thickness less than 120 microns, aiming to reduce single-use plastics.
Swachh Bharat Abhiyan and Beach Clean-Up Drives: National campaigns focus on
cleanliness and removal of plastic waste, particularly from beaches and tourist destinations.
Launched By: The Ministry of Environment, Forest Year of Introduction: The ranking system
and Climate Change (MoEF&CC), India. and guidelines for SVS were launched in
Objective: To rank Indian cities based on their efforts to September 2022 under NCAP, which is
combat air pollution, particularly by implementing the aimed at improving air quality in 130 Indian
City Action Plans (CAP) under the National Clean Air cities by reducing air pollution levels.
Programme (NCAP). Primary Goal: To reduce air pollution levels,
Targeted Cities: A total of 131 NCAP cities are specifically Particulate Matter (PM10), by
evaluated. These cities include both large metropolitan 20-30% by 2024 (initial target), and further
areas and smaller towns. extending the goal to 40% by 2025-26, using
2017-2018 levels as the base year.
Objectives:
To promote awareness about air pollution among all Ranking Framework
sections of society. The award rankings are based on data
To inform citizens about the health impacts of air submitted by cities and urban local bodies
pollution. (ULBs) through the PRANA portal, which
To compare air quality across cities and identify monitors the progress of the NCAP
areas of improvement. programme. The rankings compare air quality
To help achieve the NCAP's vision of "Clean Air for All". conditions across cities and assess the
effectiveness of measures taken to reduce
pollution.
Award Winners in 2024
The Swachh Vayu Survekshan Awards 2024 recognized cities under three population categories:
Category 1 (Cities with a population of over 10 lakhs): Surat, Jabalpur, and Agra.
Category 2 (Cities with a population between 3 and 10 lakhs): Firozabad, Amravati, and Jhansi.
Category 3 (Cities with a population under 3 lakhs): Raebareli, Nalgonda, and Nalagarh.
Municipal commissioners of these cities were awarded cash prizes, trophies, and certificates for their cities'
performance.
PRACTICE QUESTIONS
1. Where was the Swachh Vayu Survekshan Award 2024 4. How many categories were there in the Swachh Vayu
presented on the occasion of the International Day of Survekshan Award 2024, under which the best
Clean Air for Blue Skies? performing National Clean Air Campaign (NCAP) cities
(a) Jaipur (b) Mumbai were recognized?
(c) New Delhi (d) Kolkata (a) One (b) Two
(c) Three (d) Four
2. When is the International Day of Clean Air for Blue Skies
celebrated annually? 5. Which portal was launched by the Ministry of
(a) 5th June (b) 7th September Environment, Forest, and Climate Change (MoEFCC) to
(c) 22nd April (d) 16th October monitor the implementation of the National Clean Air
Campaign (NCAP), track action plans and
3. Which organization declared the International Day of implementation status of cities, and share best practices?
Clean Air for Blue Skies in 2019? (a) VAYU (b) NIRMAL
(a) World Health Organization (c) SWACHH (d) PRANA
(b) United Nations Environment Programme
(c) United Nations General Assembly
(d) International Union for Conservation of Nature
Difference between the two: The SSLV, unlike the PSLV and GSLV, is tailored for smaller payloads and quick
launches. It is highly cost-effective (estimated at around Rs 30 crore per launch) and requires significantly less
time (just 72 hours) and workforce (only 6 people) for integration.
1. Correct Answer: (c) EOS-08 4. Correct Answer: (a) New Space India Limited
Explanation: The third developmental flight of ISRO's Explanation: New Space India Limited (NSIL) is the
Small Satellite Launch Vehicle (SSLV) successfully commercial arm of the Indian Space Research
placed the Earth observation satellite EOS-08 into orbit. Organisation (ISRO) tasked with the primary
The satellite EOS-08 is part of India's ongoing efforts to responsibility of facilitating Indian industries in
enhance its capabilities in Earth observation from space. undertaking advanced technology space-related
activities.
2. Correct Answer: (b) Velocity Trimming Module
Explanation: The SSLV utilizes a liquid propulsion-based 5. Correct Answer: (a) Cryogenic Upper Stage
Velocity Trimming Module (VTM) as its terminal stage. Explanation: The third stage of the GSLV is the
This module is critical for making fine adjustments to indigenously built Cryogenic Upper Stage (CUS), which
the satellite's velocity, ensuring that it is placed carries cryogenic propellants. This stage is crucial for
accurately in its designated orbit. providing the necessary thrust and efficiency required
for placing satellites into higher orbits such as the GTO.
3. Correct Answer: (c) 1994
Explanation: The Polar Satellite Launch Vehicle (PSLV),
ISRO's third generation of satellite launch vehicles, was
first used in 1994. This launch marked the beginning of
a series of over 50 successful launches, solidifying
PSLV's reputation as a reliable vehicle for sending
satellites into various orbits.
Chandrayaan-3 Mission Success: On August 23, 2023, the Chandrayaan-3 mission landed
the Vikram Lander on the lunar surface, making India the fourth country to land on the
1
Moon and the first to land near its southern pole. The deployment of the Pragyan Rover
followed the landing, and the site was named 'Shiv Shakti' point.
Aditya L1: Following Chandrayaan-3, ISRO launched the Aditya-L1 mission on September 2,
2023, to study solar activity. By January 2024, Aditya-L1 had reached the Lagrange point L1
(first Earth-Sun Lagrange point) and conducted its first solar storm study in May 2024.
Gaganyaan TV-D1: As part of the human spaceflight mission Gaganyaan, ISRO conducted its
first successful abort test (TV-D1) on October 21, 2023, to demonstrate the Crew Escape
2
System's efficiency.
XPoSat: ISRO launched the X-ray Polarimeter Satellite (XPoSat) on January 1, 2024, which
became operational by mid-January. It focuses on studying X-ray polarisation in space, making
India the second country after NASA to develop a space-based polarimeter.
INSAT-3DS: Launched on February 17, 2024, onboard a GSLV rocket, INSAT-3DS plays a key
role in meteorological studies and preparing for the joint NASA-ISRO Synthetic Aperture
Radar (NISAR) mission in 2025.
Reusable Launch Vehicle (RLV-TD): ISRO conducted two landing experiments with its
downscale Reusable Launch Vehicle in March and June 2024. These experiments pave the
3
way for developing an orbital return flight vehicle.
Small Satellite Launch Vehicle (SSLV): The final SSLV development flight successfully
launched on August 16, 2024, marking the completion of the SSLV programme.
Next-Generation Launch Vehicle (NGLV): ISRO is actively developing the NGLV, a three-
stage launch vehicle aimed at carrying heavier payloads than its predecessors. A project report
was submitted to the Union Cabinet in February 2024, requesting funding for the NGLV.
4
The astronaut candidates (Gaganyatris) were announced by Prime Minister Modi in
February 2024, including Wing Commander Shubhanshu Shukla and Group Captains
Prashanth Nair, Ajit Krishnan, and Angad Pratap.
Shukla and Nair are currently undergoing advanced training in the U.S., and the crewed flight
to the International Space Station is expected by 2025, facilitated by SpaceX and Axiom
Space.
ISRO has laid out an ambitious roadmap for the next two decades:
Gaganyaan’s 25-year roadmap: Includes uncrewed and crewed missions, with plans for a
1
lunar landing by 2040.
Lunar Exploration Roadmap: Involves missions like a lunar sample return and long-duration
lunar surface missions, in collaboration with NASA’s Artemis programme.
Bharatiya Antariksha Station (BAS): Planned by 2035, BAS is intended to be India’s space
station.
Agnikul Cosmos successfully launched its SoRTeD-01 vehicle from Sriharikota on March 21,
1
2024, using a semi-cryogenic engine.
Skyroot Aerospace continues its progress towards launching Vikram 1, having tested its
solid-fuel engines successfully in mid-2024.
Dhruva Space and Bellatrix Aerospace launched experiments aboard the PSLV-C58 mission
on January 1, 2024.
India’s National Space Policy 2023 fosters a robust ecosystem involving private players,
streamlining public-private partnerships to enhance India's space capabilities.
1
NewSpace India Limited (NSIL): Now responsible for commercial space activities, including a
deal with SpaceX to launch GSAT-20 in August 2024.
IN-SPACe: India’s space regulator has been promoting private sector involvement, licensing
private companies for satellite broadband and ground station services.
ELECTRICALLY
HYBRID ROCKET ADJUSTABLE LAUNCH ENVIRONMENTALLY
TRIGGERED PARACHUTE
ENGINE ANGLE FRIENDLY
SYSTEM
Satellites on Board
Cube Satellites: RHUMI-1 carried three Cube Satellites. These are a type of nanosatellite, typically weighing
between 1-10 kg. Cube Satellites are known for their modular design and cost-effectiveness, widely used for
scientific research and commercial applications.
Pico Satellites: The mission also included 50 PICO Satellites, which are even smaller than Cube Satellites, weighing
between 0.1-1 kg. These satellites are primarily used for experimental purposes and gathering specific data on
atmospheric conditions, global warming, and climate change.
Space Zone India is a Chennai-based aerospace technology startup that focuses on providing innovative, low-
cost solutions for the space industry. The company is dedicated to space education and fostering talent in space
sciences, especially among students.
Dr. A.P.J Abdul Kalam Students Satellite Launch (2023): Over 2,500
Notable students from across India contributed to the design and construction of
Projects: a satellite launch vehicle that could carry 150 Pico Satellites. This
mission exemplifies the company's commitment to youth engagement in
space technology.
Martin Group
The Martin Group is an Indian conglomerate involved in various sectors, including aerospace and defense. It
collaborated with Space Zone India for the development and launch of the RHUMI-1 rocket.
Martin Group partnered with Space Zone India in developing the RHUMI-1, India’s first reusable hybrid
rocket, launched on August 24, 2024. The company provided expertise and resources to aid the rocket’s
development, focusing on hybrid propulsion systems and advanced launch technologies.
2. Correct Answer: (d) Suborbital 5. Correct Answer: (a) Space Zone India
Explanation: The RHUMI-1 rocket was launched into a Explanation: Space Zone India, a Tamil Nadu-based
suborbital trajectory using a mobile launcher. This startup, developed the RHUMI-1 rocket in collaboration
trajectory type is typically used for missions that do not with Martin Group. This collaboration highlights the
require a satellite to remain in orbit around the Earth, involvement of local industry and expertise in advancing
suitable for the rocket’s research objectives on climate India's space capabilities.
change.
Application Process:
The Lakshmibai Interested retirees can apply through a
National Institute of dedicated portal set up by the LNIPE. The
Physical Education selection process involves a thorough evaluation
(LNIPE) plays a to ascertain the suitability of candidates for
crucial role in the various courses offered under the programme.
execution of the
RESET Programme.
This partnership is intended to facilitate a robust platform for retired athletes to
acquire new skills, engage with the community, and contribute to nurturing new
generations of sports professionals.
PRACTICE QUESTIONS
1. On which day is National Sports Day celebrated 4. Which civilian award did Major Dhyan Chand receive in
annually in India? 1956?
(a) August 28 (b) August 29 (a) Padma Bhushan (b) Padma Vibhushan
(c) August 30 (d) August 27 (c) Bharat Ratna (d) Padma Shri
2. What programme did the government launch on NSD 5. Which trophy is awarded to institutions for top
2024? performance in inter-university sports tournaments over
(a) RESET (b) BOOST the last year?
(c) SPORT (d) FAST (a) Dronacharya Award
(b) Rajiv Gandhi Khel Ratna
3. In which year did the government launch the Khelo (c) Arjuna Award
India movement? (d) Maulana Abul Kalam Azad Trophy
(a) 2016 (b) 2017
(c) 2018 (d) 2019
Jay Shah, at the age of 35, will join the ranks of notable Indians who have
previously held the top positions at ICC, making him the youngest ever to do so.
Career in Cricket Administration:
Shah started his career in cricket administration in 2009 as an Executive
Board Member of the GCA. In 2013, he became the Joint Secretary of
the GCA and oversaw the construction of the Narendra Modi Stadium in
Ahmedabad.
In 2015, Shah joined the BCCI's finance and marketing committees. In
2019, he became the youngest Secretary of the BCCI. During his tenure,
the IPL became the most profitable cricket league in the world.
In January 2021, Shah was appointed as the President of the ACC. He
was re-elected as ACC President in January 2024.
In November 2022, Shah was elected as the head of the ICC's Finance
and Commercial Affairs (F&CA) committee. He is set to officially assume
office as the Chairman of the ICC on December 1, 2024.
Origins
The Board of Control for Cricket in India
(BCCI) is the governing body for cricket in
India.
Established in December 1928 as a society
under the Tamil Nadu Societies Registration
Act.
It is affiliated with the International Cricket
Council (ICC) and controls all the cricket
activities in the country. Indian Premier League (IPL):
Launched in 2008, the IPL is a
Structure and Governance: professional Twenty20 cricket league
BCCI is structured as a society, comprising in India, conceptualized by the BCCI.
state cricket associations as its members. It is usually held between March and
The key positions in BCCI include the May of every year and features players
President, Secretary, and Treasurer. from around the world.
The management is governed by its General IPL has significantly contributed to
Body and a number of committees. BCCI’s financial strength and has
popularized cricket further across the
Function and Responsibilities:
globe.
Organizes all the official cricket tournaments
within India including the domestic leagues Legal and Regulatory Challenges:
like the Ranji Trophy, Duleep Trophy, and The Lodha Committee reforms in 2016
others. were a significant legal intervention
Selects players, umpires, and officials for by the Supreme Court of India aimed
the national cricket team. at improving transparency and
BCCI is responsible for the promotion and accountability in BCCI’s operations.
development of cricket in India, focusing on The reforms included age caps for
infrastructure development, supporting officials, tenure limits, and one state-
grassroots programs, and enhancing player one vote policy to democratize cricket
performance and welfare. administration.
PRACTICE QUESTIONS
1. In what year was the Imperial Cricket Conference, the has NOT held the position of ICC chairman?
precursor to the International Cricket Council, founded? (a) Jagmohan Dalmiya (b) Sharad Pawar
(a) 1905 (b) 1909 (c) 1911 (d) 1915 (c) N Srinivasan (d) Kapil Dev
2. Jay Shah, the new ICC chairman as of December 2024, 4. Which of the following is NOT an international
previously held which position in the Board of Control tournament organized by the ICC?
for Cricket in India (BCCI)? (a) ICC Cricket World Cup
(a) President (b) Vice-President (b) ICC T20 World Cup
(c) Treasurer (d) Secretary (c) ICC Champions Trophy
(d) ICC Global Challenge Cup
3. As of 2024, Jay Shah is the fifth Indian to be elected as
chairman of the ICC. Which of the following individuals
The Tokyo 2020 Paralympics were historic for India, with the country securing 19 medals—
2020 its highest tally up to that point. Among the standout athletes were:
Avani Lekhara: Gold (Women’s 10m Air Rifle Standing SH1) and
Tokyo
Bronze (Women’s 50m Rifle 3 Positions SH1)
Sumit Antil: Gold (Men’s Javelin Throw F64)
Pramod Bhagat: Gold (Men’s Singles Badminton SL3)
Krishna Nagar: Gold (Men’s Singles Badminton SH6)
Devendra Jhajharia: Silver (Men’s Javelin Throw F46)
Manish Narwal: Gold (Men’s 50m Pistol SH1).
Silver 9
Bronze 13
Total Medals 29
Avani Lekhara Women's 10m Air Rifle Standing SH1 (Shooting) Gold
Mona Agarwal Women's 10m Air Rifle Standing SH1 (Shooting) Bronze
Rakesh Kumar & Sheetal Devi Mixed Team Compound (Archery) Bronze
PRACTICE QUESTIONS
1. How many total medals did India win in the Paris 4. In which competition class did Navdeep Singh win
Paralympics 2024? India's first-ever gold medal in the men's javelin throw
(a) 25 (b) 27 (c) 29 (d) 35 at the Paralympics?
(a) F40 (b) F41
2. In which year were Olympic-style games for athletes (c) F42 (d) F44
with a disability organized for the first time?
(a) 1956 (b) 1960 5. Which country finished on top with the highest number
(c) 1964 (d) 1972 of gold medals at the Paris Paralympics 2024?
(a) United States
3. What is the name of the mascot for the Paris Paralympic (b) Russia
Games 2024, inspired by the Phrygian cap? (c) Great Britain
(a) Paralympic Phryge (b) Paralympic Pierre (d) China
(c) Paralympic Phrix (d) Paralympic Phoenix
Key Recipients
Prof. Dhiraj Mohan Banerjee (INSA): Received the National Geoscience Award for Lifetime Achievement 2023
for outstanding contributions to geosciences.
GSI Team 1 (Abhishek Kumar Shukla, Danira Stephen Dsilva, Parsuram Behera, M.N. Praveen): Awarded for
mineral discovery and exploration (excluding fossil fuels) of economic and strategic importance.
GSI Team 2 (Sanjay Singh, Shailendra Kumar Prajapati, Shashank Shekhar Singh, Kevinguzo Chasie): Recognized
for mineral discovery and exploration (excluding fossil fuels).
Dr. Pawan Dewangan (CSIR-NIO, Goa): Awarded for innovative contributions in coal, lignite, methane, and
hydrocarbon discovery and exploration.
Dr. Harsh Kumar Verma (CSIR-CIMFR, Bilaspur): Awarded for contributions to mining technology and
sustainable mineral development.
Prof. Narasimha Mangadoddy: Recognized for work in mineral beneficiation and sustainable mineral
development.
Dr. Rahul Mohan (NCPOR, Goa): Awarded for contributions to basic geosciences including Arctic/Antarctic
research and geological mapping.
GSI Team 3 (Krishna Kumar, Pragya Pandey, Triparna Ghosh, Debasish Bhattacharya): Won team award for
contributions to basic geosciences.
Prof. Vikram Vishal (IIT Bombay): Awarded for contributions to applied geology, geothermal energy,
seismotectonics, and environmental studies.
Dr. Bantu Prasanta Kumar Patro: Recognized for advancements in geophysics, geophysical survey techniques,
and instrumentation.
Prof. Srimath Tirumala Gudemella Raghukanth: Awarded for natural hazard investigations, including
earthquakes, landslides, and tsunamis.
Dr. Ashutosh Pandey (IISER, Thiruvananthapuram): Received the National Young Geoscientist Award 2023 for
his groundbreaking research and significant insights into the geodynamic evolution of the Eastern Dharwar
Craton.
1
Establishment: The award was created in 1957 in memory of Ramon Magsaysay, the 7th
President of the Philippines, renowned for his leadership and integrity. It is awarded annually to
individuals or organizations in Asia who exhibit selfless service to their communities.
Funds for the Award: The Rockefeller Brothers Fund, based in New York City, funds the Ramon
2 Magsaysay Award. The award was established in 1957 by the trustees of the Rockefeller Brothers Fund
in collaboration with the Philippine government. The award is presented by the Ramon Magsaysay
Award Foundation Annualy.
3
Categories: Initially awarded in five categories (Government Service, Public Service, Community
Leadership, Journalism & Literature, and Peace & International Understanding), the categories were
discontinued in 2009, and since then, the awards are not limited to specific fields.
First Recipient: The first Ramon Magsaysay Awards were given in 1958. The first person to receive
4 the Ramon Magsaysay Award was Acharya Vinoba Bhave in 1958 for his contributions to community
leadership. Other 1958 winners included Dr. Mary Rutnam for Public Service, and the Operation
Brotherhood from the Philippines for Peace and International Understanding.
Prize: The Ramon Magsaysay Award includes a cash prize of USD 50,000, along with a
5 certificate and medallion. The award is given to Asian individuals and organizations who have
demonstrated excellence in their fields.
The Ramon Magsaysay Award is also known internationally as the "Nobel Prize of Asia."
6
Recent Indian Recipients: Dr. Ravi Kannan, a surgical oncologist and the Director of the Cachar
Cancer Hospital and Research Centre (CCHRC) in Assam, was the 2023 Ramon Magsaysay
Award recipient from India.
Emergent Leadership: The Ramon Magsaysay Award for Emergent Leadership is conferred to
outstanding young individuals who are 40 years old or below, and to organizations not more than 10
7 years old that are engaged in exceptionally worthy work. It is supported by a grant from the Ford
Foundation.
As of 2024, 326 individuals and 27 organizations from 23 countries across Asia have been elected
to receive Asia’s premier prize and highest honor.
Highest Number of Indian Recipients: India has had a total of 56 recipients so far,
across various categories including Public Service, Journalism, Community Leadership,
and more.
First Female Indian Recipient: Mother Teresa (1962) was one of the earliest female
winners for her work with the poor and destitute in Kolkata.
Multiple Awards in a Single Year: In 1963, Verghese Kurien, Dara Nusserwanji
Khurody, and Tribhuvandas Patel all received the award for their pioneering work in the
Indian dairy cooperative movement (Operation Flood), which later led to the creation of
Amul.
Youngest Indian Recipient: Sandeep Pandey received the award in 2002 at the age of
37, for his work in grassroots activism through Asha for Education, promoting education
for underprivileged children.
First Ramon Magsaysay Award for Emergent Leadership: Introduced in 2001, the
Emergent Leadership category aims to recognize emerging leaders like Arvind Kejriwal
(2006) for his work on Right to Information (RTI) activism, which paved the way for
greater governmental transparency in India.
Notable Contributions by Indian Awardees: Ela Bhatt (1977) received the award for her
work with SEWA (Self Employed Women's Association), empowering millions of women
in India through self-reliance and cooperative movements.
Aruna Roy (2000) is widely known for her role in drafting and campaigning for India’s RTI
Act, which transformed citizens' ability to hold the government accountable.
PRACTICE QUESTIONS
1. Who was the first recipient of the Ramon Magsaysay (a) Bharat Vatwani (b) Ravish Kumar
Award from India? (c) Thodur Madabusi Krishna (d) Sonam Wangchuk
(a) Mother Teresa (b) Vinoba Bhave
(c) Satyajit Ray (d) Amitabha Chowdhury 4. What is the cash prize amount associated with the
Ramon Magsaysay Award as of 2024?
2. Which award category was introduced in 2001 to (a) USD 30,000 (b) USD 40,000
recognize emerging leaders? (c) USD 50,000 (d) USD 60,000
(a) Community Leadership
(b) Emergent Leadership 5. Which organization from Thailand won the Ramon
(c) Peace and International Understanding Magsaysay Award in 2024 for transforming healthcare in
(d) Journalism, Literature, and Creative Communication rural areas?
Arts (a) Bangkok Health Foundation
(b) Rural Doctors Movement
3. Who among the following Indian recipients won the (c) Thai Health Promotion Foundation
Ramon Magsaysay Award for Journalism, Literature, (d) Thailand Medical Services
and Creative Communication Arts in 2019?
The Eastern Bridge Exercise 2024 is the The 20th edition of Exercise Yudh Abhyas—
seventh edition of a bilateral military exercise the India-USA Joint Military Exercise—
between the Indian Air Force (IAF) and the commenced on September 9, 2024, at the
Royal Oman Air Force (RAFO). Taking place at Mahajan Field Firing Ranges in Rajasthan.
the Masirah Air Force Base in Oman from
September 11 to 22, 2024
2. Since which year has Exercise YUDH ABHYAS been held 5. Which chapter of the United Nations Charter provides
annually? the framework for enforcement actions, including
(a) 2002 (b) 2003 military measures, in Exercise YUDH ABHYAS-2024?
(c) 2004 (d) 2005 (a) Chapter V
(b) Chapter VI
3. Which edition of the India-USA Joint Military Exercise (c) Chapter VII
YUDH ABHYAS took place in 2024? (d) Chapter VIII
(a) 17th edition (b) 18th edition
(c) 19th edition (d) 20th edition
Christians:
Christians represent the
largest share of global
migrants, constituting
47% of the migrant
population.
Hindus:
Hindus make up only 5%
of the global migrant
population, though they
represent 15% of the
world’s total population.
Key Findings on Global Migration Trends (Pew Report 2024)
Global Migration Statistics
Total International Migrants: Over 280
million people or 3.6% of the global
population were living as international
migrants in 2020. Muslims:
Migration Growth: The number of international Muslims account for
migrants increased by 83% from 1990 to 2020, 33% of Indian-born
far outpacing global population growth of 47%. migrants, while globally,
Average Distance Traveled: Migrants travel an India is the second-
average of 2,200 miles. However, Hindus travel largest source of Muslim
the longest, averaging 3,100 miles. migrants after Pakistan.
PRACTICE QUESTIONS
1. When is World Suicide Prevention Day (WSPD) 4. What is the triennial theme for World Suicide Prevention
observed annually? Day from 2024 to 2026?
(a) 1st September (b) 10th September (a) "Changing the Narrative on Suicide"
(c) 20th September (d) 30th September (b) "Hope for Life"
(c) "Speak Up, Save Lives"
2. Where was the first World Suicide Prevention Day (d) "Together for Mental Health"
(WSPD) launched in 2003?
(a) New York (b) London 5. Which ministry announced the "National Suicide
(c) Geneva (d) Stockholm Prevention Strategy" in India?
(a) Ministry of Social Justice and Empowerment
3. Which organizations initiated the first World Suicide (b) Ministry of Home Affairs
Prevention Day (WSPD) in 2003? (c) Ministry of Health and Family Welfare
(a) United Nations and WHO (d) Ministry of Education
(b) Red Cross and IASP
(c) IASP and WHO (d) UNICEF and WHO
Natal Indian Congress Marks 130th Foundation Year: A Legacy of Resistance and Unity
The Natal Indian Congress (NIC), established on August 22, 1894, by Mahatma Gandhi, recently marked its 130th
foundation year.
Originally founded to combat discrimination against Indians in Natal, South Africa, the NIC was the first Indian
Congress of its kind.
Over the decades, it evolved under the umbrella of the South African Indian Congress (SAIC) from the 1920s and saw
a shift towards more radical leadership in the 1930s and 1940s, culminating in Dr. G.M. Naicker’s leadership started
in 1945.
The NIC’s activism intensified, leading to the imprisonment of several leaders by the 1950s and 1960s.
160
State of Healthcare in Rural India, 2024" Report Highlights Critical Gaps
The "State of Healthcare in Rural India, 2024" report highlights significant healthcare challenges in rural areas
across 21 Indian states.
Approximately 50% of rural households have government health insurance, yet 34% lack any insurance, and 61% are
without life insurance. Access issues persist, with only 39% having nearby diagnostic facilities and a mere 12.2%
accessing subsidised medicines.
Additionally, 73% of households with elderly require constant care, predominantly provided by family members,
underscoring the need for enhanced caregiver support and infrastructure improvements.
India Launches Jan Poshan Kendra Initiative to Boost Nutrition and Efficiency in Public Distribution
The Indian government has initiated a pilot project to transform 60 Fair Price Shops (FPS) into "Jan Poshan
Kendras" across Gujarat, Rajasthan, Telangana, and Uttar Pradesh.
This transformation is part of an effort to enhance the nutritional offerings available under the Pradhan Mantri Garib
Kalyan Anna Yojana (PMGKAY). The Jan Poshan Kendra initiative not only aims to sell additional items beyond
subsidised grains, addressing the income challenges of ration dealers, but also to improve service quality with the
introduction of digital tools and support systems to increase transparency and efficiency.
Presently, 0.54 million FPSs distribute about 60-70 million tonnes of food grain annually to over 800 million
beneficiaries.
The government plans to expand this initiative, converting approximately 100,000 fair price shops into "nutri-hubs"
over the next three years, aiming to further enhance the nutritional access and efficiency of India's vast public
distribution system.
Union Minister of Power Launches Three Innovative Online Platforms to Boost Power Sector Efficiency
The Union Minister of Power has launched three online platforms—PROMPT, DRIPS, and JALVIDYUT DPR—to
enhance efficiency, transparency, and effectiveness in India's power sector.
Developed by the Central Electricity Authority (CEA) and NTPC, PROMPT facilitates real-time tracking of thermal
power projects, DRIPS addresses power disruptions from natural disasters, and JAL VIDYUT DPR monitors
hydroelectric projects.
These platforms aim to improve project management, disaster response, and coordination across the sector, reflecting
the Ministry's commitment to modernising infrastructure management and increasing operational transparency.
Indian Prime Minister's Historic Visit to Ukraine: Strengthening Bilateral Ties and Promoting Peace
India's Prime Minister visited Ukraine, marking the first visit by an Indian head of state since Ukraine's independence
in 1991.
The visit highlighted India's active stance on the Russia-Ukraine conflict, emphasising peace and resolution through
stakeholder engagement.
Key outcomes included the establishment of an Intergovernmental Commission to enhance bilateral trade, the signing
of four agreements aimed at strengthening cooperation in agriculture, medical regulation, and cultural ties, and the
donation of four mobile hospital units to Ukraine.
161
Additionally, the Prime Minister extended an invitation to the Ukrainian President to visit India, reinforcing bilateral
relations.
162
51st Fighter Wing Enhances Readiness in Ulchi Freedom Shield 24 Exercise
The 51st Fighter Wing at Osan Air Base, South Korea, is actively participating in the Ulchi Freedom Shield 24, a joint
military exercise with South Korean forces running from August 19 to August 23, 2024.
The exercise aims to boost the wing's operational readiness and strengthen command and control systems in
collaboration with the Seventh Air Force and U.S. Forces Korea.
Key activities include local flying operations and Agile Combat Employment, emphasizing rapid response and
adaptability across various bases.
This strategic training underscores the U.S.'s commitment to regional security and stability in the Pacific, reinforcing
a combined defense posture with South Korea.
India-Denmark Partnership Launches Smart Laboratory on Clean Rivers to Revitalize Varuna River
Under the collaborative Environmental Strategic Partnership between India and Denmark, the Smart Laboratory on
Clean Rivers (SLCR) project has been initiated to rejuvenate the Varuna River, with a combined funding of ₹21.8 crore.
This innovative project aims to leverage advanced technology and a comprehensive river management plan to
enhance the river’s health and bolster the local economy and social conditions over the next 2-3 years.
IIM Bangalore Partners with Alumnus Mathew Cyriac to Launch India’s First Global Centre for PE and VC
On August 25, 2024, the Indian Institute of Management Bangalore (IIM-B) signed a landmark Memorandum of
Understanding with alumnus Mathew Cyriac, Executive Chairman of Florintree Advisors, to establish India’s inaugural
Global Centre of Excellence in Private Equity and Venture Capital.
Named after prominent investment banker Tony James, the Centre aims to be a global leader in these fields, fostering
advanced research and educational endeavours.
ASSOCHAM Environment and Carbon Conference Opens in New Delhi with Focus on Achieving Net Zero by 2070
Shri Kirti Vardhan Singh, the Union Minister of State for Environment, Forest, and Climate Change, inaugurated the
ASSOCHAM Environment and Carbon Conference in New Delhi, emphasising collaborative efforts to achieve India's
goal of net zero emissions by 2070.
Highlighting climate change as a global challenge requiring international cooperation, the Minister outlined India’s
commitment to bolstering climate resilience and fulfilling its Nationally Determined Contributions.
Assam Assembly Passes Law to Mandate Registration of Muslim Marriages and Divorces, Aiming to Curb Child
Marriage and Polygamy
The Assam Legislative Assembly has enacted the Assam Compulsory Registration of Muslim Marriages and Divorces
Bill, 2024, repealing the outdated 1935 Act that allowed child marriage and marriages without mutual consent.
This new legislation sets stringent age and consent requirements for Muslim marriages in Assam, with a woman
needing to be at least 18 and a man 21. Couples must now register their intent to marry 30 days in advance, allowing
time for any legal objections.
163
Google DeepMind's "Morni Project" Aims to Develop AI for 125 Indian Languages
Google DeepMind's India-based "Morni Project" is pioneering AI development to comprehend and interact with 125
Indian languages, enhancing digital inclusivity across India's diverse linguistic landscape.
Despite India's recognition of 22 official languages, Morni seeks to support over 100 additional languages, some
spoken by massive, yet digitally underrepresented populations.
Faced with the significant challenge of scarce digital data for 73 of these languages, Google has initiated Project Vaani
in collaboration with the Indian Institute of Science and ARTPARK.
Prime Minister Modi Inaugurates Vadhvan Port Project in Maharashtra, Aiming for Global Top Ten Status
Prime Minister Narendra Modi laid the foundation stone for the ambitious Vadhvan Port project in Palghar,
Maharashtra, on August 30, 2024.
With an estimated cost of ₹76,000 crore, the port is designed to handle 23.2 million TEUs and aims to rank among the
world's top ten ports.
Strategically located near Dahanu at a deep-water site, Vadhvan Port is set to enhance India's trade capabilities by
facilitating large container ships and reducing shipping costs and times.
India Conducts First National Mock Drill on Zoonotic Diseases, "Vishanu Yuddh Abhyas"
India held its first national mock drill, Vishanu Yuddh Abhyas, from August 27 to 31, 2024, to assess preparedness for
zoonotic disease outbreaks. Initiated under the National One Health Mission (NOHM), the exercise aimed to test
coordination between sectors like human health, animal husbandry, and wildlife management. Key organisations,
including the NCDC, ICMR, and DAHD, participated, focusing on disease identification and control.
India Launches Vishvasya Blockchain Stack and National Blockchain Portal to Boost Trust in Digital Services
India's Ministry of Electronics and Information Technology (MeitY) has introduced the Vishvasya Blockchain
Technology Stack, NBFLite, Praamaanik, and the National Blockchain Portal to enhance blockchain adoption and trust
in digital services.
Central to this initiative is the National Blockchain Framework (NBF), designed to build secure, transparent, and
efficient public services using distributed infrastructure, smart contracts, and strong privacy features. Collaborating
with institutions like C-DAC and IIT Hyderabad, this initiative aims to tackle challenges like skill shortages and vendor
lock-in while promoting research and development in blockchain technology.
Indian Army Celebrates Lance Naik Charan Singh’s 100th Birthday, Honouring His Service in World War II
Lance Naik Charan Singh (retired), a World War II veteran, was honoured by the Indian Army on his 100th birthday
in a special ceremony at his home in Himachal Pradesh.
Enlisting in 1942 at 18, Singh served 17 years in the Army Service Corps, including during World War II and the Burma
campaign, earning honours like the Burma Star Award.
The Indian Army, led by a Brigadier, celebrated his milestone birthday, reflecting its enduring respect for veterans.
Singh’s life and military service continue to inspire, particularly in his hometown, where his contributions are deeply
cherished.
164
Union Health Secretary Releases “Health Dynamics of India 2022-23” Report Highlighting Key Data on Healthcare
Infrastructure and Workforce
Union Health Secretary Shri Apurva Chandra released the “Health Dynamics of India (Infrastructure and Human
Resources) 2022-23” report, providing comprehensive data on India's healthcare facilities and workforce.
Formerly known as “Rural Health Statistics,” this report offers valuable insights to aid in policy-making under the
National Health Mission (NHM).
It details 1,69,615 Sub-Centres, 31,882 Primary Health Centres, 6,359 Community Health Centres, and 1,340 District
Hospitals, supported by over 2.39 lakh health workers.
Indian Army Participates in 5th Edition of Joint Military Exercise AL NAJAH with Oman
The Indian Army has sent a contingent to Oman for the 5th edition of the India-Oman Joint Military Exercise AL NAJAH,
being held from September 13 to 26, 2024, in Salalah, Oman.
This biennial exercise, which began in 2015, aims to strengthen military cooperation, focusing on counter-terrorism
operations.
The Indian Army’s 60-member team, primarily from a Mechanised Infantry Regiment, will join an equal number of
personnel from Oman’s Royal Army Frontier Force.
Union Minister Launches “Rangeen Machhli” App to Boost India's Ornamental Fisheries Sector
Union Minister Shri Rajiv Ranjan Singh launched the Rangeen Machhli mobile app in Bhubaneswar, aimed at
supporting India's growing ornamental fisheries sector under the Pradhan Mantri Matsya Sampada Yojana (PMMSY).
The app provides multilingual information on popular ornamental fish, helps users find local aquarium shops, and
offers educational modules on aquarium care and ornamental aquaculture.
Port Blair to be Renamed ‘Sri Vijaya Puram’ in Honor of India’s Historical Legacy
Union Minister Amit Shah announced that Port Blair, the capital of the Andaman and Nicobar Islands, will be renamed
Sri Vijaya Puram.
This renaming reflects Prime Minister Narendra Modi’s vision of replacing colonial-era names with those that honor
India’s rich history and freedom struggle.
The new name acknowledges the region's ancient connections, particularly to the Chola dynasty, which played a
significant role in Southeast Asia.
Typhoon Yagi: Asia’s Strongest Storm of 2024 Causes Over 400 Deaths, Massive Destruction Across Multiple
Countries
In early September 2024, Typhoon Yagi, the strongest storm of the year in Asia, struck Myanmar, Vietnam, Laos, and
Thailand, causing widespread devastation.
The typhoon resulted in over 400 deaths, forced hundreds of thousands from their homes, and led to severe flooding
and landslides.
165
The scheme will distribute funds in two installments: ₹5,000 on Rakhi Purnima and ₹5,000 on International Women’s
Day, deposited directly into Aadhaar-linked bank accounts. Over five years, it is expected to benefit 1 crore women,
with an allocation of ₹55,825 crore. This initiative seeks to attract women voters and compete with the BJD's Mission
Shakti programme.
Chennai Celebrates 146th Birth Anniversary of Social Reformer Periyar E.V. Ramasamy
On September 17, 2024, Chennai commemorated the 146th birth anniversary of Periyar E.V. Ramasamy, a renowned
social reformer from Tamil Nadu, with Chief Minister M.K. Stalin leading the tributes.
Born in 1879 in Erode, Periyar initially joined the Congress party but left due to disagreements over caste issues. He
founded the Self-Respect Movement in 1925 and gained national prominence during the Vaikom Satyagraha in 1924.
4th Global Renewable Energy Investors Meet and Expo Highlights Gujarat's Leadership
The 4th Global Renewable Energy Investors Meet and Expo was successfully held in Gujarat, marking the first
occurrence outside Delhi. Gujarat emerged as the top-performing state, excelling in various renewable energy
capacities, particularly in wind power and ranking second in solar power, with only Rajasthan surpassing it.
The event featured significant initiatives such as the “Mission 100 GW of Renewable Energy in Gujarat” and “Gujarat
Energy Vision 2047,” presented by Chief Minister Bhupendra Patel. Union Minister Pralhad Joshi acknowledged the
state’s contribution of about 53% to India’s solar generation capacity, further reinforcing Gujarat's critical role in the
nation’s renewable energy landscape.
INTERNATIONAL
Cyber Attack Disrupts Elon Musk's Interview with Donald Trump on X
Recently, an interview between Tesla CEO Elon Musk and former US President Donald Trump on the platform X
(formerly Twitter) was interrupted by a cyber attack known as Denial of Service (DoS).
Types of DoS attacks include the Smurf Attack, where attackers flood their own servers by using a spoofed IP address
of the target, and the SYN Flood Attack, where the attacker begins but does not complete multiple connection requests
to the server, blocking legitimate traffic.
UN General Assembly Introduces Multidimensional Vulnerability Index to Aid Small Island States
The UN General Assembly has recently launched the Multidimensional Vulnerability Index (MVI), a new quantitative
tool designed to assist small island developing states (SIDS) in obtaining low-interest financing.
The MVI comprises two main components, Universal Level Quantitative Assessment which provides a summary index
that ranks countries based on their structural vulnerability and resilience, using a unified methodology to produce an
overall MVI score and Vulnerability-Resilience Country Profiles which offer a detailed, tailored analysis of a country's
specific vulnerability and resilience factors.
ESA's JUICE Probe Successfully Executes Double Slingshot Maneuver to Journey Towards Jupiter
The European Space Agency's Jupiter Icy Moons Explorer (JUICE) Probe recently performed a critical double slingshot
manoeuvre, utilising the gravitational forces of both the Moon and Earth.
This intricate navigation sequence involved JUICE passing 434 miles from the Moon and then 4,229 miles from Earth,
leveraging these gravity assists to adjust its trajectory efficiently while conserving propellant.
166
This successful execution has set JUICE on a course to reach Jupiter by 2031, aided by additional gravity assists from
Venus in 2025 and Earth in 2026 and 2029.
JUICE aims to explore Jupiter’s three large icy moons—Callisto, Europa, and Ganymede—with the ultimate goal of
orbiting Ganymede to assess its potential to support life, building on the legacy of NASA’s Galileo mission and other
significant Jupiter missions like Juno and Cassini-Huygens.
Telegram CEO Pavel Durov Arrested in France Amid Privacy Concerns and Legal Scrutiny
Pavel Durov, the CEO and co-founder of Telegram, was arrested at Le Bourget airport in Paris, linked to a French
investigation into the app’s role in enabling illegal activities such as drug trafficking and cyberstalking through its
privacy features.
A Russian expatriate and staunch privacy advocate, Durov created Telegram in 2013 to offer secure communication,
distinguishing the platform with strong encryption and large group chat capabilities.
Pacific Islands Forum 2024 Tackles Climate Change and US-China Rivalry in Tonga
The Pacific Islands Forum (PIF) convened in Nuku'alofa, Tonga, on August 26, 2024, drawing over 1,500 delegates
from approximately 40 countries to discuss pressing issues like climate change and the strategic tussle between China
and the US in the Pacific.
Founded in 1971, the PIF comprises 18 Pacific nations including Australia, New Zealand, and Fiji, focusing on regional
growth, security, and cooperation.
This year's key agenda includes addressing the dire impacts of climate change, with Pacific nations facing existential
threats from rising sea levels and a significant funding gap in the Pacific Resilience Facility.
Berkshire Hathaway Hits $1 Trillion Market Cap, Led by Warren Buffett's Strategic Decisions
Berkshire Hathaway, under the leadership of Warren Buffett, reached a landmark $1 trillion market value on August
28, becoming the first non-tech company to achieve this milestone.
This historic valuation followed Buffett's strategic move to sell nearly half of Berkshire's Apple shares, accumulating
a massive $280 billion cash reserve. Buffett, who began his career as an investment analyst, co-founded Berkshire
with Charlie Munger in 1965, transitioning it from a textile company to a diversified conglomerate.
167
Angkor Wat Named Most Photogenic UNESCO World Heritage Site in Asia
Cambodian Prime Minister Hun Manet announced that Angkor Wat has been recognized as the most photogenic
UNESCO World Heritage site in Asia by Times Travel, as reported by the Times of India.
The list includes other iconic sites such as the Taj Mahal and Hampi in India, the Great Wall of China, the Ancient City
of Bagan in Myanmar, Borobudur in Indonesia, Ha Long Bay in Vietnam, Historic Monuments of Kyoto in Japan, Petra
in Jordan, and the Rice Terraces of the Philippine Cordilleras.
Astronomers Discover Brightest Quasar in the Universe, Providing Insight into Cosmic Beginnings
Astronomers using the Very Large Telescope have identified J0529-4351, the brightest quasar ever observed, offering
unprecedented clarity into the early universe.
Located in a galaxy with a supermassive black hole, this quasar outshines the Sun by over 500 trillion times and
expands at an extraordinary rate, absorbing one solar mass per day.
Situated more than 12 billion light-years away, J0529-4351's light provides a snapshot of cosmic conditions billions
of years ago, with its vast accretion disk spanning seven light-years.
168
Despite achieving a soft landing on the Moon in January 2024, which marked Japan as the fifth country to accomplish
this, subsequent communication failures led to the cessation of its mission. This event follows other successful lunar
landing by nations like India, Russia, the USA, and China.
DRDO and Indian Navy Successfully Test Vertical Launch Short Range Surface-to-Air Missile (VL-SRSAM)
The Defence Research & Development Organisation (DRDO) and the Indian Navy have successfully flight-tested the
Vertical Launch Short Range Surface-to-Air Missile (VL-SRSAM) at the Integrated Test Range in Chandipur, Odisha.
The missile, launched from a land-based vertical launcher, accurately intercepted a high-speed aerial target,
demonstrating its capability to defend against airborne threats.
Indian Immunologicals and ICMR Collaborate to Develop India's First Zika Virus Vaccine
Indian Immunologicals Limited (IIL) has partnered with the Indian Council of Medical Research (ICMR) to develop
India's first Zika virus vaccine. The Memorandum of Agreement (MoA) outlines plans for phase I clinical trials, funded
by ICMR, to test the vaccine’s safety and efficacy at four key locations in India, including ACTREC Mumbai and PGIMER
Chandigarh.
IIL is also collaborating with Griffith University, Australia, and has completed pre-clinical tests. Besides Zika, IIL is
working on vaccines for Kyasanur Forest Disease, Chikungunya, and a SARS-CoV-2 nasal booster. The Zika virus,
primarily spread by mosquitoes, currently has no licensed vaccine, making this development crucial for public health.
Asteroid 2024 PT5: Earth’s Temporary Mini-Moon from September to November 2024
From September 29 to November 25, 2024, Earth will gain a temporary companion—a mini-moon named asteroid
2024 PT5.
This small asteroid, about 10 meters in size, will be captured by Earth’s gravity, briefly orbiting alongside our
permanent Moon.
Discovered by NASA’s ATLAS program, this mini-moon will follow a unique horseshoe-shaped orbit before moving
away.
169
Notably, Raina is only the second non-IPS officer to achieve this rank within the ITBP. Raina's tenure will be brief, as
he is set to retire in over a month, succeeded by IG Jaspal Singh.
TV Somanathan Appointed as New Cabinet Secretary of India Following Rajiv Gauba’s Retirement
TV Somanathan, a seasoned IAS officer from the 1987 Tamil Nadu cadre, has been appointed as India’s new Cabinet
Secretary on August 10, 2024, following the retirement of Rajiv Gauba, who served in the position for five years.
Somanathan, previously the Union Finance Secretary, brings a wealth of experience in finance and administration,
having held prominent roles, including Joint Secretary in the Prime Minister's Office and Director for Corporate Affairs
at the World Bank.
Amrit Mohan Prasad Appointed New Director General of Sashastra Seema Bal
Amrit Mohan Prasad, an Indian Police Service (IPS) officer from the Odisha cadre, has been appointed as the Director
General (DG) of the Sashastra Seema Bal (SSB), as approved by the Appointments Committee of the Cabinet. Prasad
previously served as the Special Director General of the Central Reserve Police Force (CRPF).
Atishi Marlena Appointed as New Chief Minister of Delhi Following Kejriwal's Resignation
Arvind Kejriwal has resigned as Chief Minister of Delhi, paving the way for Atishi Marlena to take over leadership of
the Aam Aadmi Party (AAP).
Born into an academic family, Atishi is an alumnus of St. Stephen’s College and has pursued further studies at Oxford
University on prestigious scholarships.
In 2018, she dropped "Marlena" from her public name to focus on her professional identity as a leader in social work
and education.
170
BOOKS & AUTHORS
Tarun Chugh’s Book "Modi’s Governance Triumph" Released in New Delhi
BJP General Secretary Tarun Chugh's book, Modi’s Governance Triumph: Reshaping India’s Path to Prosperity, was
launched in New Delhi, attended by Home Minister Amit Shah and Coal Minister G. Kishan Reddy.
The book analyzes key initiatives like Make in India and GST, highlighting Modi's policies on poverty alleviation,
economic disparity, and national security. It also delves into the Atmanirbhar Bharat vision, challenges faced by the
administration, and the ambitious outlook for India's future as a global power by 2047.
IAF Chief Launches Comic Book Series Celebrating Air Force Heroes
Air Chief Marshal V R Chaudhari launched the first comic book in a series highlighting the Indian Air Force's rich
history and heroic deeds. Aimed at inspiring youth, the book features stories of Marshal of the Air Force Arjan Singh
DFC and the 1971 Battle of Boyra. It also provides career information for aspiring IAF personnel.
The comic will be freely distributed during induction drives, with a digital version releasing on Air Force Day, October
8. This initiative bridges generational gaps and promotes Air Force careers through engaging storytelling.
IMPORTANT DAYS
September 10, 2024, World Suicide Prevention Day
On September 10, 2024, the world will observe World Suicide Prevention Day, focusing on raising awareness about
suicide prevention and reducing stigma surrounding mental health. Organized by the International Association for
Suicide Prevention (IASP) in collaboration with the World Health Organization (WHO), the event aims to inspire
action, educate the public, and offer hope. The theme for World Suicide Prevention Day 2024-2026 is “Changing the
Narrative on Suicide” with the call to action “Start the Conversation”.
171
The International Association for Suicide Prevention (IASP) first established World Suicide Prevention Day in 2003.
National Suicide Prevention Strategy (NSPS) launched by the Ministry of Health and Family Welfare in 2022, the NSPS
is India's first suicide prevention policy aimed at reducing suicide mortality by 10% by 2030.
172
September 1-7: National Nutrition Week (Rashtriya Poshan Maah)
National Nutrition Week, observed annually from September 1 to September 7 in India, focuses on the significance of
a nutritious diet for physical and mental well-being. First observed in 1982, it promotes awareness about the dangers
of fast food and encourages healthy eating habits.
Originally initiated by the American Dietetic Association in 1975, India adopted this observance to spread knowledge
about nutrition’s role in longevity and vitality. The 2023 theme, “Healthy Diet Gawing Affordable for All,” as per
UNICEF, emphasises making nutritious food accessible to everyone.
173
The 2024 theme, “Montreal Protocol: Advancing Climate Action". The ozone layer, vital for shielding life on Earth from
harmful ultraviolet radiation, has been gradually healing, with projections for full recovery in mid-latitudes by the
2030s and in polar regions by 2060.
OBITUARIES
Rakesh Pal, Director General of the Indian Coast Guard
Rakesh Pal, who served as the 25th Director General of the Indian Coast Guard, passed away at the age of 59 due to a
severe cardiac arrest in Chennai. He was in the city to attend a Coast Guard event led by Defence Minister Rajnath
Singh.
Pal, a flag officer from Uttar Pradesh, joined the Indian Coast Guard in January 1989 after attending the Indian Naval
Academy. Over his 34-year career, he played a crucial role in modernizing the Coast Guard, earning the Vishist Seva
medal for his contributions. He became Director General in July 2023, leading the maritime law enforcement agency
which operates under the Ministry of Defence.
Dr. Ram Narain Agarwal, Renowned Indian Scientist and ‘Father of Agni Missiles’
Dr. Ram Narain Agarwal, a pioneering figure in India’s missile program, passed away at the age of 84 in Hyderabad.
Fondly known as the 'Father of Agni Missiles,' Dr. Agarwal's revolutionary contributions spanned over 22 years in
defence research, where he collaborated with luminaries like Dr. APJ Abdul Kalam.
He played a crucial role in developing missile technologies such as re-entry systems, heat shields, and guidance
systems. His leadership in the Agni missile program, especially Agni V, significantly boosted India's strategic defence
capabilities. Dr. Agarwal also founded the Advanced Systems Laboratory (ASL), leaving behind a legacy of innovation
and excellence in missile technology. He was honoured with several prestigious awards, including the Padma Shri,
Padma Bhushan, and Lifetime Achievement Award.
174
Death of Māori King Marks End of an Era in New Zealand
Kiingi Tuheitia Pootatau Te Wherowhero VII, the Māori King, passed away shortly after celebrating 18 years of
leadership. He was the seventh monarch of the Kiingitanga movement, initiated in 1858 to unify New Zealand's
indigenous Māori tribes against British colonisation.
The movement aimed to halt land sales to non-indigenous people and cease inter-tribal conflicts. Throughout New
Zealand’s history, significant clashes like the Wairau Valley incident and the Waikato War marked the struggle
between Māori and European settlers for land and sovereignty.
RANKING
Global Air Quality Study Reveals Life Expectancy Gains from Pollution Reduction
The 2024 Air Quality Life Index by the University of Chicago highlights a critical global health issue: reducing PM2.5
levels to WHO standards could extend global life expectancy by 14.9 billion years. In India, over 40% of the population
breathes air that surpasses the national standard of 40 µg/m³, with potential gains of 1.9 additional years per person
if WHO guidelines are met.
The report stresses uneven pollution distribution, with severe impacts in regions like South Asia and significant
improvements in countries like the US and China through rigorous policies. Compliance remains a challenge globally,
affecting life expectancy and health.
Rajiv Gandhi International Airport Named Best Airport for Third Consecutive Year
Rajiv Gandhi International Airport (RGIA) in Hyderabad has clinched the title of Best Airport at the India Travel
Awards for the third consecutive year, underscoring its commitment to superior passenger service.
This accolade, complemented by the Skytrax award for ‘Best Airport Staff in India & South Asia 2024,’ highlights RGIA's
consistent quality and its pivotal role in positioning Hyderabad as a key hub in South Asian air travel.
NHPC, SJVN, and RailTel Attain Prestigious Navratna Status, Expanding Investment Autonomy
NHPC, SJVN, and RailTel have been awarded Navratna status by the Government of India, marking a significant
milestone for these public sector enterprises.
This recognition, previously held by 17 companies, allows them greater operational freedom, including investment
autonomy of up to ₹1,000 crore per project. The status, earned through strong financial performance and high MoU
ratings, enables these companies to form joint ventures, set up subsidiaries abroad, and invest further in their sectors.
Telangana, Tamil Nadu, and Rajasthan Emerge as Fastest-Growing Large States in India
The Ministry of Statistics and Programme Implementation (MoSPI) has recognized Telangana, Tamil Nadu, and
Rajasthan as the fastest-growing large states in India, with each surpassing the national average GDP growth of 8.2%.
Telangana led the way with a 9.2% growth, reaching a GSDP of ₹7.9 lakh crore, followed by Tamil Nadu at 8.2% (₹15.7
lakh crore), and Rajasthan at 8%.
India Achieves Tier 1 Status in Global Cybersecurity Index 2024, Ranking Among World's Best
India has achieved Tier 1 status in the Global Cybersecurity Index (GCI) 2024, a recognition of its commitment to
building a robust cybersecurity infrastructure.
The GCI, developed by the International Telecommunication Union (ITU), evaluates countries based on legal, technical,
organisational, capacity development, and cooperative measures to ensure online security. India’s impressive score
of 98.49 reflects its strong legal framework, such as the Information Technology Act (2000) and the Digital Personal
Data Protection Bill (2022), placing it among the top 47 nations globally for cybersecurity practices.
175
Its dominance in India’s dairy market and strong metrics in familiarity and recommendation contributed to its AAA+
rating. Hershey’s, with a $3.9 billion valuation, took second place. Despite a 4% drop in the broader food sector’s brand
value, Amul continues to thrive.
Mumbai, Delhi Rank Among Top 3 Cities Globally for Prime Residential Price Growth
According to Knight Frank's report, Mumbai and Delhi ranked second and third globally for annual price rise in prime
residential properties in the June 2024 quarter. Mumbai saw a 13% increase, up from 6th place a year ago, while New
Delhi jumped from 26th to third with a 10.6% rise.
Bengaluru ranked 15th with a 3.7% increase. Globally, Manila topped the list with a 26% rise, while Los Angeles and
Miami ranked 4th and 5th. Cities like Vienna and Bangkok saw declines, with Wellington ranking last.
Mumbai Named Billionaire Capital of Asia in Hurun India Rich List 2024
The Hurun India Rich List 2024 has named Mumbai as Asia’s new billionaire capital, surpassing Beijing. Mumbai now
hosts 386 billionaires, up by 58, with a total wealth of $445 billion. This positions Mumbai third globally behind New
York (119) and London (97). Delhi ranks second in India with 217 billionaires, while Hyderabad (104) overtakes
Bengaluru (100).
Mumbai’s rise is attributed to its robust financial sector and entrepreneurial ecosystem. Beijing's decline, with 18
fewer billionaires, reflects broader economic challenges in China.
Switzerland Tops "Best Countries for 2024" Survey for Third Consecutive Year
In the “Best Countries for 2024” survey, Switzerland ranked as the best country for the third year in a row, out of 89
countries.
The survey, which assessed categories like adventure, entrepreneurship, and quality of life, saw European countries
dominate the top 25. Switzerland was followed by Japan, the USA, Canada, and Australia.
India ranked 33rd, dropping three spots from last year, performing best in the "movers" and heritage categories, but
scoring low in social purpose and adventure. The UAE and Qatar were the only Middle Eastern countries in the top 25.
176
o X (formerly Twitter): 113 million
o YouTube: 60.6 million
SPORTS
Vithya Ramraj Shatters P.T. Usha’s 39-Year-Old Record at the 63rd National Open Athletics Championships
At the National Open Athletics Championships in Bengaluru, Vithya Ramraj broke a nearly four-decade-old Indian
record in the women's 400m hurdles, previously held by the iconic P.T. Usha. Ramraj clocked in at 56.23 seconds,
besting Usha's 1985 record of 56.80 seconds.
This historic achievement underscores a significant moment in Indian athletics, spotlighting Ramraj’s rising
prominence. The event also witnessed standout performances across various disciplines, with Tamil Nadu's Nithin
breaking the men’s 200m meet record. Ancy Sojan earned the best woman athlete title with a remarkable long jump
of 6.71 metres.
Women's T20 World Cup 2024 Relocated from Bangladesh to UAE Due to Safety Concerns
The ICC has announced the relocation of the 2024 Women's T20 World Cup from Bangladesh to the UAE, set to take
place from October 3 to 20 in Dubai and Sharjah.
Political unrest in Bangladesh, following Prime Minister Sheikh Hasina’s resignation and ongoing turmoil, led to safety
concerns and travel advisories from several participating nations.
Jannik Sinner and Aryna Sabalenka Win First Cincinnati Open Titles
Top-ranked Jannik Sinner and women’s No. 2 Aryna Sabalenka claimed their first Cincinnati Open titles with straight-
set victories. Sabalenka defeated Jessica Pegula 6-3, 7-5, securing her 15th WTA title, while Sinner triumphed over
Frances Tiafoe 7-6 (4), 6-2, becoming the youngest Cincinnati champion since Andy Murray in 2008.
Sabalenka’s win reinforces her as a U.S. Open contender despite recent injuries, while Sinner looks forward to the U.S.
Open after his milestone victory. Tiafoe and Pegula faced challenges in their respective finals, with errors and fatigue
contributing to their defeats.
Neeraj Chopra Secures Second Place at Lausanne Diamond League with Season's Best Throw
Neeraj Chopra, India’s star javelin thrower, claimed a second-place finish at the Lausanne Diamond League,
showcasing his resilience.
Despite initial struggles, Chopra surged in the final rounds, delivering an 89.49m throw in the sixth round—his
season’s best, surpassing his Paris Olympics throw. Grenada’s Anderson Peters won gold with a 90.61m throw, while
177
Germany's Julian Weber secured bronze. Battling a groin injury, Chopra’s performance earned him 15 points in the
Diamond League standings, tying him for third place.
Diana Pundole Becomes First Indian Woman to Win National Car Racing Championship
Diana Pundole, a teacher and mother from Pune, made history by becoming the first Indian woman to win the national
championship in the saloon category at the MRF Indian National Car Racing Championship 2024 in Chennai.
Achieving this milestone on her birthday weekend, Diana overcame personal and societal challenges in a male-
dominated sport. Her racing career, fueled by a lifelong passion, includes competing internationally at prestigious
tracks like Dubai Autodrome and Hockenheimring.
Lando Norris Triumphs at Dutch Grand Prix, Secures Second Career Win
Lando Norris delivered an exceptional performance at the Dutch Grand Prix, claiming his second career victory with
a commanding lead over Max Verstappen. Despite losing the lead early, Norris executed a decisive overtake to regain
control, finishing over 20 seconds ahead of Verstappen, who had to settle for second on home turf. Ferrari’s Charles
Leclerc secured third place, marking a strong result for the team. McLaren enjoyed mixed fortunes with Oscar Piastri
finishing fourth, while Carlos Sainz earned fifth for Ferrari.
Siddhartha Aggarwal Becomes Oldest Indian to Swim Solo Across English Channel
On August 29, 49-year-old Siddhartha Aggarwal from Bengaluru became the oldest Indian to swim solo across the
English Channel, completing the 42 km swim in 15 hours and six minutes. His achievement surpasses the previous
record set by Srikaanth Viswanathan at age 46.
Aggarwal’s journey began in 2018 with a relay swim across the Channel, and after 15 months of rigorous training
under coach Satish Kumar, he accomplished this solo feat despite challenging tides and winds. His remarkable
dedication serves as an inspiration for swimmers of all ages.
India Announces Bid for 2030 Youth Olympics at OCA General Assembly
India's Union Minister of Youth Affairs & Sports, Dr. Mansukh Mandaviya, announced India's bid to host the 2030
Youth Olympics at the Olympic Council of Asia's (OCA) 44th General Assembly on September 8, 2024.
Competing against countries like Peru, Mexico, and Thailand, this bid aligns with India’s long-term goal of hosting the
2036 Olympics. Dr. Mandaviya highlighted India's sports initiatives, such as the ‘Khelo India’ scheme and the ‘Target
Olympic Podium Scheme (TOPS)’, which have significantly boosted the country’s sports infrastructure and athlete
development. The ASMITA program, promoting women's participation in sports, was also spotlighted.
178
Jannik Sinner Wins U.S. Open, Securing His Second Grand Slam Title
World No. 1 Jannik Sinner clinched his second Grand Slam title at the U.S. Open, defeating Taylor Fritz in a commanding
final. Despite being cleared of a doping controversy just before the tournament, Sinner overcame doubts and delivered
a dominant performance, showcasing his powerful baseline game.
His victory not only reaffirmed his top ranking but also ended a long Grand Slam drought for American men, with the
last U.S. winner being Andy Roddick in 2003. Fritz, playing in his first Grand Slam final, fell short but remains optimistic
about his future in the sport.
Saleema Imtiaz Becomes First Pakistani Woman Nominated to ICC Development Umpires Panel
Saleema Imtiaz has made history as the first Pakistani woman to be nominated to the ICC International Panel of
Development Umpires, as announced by the Pakistan Cricket Board (PCB). Imtiaz, who began her umpiring career
with the PCB women’s panel in 2008, has officiated in notable events like the 2022 Asia Cup and the 2023 ACC
Emerging Women’s Cup.
With 22 T20Is under her belt, her nomination allows her to officiate in women’s bilateral international matches and
ICC events. Imtiaz’s achievement symbolises a growing influence of women in cricket and the PCB's commitment to
gender development in the sport.
India Clinches Fifth Men’s Asian Champions Trophy Title with Jugraj Singh’s Winning Goal
India’s Men’s Hockey Team secured their fifth Asian Champions Trophy title in 2024 by defeating hosts China 1-0 in a
tense final. Jugraj Singh scored the decisive goal with an assist from Harmanpreet Singh, breaking the deadlock after
China’s strong defensive effort in the first half. India dominated possession but faced challenges from China’s
counterattacks, keeping the scoreboard at 0-0 until Jugraj’s second-half strike secured the win.
179
investment to enhance the cross-border defence innovation ecosystem, continuing the momentum from its inception
in June 2023.
India and South Africa Sign Agreement to Boost Submarine Rescue Capabilities
On September 4, 2024, the Indian and South African navies formalised an agreement to enhance submarine rescue
support, ensuring South African submarine crew safety during emergencies.
The agreement grants South Africa access to India's Deep Submergence Rescue Vehicle (DSRV), capable of rescuing
submariners from depths of up to 650 metres. This partnership, initiated after a 2023 accident that claimed the lives
of three South African navy members, reflects a commitment to maritime safety.
India and Malaysia Sign MoU on Public Administration and Governance Reforms
The Department of Administrative Reforms and Public Grievances (DARPG) of India and the Public Service
Department of Malaysia signed a Memorandum of Understanding (MoU) on August 20, 2024, for cooperation in public
administration and governance reforms.
This five-year agreement focuses on citizen-centric services, e-governance, transparency, and public grievance
redress mechanisms. The MoU was exchanged during a ceremonial function attended by Indian Prime Minister
Narendra Modi and Malaysian Prime Minister Anwar Bin Ibrahim at Hyderabad House, New Delhi.
Tata Power Solar Systems Ltd Signs Financing Agreement with ICICI Bank
Tata Power Solar Systems Ltd (TPSSL) has partnered with ICICI Bank to offer loans for solar unit financing to
residential and corporate customers. Customers can avail loans up to Rs 90 lakh, with collateral-free options and
repayment tenures of up to 5 years.
Additionally, higher loan amounts with collaterals are available for up to 20-year tenures. This initiative, part of
TPSSL's efforts to expand solar energy adoption, is set to make solar panel purchases more accessible to a broader
customer base.
180
54th GST Council Meeting Proposes Tax Cuts on Cancer Drugs and Health Insurance, Considers Adjustments for
Other Goods
The 54th Goods and Services Tax (GST) Council meeting, chaired by Finance Minister Nirmala Sitharaman in New
Delhi, focused on reducing tax rates for essential items such as cancer drugs, health insurance, and savoury snacks.
Key cancer drugs like Trastuzumab Deruxtecan and Osimertinib may see GST reductions from 12% to 5%, easing
treatment costs for patients.
RBI Penalizes UCO Bank and Cent Bank Home Finance Ltd for Regulatory Non-Compliance
The Reserve Bank of India (RBI) has imposed penalties on two financial institutions for regulatory violations. UCO
Bank was fined Rs 2.68 crore for breaches related to current account openings, deposit interest rates, and fraud
classification.
Cent Bank Home Finance Ltd faces a Rs 2.1 lakh penalty for non-compliance with Know Your Customer (KYC)
regulations.
181
RBI Fines HDFC Bank and Axis Bank for Regulatory Non-Compliance
The Reserve Bank of India (RBI) has imposed penalties on HDFC Bank (₹1.91 crore) and Axis Bank (₹1 crore) for
violations related to deposit regulations, KYC norms, and agricultural loans.
HDFC Bank faced penalties for non-compliance in areas like deposit interest rates, recovery agent conduct, and
operational issues. Axis Bank was fined for issuing multiple customer identification codes, collateral violations on
agricultural loans, and subsidiary activities outside permitted regulations.
Union Bank of India Joins Partnership for Carbon Accounting Financials (PCAF)
Union Bank of India has become the first major Indian bank to join the Partnership for Carbon Accounting Financials
(PCAF), reflecting a global focus on managing climate risks. PCAF provides a standardised approach for financial
institutions to assess and disclose greenhouse gas (GHG) emissions tied to their loans and investments, often referred
to as Scope 3 emissions.
This move aligns with the RBI’s draft guidelines on climate risk disclosures, which emphasise governance, strategy,
and risk management for regulated entities.
LIC Selects Infosys for Next-Gen Digital Platform under DIVE Programme
The Life Insurance Corporation of India (LIC) has partnered with Infosys to build a next-generation digital platform
as part of its DIVE (Digital Innovation and Value Enhancement) programme.
This platform will enhance customer services, business lifecycle management, and streamline operations for sales
intermediaries. It marks a strategic shift for LIC from previous IT collaborations with Tata Consultancy Services (TCS)
and Tech Mahindra. The platform will be cloud-native, modular, and designed to integrate emerging technologies.
182
IMPORTANT ONE LINERS
NATIONAL
Uttarakhand Pioneers in Valuing Natural Resources with Gross Environment Product
Uttarakhand has become the first state in India to assign monetary values to its natural resources,
encompassing air, water, forest, and soil, introducing the Gross Environment Product (GEP).
The GEP, distinguishing between man-made conservation efforts like 'amrit sarovars' and natural processes
such as rainfall and reflects a 0.9% increase in environmental products from 2020 to 2022, highlighting the state's
commitment to integrating ecological health into its economic metrics.
Natal Indian Congress Marks 130th Foundation Year: A Legacy of Resistance and Unity
The Natal Indian Congress (NIC), established on August 22, 1894, by Mahatma Gandhi, recently marked its 130th
foundation year.
Originally founded to combat discrimination against Indians in Natal, South Africa, the NIC was the first Indian
Congress of its kind.
Over the decades, it evolved under the umbrella of the South African Indian Congress (SAIC) from the 1920s and saw
a shift towards more radical leadership in the 1930s and 1940s, culminating in Dr. G.M. Naicker’s leadership started
in 1945.
The NIC’s activism intensified, leading to the imprisonment of several leaders by the 1950s and 1960s.
176
State of Healthcare in Rural India, 2024" Report Highlights Critical Gaps
The "State of Healthcare in Rural India, 2024" report highlights significant healthcare challenges in rural areas
across 21 Indian states.
Approximately 50% of rural households have government health insurance, yet 34% lack any insurance, and 61% are
without life insurance. Access issues persist, with only 39% having nearby diagnostic facilities and a mere 12.2%
accessing subsidised medicines.
Additionally, 73% of households with elderly require constant care, predominantly provided by family members,
underscoring the need for enhanced caregiver support and infrastructure improvements.
India Launches Jan Poshan Kendra Initiative to Boost Nutrition and Efficiency in Public Distribution
The Indian government has initiated a pilot project to transform 60 Fair Price Shops (FPS) into "Jan Poshan
Kendras" across Gujarat, Rajasthan, Telangana, and Uttar Pradesh.
This transformation is part of an effort to enhance the nutritional offerings available under the Pradhan Mantri Garib
Kalyan Anna Yojana (PMGKAY). The Jan Poshan Kendra initiative not only aims to sell additional items beyond
subsidised grains, addressing the income challenges of ration dealers, but also to improve service quality with the
introduction of digital tools and support systems to increase transparency and efficiency.
Presently, 0.54 million FPSs distribute about 60-70 million tonnes of food grain annually to over 800 million
beneficiaries.
The government plans to expand this initiative, converting approximately 100,000 fair price shops into "nutri-hubs"
over the next three years, aiming to further enhance the nutritional access and efficiency of India's vast public
distribution system.
Union Minister of Power Launches Three Innovative Online Platforms to Boost Power Sector Efficiency
The Union Minister of Power has launched three online platforms—PROMPT, DRIPS, and JALVIDYUT DPR—to
enhance efficiency, transparency, and effectiveness in India's power sector.
Developed by the Central Electricity Authority (CEA) and NTPC, PROMPT facilitates real-time tracking of thermal
power projects, DRIPS addresses power disruptions from natural disasters, and JAL VIDYUT DPR monitors
hydroelectric projects.
These platforms aim to improve project management, disaster response, and coordination across the sector, reflecting
the Ministry's commitment to modernising infrastructure management and increasing operational transparency.
Indian Prime Minister's Historic Visit to Ukraine: Strengthening Bilateral Ties and Promoting Peace
India's Prime Minister visited Ukraine, marking the first visit by an Indian head of state since Ukraine's independence
in 1991.
The visit highlighted India's active stance on the Russia-Ukraine conflict, emphasising peace and resolution through
stakeholder engagement.
Key outcomes included the establishment of an Intergovernmental Commission to enhance bilateral trade, the signing
of four agreements aimed at strengthening cooperation in agriculture, medical regulation, and cultural ties, and the
donation of four mobile hospital units to Ukraine.
177
Additionally, the Prime Minister extended an invitation to the Ukrainian President to visit India, reinforcing bilateral
relations.
178
The 51st Fighter Wing at Osan Air Base, South Korea, is actively participating in the Ulchi Freedom Shield 24, a joint
military exercise with South Korean forces running from August 19 to August 23, 2024.
The exercise aims to boost the wing's operational readiness and strengthen command and control systems in
collaboration with the Seventh Air Force and U.S. Forces Korea.
Key activities include local flying operations and Agile Combat Employment, emphasizing rapid response and
adaptability across various bases.
This strategic training underscores the U.S.'s commitment to regional security and stability in the Pacific, reinforcing
a combined defense posture with South Korea.
India-Denmark Partnership Launches Smart Laboratory on Clean Rivers to Revitalize Varuna River
Under the collaborative Environmental Strategic Partnership between India and Denmark, the Smart Laboratory on
Clean Rivers (SLCR) project has been initiated to rejuvenate the Varuna River, with a combined funding of ₹21.8 crore.
This innovative project aims to leverage advanced technology and a comprehensive river management plan to
enhance the river’s health and bolster the local economy and social conditions over the next 2-3 years.
IIM Bangalore Partners with Alumnus Mathew Cyriac to Launch India’s First Global Centre for PE and VC
On August 25, 2024, the Indian Institute of Management Bangalore (IIM-B) signed a landmark Memorandum of
Understanding with alumnus Mathew Cyriac, Executive Chairman of Florintree Advisors, to establish India’s inaugural
Global Centre of Excellence in Private Equity and Venture Capital.
Named after prominent investment banker Tony James, the Centre aims to be a global leader in these fields, fostering
advanced research and educational endeavours.
ASSOCHAM Environment and Carbon Conference Opens in New Delhi with Focus on Achieving Net Zero by 2070
Shri Kirti Vardhan Singh, the Union Minister of State for Environment, Forest, and Climate Change, inaugurated the
ASSOCHAM Environment and Carbon Conference in New Delhi, emphasising collaborative efforts to achieve India's
goal of net zero emissions by 2070.
Highlighting climate change as a global challenge requiring international cooperation, the Minister outlined India’s
commitment to bolstering climate resilience and fulfilling its Nationally Determined Contributions.
Assam Assembly Passes Law to Mandate Registration of Muslim Marriages and Divorces, Aiming to Curb Child
Marriage and Polygamy
The Assam Legislative Assembly has enacted the Assam Compulsory Registration of Muslim Marriages and Divorces
Bill, 2024, repealing the outdated 1935 Act that allowed child marriage and marriages without mutual consent.
This new legislation sets stringent age and consent requirements for Muslim marriages in Assam, with a woman
needing to be at least 18 and a man 21. Couples must now register their intent to marry 30 days in advance, allowing
time for any legal objections.
Google DeepMind's "Morni Project" Aims to Develop AI for 125 Indian Languages
179
Google DeepMind's India-based "Morni Project" is pioneering AI development to comprehend and interact with 125
Indian languages, enhancing digital inclusivity across India's diverse linguistic landscape.
Despite India's recognition of 22 official languages, Morni seeks to support over 100 additional languages, some
spoken by massive, yet digitally underrepresented populations.
Faced with the significant challenge of scarce digital data for 73 of these languages, Google has initiated Project Vaani
in collaboration with the Indian Institute of Science and ARTPARK.
Prime Minister Modi Inaugurates Vadhvan Port Project in Maharashtra, Aiming for Global Top Ten Status
Prime Minister Narendra Modi laid the foundation stone for the ambitious Vadhvan Port project in Palghar,
Maharashtra, on August 30, 2024.
With an estimated cost of ₹76,000 crore, the port is designed to handle 23.2 million TEUs and aims to rank among the
world's top ten ports.
Strategically located near Dahanu at a deep-water site, Vadhvan Port is set to enhance India's trade capabilities by
facilitating large container ships and reducing shipping costs and times.
India Conducts First National Mock Drill on Zoonotic Diseases, "Vishanu Yuddh Abhyas"
India held its first national mock drill, Vishanu Yuddh Abhyas, from August 27 to 31, 2024, to assess preparedness for
zoonotic disease outbreaks. Initiated under the National One Health Mission (NOHM), the exercise aimed to test
coordination between sectors like human health, animal husbandry, and wildlife management. Key organisations,
including the NCDC, ICMR, and DAHD, participated, focusing on disease identification and control.
India Launches Vishvasya Blockchain Stack and National Blockchain Portal to Boost Trust in Digital Services
India's Ministry of Electronics and Information Technology (MeitY) has introduced the Vishvasya Blockchain
Technology Stack, NBFLite, Praamaanik, and the National Blockchain Portal to enhance blockchain adoption and trust
in digital services.
Central to this initiative is the National Blockchain Framework (NBF), designed to build secure, transparent, and
efficient public services using distributed infrastructure, smart contracts, and strong privacy features. Collaborating
with institutions like C-DAC and IIT Hyderabad, this initiative aims to tackle challenges like skill shortages and vendor
lock-in while promoting research and development in blockchain technology.
Indian Army Celebrates Lance Naik Charan Singh’s 100th Birthday, Honouring His Service in World War II
Lance Naik Charan Singh (retired), a World War II veteran, was honoured by the Indian Army on his 100th birthday
in a special ceremony at his home in Himachal Pradesh.
Enlisting in 1942 at 18, Singh served 17 years in the Army Service Corps, including during World War II and the Burma
campaign, earning honours like the Burma Star Award.
The Indian Army, led by a Brigadier, celebrated his milestone birthday, reflecting its enduring respect for veterans.
Singh’s life and military service continue to inspire, particularly in his hometown, where his contributions are deeply
cherished.
180
Union Health Secretary Releases “Health Dynamics of India 2022-23” Report Highlighting Key Data on Healthcare
Infrastructure and Workforce
Union Health Secretary Shri Apurva Chandra released the “Health Dynamics of India (Infrastructure and Human
Resources) 2022-23” report, providing comprehensive data on India's healthcare facilities and workforce.
Formerly known as “Rural Health Statistics,” this report offers valuable insights to aid in policy-making under the
National Health Mission (NHM).
It details 1,69,615 Sub-Centres, 31,882 Primary Health Centres, 6,359 Community Health Centres, and 1,340 District
Hospitals, supported by over 2.39 lakh health workers.
Indian Army Participates in 5th Edition of Joint Military Exercise AL NAJAH with Oman
The Indian Army has sent a contingent to Oman for the 5th edition of the India-Oman Joint Military Exercise AL NAJAH,
being held from September 13 to 26, 2024, in Salalah, Oman.
This biennial exercise, which began in 2015, aims to strengthen military cooperation, focusing on counter-terrorism
operations.
The Indian Army’s 60-member team, primarily from a Mechanised Infantry Regiment, will join an equal number of
personnel from Oman’s Royal Army Frontier Force.
Union Minister Launches “Rangeen Machhli” App to Boost India's Ornamental Fisheries Sector
Union Minister Shri Rajiv Ranjan Singh launched the Rangeen Machhli mobile app in Bhubaneswar, aimed at
supporting India's growing ornamental fisheries sector under the Pradhan Mantri Matsya Sampada Yojana (PMMSY).
The app provides multilingual information on popular ornamental fish, helps users find local aquarium shops, and
offers educational modules on aquarium care and ornamental aquaculture.
Port Blair to be Renamed ‘Sri Vijaya Puram’ in Honor of India’s Historical Legacy
Union Minister Amit Shah announced that Port Blair, the capital of the Andaman and Nicobar Islands, will be renamed
Sri Vijaya Puram.
This renaming reflects Prime Minister Narendra Modi’s vision of replacing colonial-era names with those that honor
India’s rich history and freedom struggle.
The new name acknowledges the region's ancient connections, particularly to the Chola dynasty, which played a
significant role in Southeast Asia.
Typhoon Yagi: Asia’s Strongest Storm of 2024 Causes Over 400 Deaths, Massive Destruction Across Multiple
Countries
In early September 2024, Typhoon Yagi, the strongest storm of the year in Asia, struck Myanmar, Vietnam, Laos, and
Thailand, causing widespread devastation.
The typhoon resulted in over 400 deaths, forced hundreds of thousands from their homes, and led to severe flooding
and landslides.
181
The scheme will distribute funds in two installments: ₹5,000 on Rakhi Purnima and ₹5,000 on International Women’s
Day, deposited directly into Aadhaar-linked bank accounts. Over five years, it is expected to benefit 1 crore women,
with an allocation of ₹55,825 crore. This initiative seeks to attract women voters and compete with the BJD's Mission
Shakti programme.
Chennai Celebrates 146th Birth Anniversary of Social Reformer Periyar E.V. Ramasamy
On September 17, 2024, Chennai commemorated the 146th birth anniversary of Periyar E.V. Ramasamy, a renowned
social reformer from Tamil Nadu, with Chief Minister M.K. Stalin leading the tributes.
Born in 1879 in Erode, Periyar initially joined the Congress party but left due to disagreements over caste issues. He
founded the Self-Respect Movement in 1925 and gained national prominence during the Vaikom Satyagraha in 1924.
4th Global Renewable Energy Investors Meet and Expo Highlights Gujarat's Leadership
The 4th Global Renewable Energy Investors Meet and Expo was successfully held in Gujarat, marking the first
occurrence outside Delhi. Gujarat emerged as the top-performing state, excelling in various renewable energy
capacities, particularly in wind power and ranking second in solar power, with only Rajasthan surpassing it.
The event featured significant initiatives such as the “Mission 100 GW of Renewable Energy in Gujarat” and “Gujarat
Energy Vision 2047,” presented by Chief Minister Bhupendra Patel. Union Minister Pralhad Joshi acknowledged the
state’s contribution of about 53% to India’s solar generation capacity, further reinforcing Gujarat's critical role in the
nation’s renewable energy landscape.
INTERNATIONAL
Cyber Attack Disrupts Elon Musk's Interview with Donald Trump on X
Recently, an interview between Tesla CEO Elon Musk and former US President Donald Trump on the platform X
(formerly Twitter) was interrupted by a cyber attack known as Denial of Service (DoS).
Types of DoS attacks include the Smurf Attack, where attackers flood their own servers by using a spoofed IP address
of the target, and the SYN Flood Attack, where the attacker begins but does not complete multiple connection requests
to the server, blocking legitimate traffic.
UN General Assembly Introduces Multidimensional Vulnerability Index to Aid Small Island States
The UN General Assembly has recently launched the Multidimensional Vulnerability Index (MVI), a new quantitative
tool designed to assist small island developing states (SIDS) in obtaining low-interest financing.
The MVI comprises two main components, Universal Level Quantitative Assessment which provides a summary index
that ranks countries based on their structural vulnerability and resilience, using a unified methodology to produce an
overall MVI score and Vulnerability-Resilience Country Profiles which offer a detailed, tailored analysis of a country's
specific vulnerability and resilience factors.
ESA's JUICE Probe Successfully Executes Double Slingshot Maneuver to Journey Towards Jupiter
The European Space Agency's Jupiter Icy Moons Explorer (JUICE) Probe recently performed a critical double slingshot
manoeuvre, utilising the gravitational forces of both the Moon and Earth.
This intricate navigation sequence involved JUICE passing 434 miles from the Moon and then 4,229 miles from Earth,
leveraging these gravity assists to adjust its trajectory efficiently while conserving propellant.
182
This successful execution has set JUICE on a course to reach Jupiter by 2031, aided by additional gravity assists from
Venus in 2025 and Earth in 2026 and 2029.
JUICE aims to explore Jupiter’s three large icy moons—Callisto, Europa, and Ganymede—with the ultimate goal of
orbiting Ganymede to assess its potential to support life, building on the legacy of NASA’s Galileo mission and other
significant Jupiter missions like Juno and Cassini-Huygens.
Telegram CEO Pavel Durov Arrested in France Amid Privacy Concerns and Legal Scrutiny
Pavel Durov, the CEO and co-founder of Telegram, was arrested at Le Bourget airport in Paris, linked to a French
investigation into the app’s role in enabling illegal activities such as drug trafficking and cyberstalking through its
privacy features.
A Russian expatriate and staunch privacy advocate, Durov created Telegram in 2013 to offer secure communication,
distinguishing the platform with strong encryption and large group chat capabilities.
Pacific Islands Forum 2024 Tackles Climate Change and US-China Rivalry in Tonga
The Pacific Islands Forum (PIF) convened in Nuku'alofa, Tonga, on August 26, 2024, drawing over 1,500 delegates
from approximately 40 countries to discuss pressing issues like climate change and the strategic tussle between China
and the US in the Pacific.
Founded in 1971, the PIF comprises 18 Pacific nations including Australia, New Zealand, and Fiji, focusing on regional
growth, security, and cooperation.
This year's key agenda includes addressing the dire impacts of climate change, with Pacific nations facing existential
threats from rising sea levels and a significant funding gap in the Pacific Resilience Facility.
Berkshire Hathaway Hits $1 Trillion Market Cap, Led by Warren Buffett's Strategic Decisions
Berkshire Hathaway, under the leadership of Warren Buffett, reached a landmark $1 trillion market value on August
28, becoming the first non-tech company to achieve this milestone.
This historic valuation followed Buffett's strategic move to sell nearly half of Berkshire's Apple shares, accumulating
a massive $280 billion cash reserve. Buffett, who began his career as an investment analyst, co-founded Berkshire
with Charlie Munger in 1965, transitioning it from a textile company to a diversified conglomerate.
183
Angkor Wat Named Most Photogenic UNESCO World Heritage Site in Asia
Cambodian Prime Minister Hun Manet announced that Angkor Wat has been recognized as the most photogenic
UNESCO World Heritage site in Asia by Times Travel, as reported by the Times of India.
The list includes other iconic sites such as the Taj Mahal and Hampi in India, the Great Wall of China, the Ancient City
of Bagan in Myanmar, Borobudur in Indonesia, Ha Long Bay in Vietnam, Historic Monuments of Kyoto in Japan, Petra
in Jordan, and the Rice Terraces of the Philippine Cordilleras.
Astronomers Discover Brightest Quasar in the Universe, Providing Insight into Cosmic Beginnings
Astronomers using the Very Large Telescope have identified J0529-4351, the brightest quasar ever observed, offering
unprecedented clarity into the early universe.
Located in a galaxy with a supermassive black hole, this quasar outshines the Sun by over 500 trillion times and
expands at an extraordinary rate, absorbing one solar mass per day.
Situated more than 12 billion light-years away, J0529-4351's light provides a snapshot of cosmic conditions billions
of years ago, with its vast accretion disk spanning seven light-years.
184
Despite achieving a soft landing on the Moon in January 2024, which marked Japan as the fifth country to accomplish
this, subsequent communication failures led to the cessation of its mission. This event follows other successful lunar
landing by nations like India, Russia, the USA, and China.
DRDO and Indian Navy Successfully Test Vertical Launch Short Range Surface-to-Air Missile (VL-SRSAM)
The Defence Research & Development Organisation (DRDO) and the Indian Navy have successfully flight-tested the
Vertical Launch Short Range Surface-to-Air Missile (VL-SRSAM) at the Integrated Test Range in Chandipur, Odisha.
The missile, launched from a land-based vertical launcher, accurately intercepted a high-speed aerial target,
demonstrating its capability to defend against airborne threats.
Indian Immunologicals and ICMR Collaborate to Develop India's First Zika Virus Vaccine
Indian Immunologicals Limited (IIL) has partnered with the Indian Council of Medical Research (ICMR) to develop
India's first Zika virus vaccine. The Memorandum of Agreement (MoA) outlines plans for phase I clinical trials, funded
by ICMR, to test the vaccine’s safety and efficacy at four key locations in India, including ACTREC Mumbai and PGIMER
Chandigarh.
IIL is also collaborating with Griffith University, Australia, and has completed pre-clinical tests. Besides Zika, IIL is
working on vaccines for Kyasanur Forest Disease, Chikungunya, and a SARS-CoV-2 nasal booster. The Zika virus,
primarily spread by mosquitoes, currently has no licensed vaccine, making this development crucial for public health.
Asteroid 2024 PT5: Earth’s Temporary Mini-Moon from September to November 2024
From September 29 to November 25, 2024, Earth will gain a temporary companion—a mini-moon named asteroid
2024 PT5.
This small asteroid, about 10 meters in size, will be captured by Earth’s gravity, briefly orbiting alongside our
permanent Moon.
Discovered by NASA’s ATLAS program, this mini-moon will follow a unique horseshoe-shaped orbit before moving
away.
185
Notably, Raina is only the second non-IPS officer to achieve this rank within the ITBP. Raina's tenure will be brief, as
he is set to retire in over a month, succeeded by IG Jaspal Singh.
TV Somanathan Appointed as New Cabinet Secretary of India Following Rajiv Gauba’s Retirement
TV Somanathan, a seasoned IAS officer from the 1987 Tamil Nadu cadre, has been appointed as India’s new Cabinet
Secretary on August 10, 2024, following the retirement of Rajiv Gauba, who served in the position for five years.
Somanathan, previously the Union Finance Secretary, brings a wealth of experience in finance and administration,
having held prominent roles, including Joint Secretary in the Prime Minister's Office and Director for Corporate Affairs
at the World Bank.
Amrit Mohan Prasad Appointed New Director General of Sashastra Seema Bal
Amrit Mohan Prasad, an Indian Police Service (IPS) officer from the Odisha cadre, has been appointed as the Director
General (DG) of the Sashastra Seema Bal (SSB), as approved by the Appointments Committee of the Cabinet. Prasad
previously served as the Special Director General of the Central Reserve Police Force (CRPF).
Atishi Marlena Appointed as New Chief Minister of Delhi Following Kejriwal's Resignation
Arvind Kejriwal has resigned as Chief Minister of Delhi, paving the way for Atishi Marlena to take over leadership of
the Aam Aadmi Party (AAP).
Born into an academic family, Atishi is an alumnus of St. Stephen’s College and has pursued further studies at Oxford
University on prestigious scholarships.
In 2018, she dropped "Marlena" from her public name to focus on her professional identity as a leader in social work
and education.
186
BOOKS & AUTHORS
Tarun Chugh’s Book "Modi’s Governance Triumph" Released in New Delhi
BJP General Secretary Tarun Chugh's book, Modi’s Governance Triumph: Reshaping India’s Path to Prosperity, was
launched in New Delhi, attended by Home Minister Amit Shah and Coal Minister G. Kishan Reddy.
The book analyzes key initiatives like Make in India and GST, highlighting Modi's policies on poverty alleviation,
economic disparity, and national security. It also delves into the Atmanirbhar Bharat vision, challenges faced by the
administration, and the ambitious outlook for India's future as a global power by 2047.
IAF Chief Launches Comic Book Series Celebrating Air Force Heroes
Air Chief Marshal V R Chaudhari launched the first comic book in a series highlighting the Indian Air Force's rich
history and heroic deeds. Aimed at inspiring youth, the book features stories of Marshal of the Air Force Arjan Singh
DFC and the 1971 Battle of Boyra. It also provides career information for aspiring IAF personnel.
The comic will be freely distributed during induction drives, with a digital version releasing on Air Force Day, October
8. This initiative bridges generational gaps and promotes Air Force careers through engaging storytelling.
IMPORTANT DAYS
September 10, 2024, World Suicide Prevention Day
On September 10, 2024, the world will observe World Suicide Prevention Day, focusing on raising awareness about
suicide prevention and reducing stigma surrounding mental health. Organized by the International Association for
Suicide Prevention (IASP) in collaboration with the World Health Organization (WHO), the event aims to inspire
action, educate the public, and offer hope. The theme for World Suicide Prevention Day 2024-2026 is “Changing the
Narrative on Suicide” with the call to action “Start the Conversation”.
187
The International Association for Suicide Prevention (IASP) first established World Suicide Prevention Day in 2003.
National Suicide Prevention Strategy (NSPS) launched by the Ministry of Health and Family Welfare in 2022, the NSPS
is India's first suicide prevention policy aimed at reducing suicide mortality by 10% by 2030.
188
September 1-7: National Nutrition Week (Rashtriya Poshan Maah)
National Nutrition Week, observed annually from September 1 to September 7 in India, focuses on the significance of
a nutritious diet for physical and mental well-being. First observed in 1982, it promotes awareness about the dangers
of fast food and encourages healthy eating habits.
Originally initiated by the American Dietetic Association in 1975, India adopted this observance to spread knowledge
about nutrition’s role in longevity and vitality. The 2023 theme, “Healthy Diet Gawing Affordable for All,” as per
UNICEF, emphasises making nutritious food accessible to everyone.
189
The 2024 theme, “Montreal Protocol: Advancing Climate Action". The ozone layer, vital for shielding life on Earth from
harmful ultraviolet radiation, has been gradually healing, with projections for full recovery in mid-latitudes by the
2030s and in polar regions by 2060.
OBITUARIES
Rakesh Pal, Director General of the Indian Coast Guard
Rakesh Pal, who served as the 25th Director General of the Indian Coast Guard, passed away at the age of 59 due to a
severe cardiac arrest in Chennai. He was in the city to attend a Coast Guard event led by Defence Minister Rajnath
Singh.
Pal, a flag officer from Uttar Pradesh, joined the Indian Coast Guard in January 1989 after attending the Indian Naval
Academy. Over his 34-year career, he played a crucial role in modernizing the Coast Guard, earning the Vishist Seva
medal for his contributions. He became Director General in July 2023, leading the maritime law enforcement agency
which operates under the Ministry of Defence.
Dr. Ram Narain Agarwal, Renowned Indian Scientist and ‘Father of Agni Missiles’
Dr. Ram Narain Agarwal, a pioneering figure in India’s missile program, passed away at the age of 84 in Hyderabad.
Fondly known as the 'Father of Agni Missiles,' Dr. Agarwal's revolutionary contributions spanned over 22 years in
defence research, where he collaborated with luminaries like Dr. APJ Abdul Kalam.
He played a crucial role in developing missile technologies such as re-entry systems, heat shields, and guidance
systems. His leadership in the Agni missile program, especially Agni V, significantly boosted India's strategic defence
capabilities. Dr. Agarwal also founded the Advanced Systems Laboratory (ASL), leaving behind a legacy of innovation
and excellence in missile technology. He was honoured with several prestigious awards, including the Padma Shri,
Padma Bhushan, and Lifetime Achievement Award.
190
Death of Māori King Marks End of an Era in New Zealand
Kiingi Tuheitia Pootatau Te Wherowhero VII, the Māori King, passed away shortly after celebrating 18 years of
leadership. He was the seventh monarch of the Kiingitanga movement, initiated in 1858 to unify New Zealand's
indigenous Māori tribes against British colonisation.
The movement aimed to halt land sales to non-indigenous people and cease inter-tribal conflicts. Throughout New
Zealand’s history, significant clashes like the Wairau Valley incident and the Waikato War marked the struggle
between Māori and European settlers for land and sovereignty.
RANKING
Global Air Quality Study Reveals Life Expectancy Gains from Pollution Reduction
The 2024 Air Quality Life Index by the University of Chicago highlights a critical global health issue: reducing PM2.5
levels to WHO standards could extend global life expectancy by 14.9 billion years. In India, over 40% of the population
breathes air that surpasses the national standard of 40 µg/m³, with potential gains of 1.9 additional years per person
if WHO guidelines are met.
The report stresses uneven pollution distribution, with severe impacts in regions like South Asia and significant
improvements in countries like the US and China through rigorous policies. Compliance remains a challenge globally,
affecting life expectancy and health.
Rajiv Gandhi International Airport Named Best Airport for Third Consecutive Year
Rajiv Gandhi International Airport (RGIA) in Hyderabad has clinched the title of Best Airport at the India Travel
Awards for the third consecutive year, underscoring its commitment to superior passenger service.
This accolade, complemented by the Skytrax award for ‘Best Airport Staff in India & South Asia 2024,’ highlights RGIA's
consistent quality and its pivotal role in positioning Hyderabad as a key hub in South Asian air travel.
NHPC, SJVN, and RailTel Attain Prestigious Navratna Status, Expanding Investment Autonomy
NHPC, SJVN, and RailTel have been awarded Navratna status by the Government of India, marking a significant
milestone for these public sector enterprises.
This recognition, previously held by 17 companies, allows them greater operational freedom, including investment
autonomy of up to ₹1,000 crore per project. The status, earned through strong financial performance and high MoU
ratings, enables these companies to form joint ventures, set up subsidiaries abroad, and invest further in their sectors.
Telangana, Tamil Nadu, and Rajasthan Emerge as Fastest-Growing Large States in India
The Ministry of Statistics and Programme Implementation (MoSPI) has recognized Telangana, Tamil Nadu, and
Rajasthan as the fastest-growing large states in India, with each surpassing the national average GDP growth of 8.2%.
Telangana led the way with a 9.2% growth, reaching a GSDP of ₹7.9 lakh crore, followed by Tamil Nadu at 8.2% (₹15.7
lakh crore), and Rajasthan at 8%.
India Achieves Tier 1 Status in Global Cybersecurity Index 2024, Ranking Among World's Best
India has achieved Tier 1 status in the Global Cybersecurity Index (GCI) 2024, a recognition of its commitment to
building a robust cybersecurity infrastructure.
The GCI, developed by the International Telecommunication Union (ITU), evaluates countries based on legal, technical,
organisational, capacity development, and cooperative measures to ensure online security. India’s impressive score
of 98.49 reflects its strong legal framework, such as the Information Technology Act (2000) and the Digital Personal
Data Protection Bill (2022), placing it among the top 47 nations globally for cybersecurity practices.
191
Its dominance in India’s dairy market and strong metrics in familiarity and recommendation contributed to its AAA+
rating. Hershey’s, with a $3.9 billion valuation, took second place. Despite a 4% drop in the broader food sector’s brand
value, Amul continues to thrive.
Mumbai, Delhi Rank Among Top 3 Cities Globally for Prime Residential Price Growth
According to Knight Frank's report, Mumbai and Delhi ranked second and third globally for annual price rise in prime
residential properties in the June 2024 quarter. Mumbai saw a 13% increase, up from 6th place a year ago, while New
Delhi jumped from 26th to third with a 10.6% rise.
Bengaluru ranked 15th with a 3.7% increase. Globally, Manila topped the list with a 26% rise, while Los Angeles and
Miami ranked 4th and 5th. Cities like Vienna and Bangkok saw declines, with Wellington ranking last.
Mumbai Named Billionaire Capital of Asia in Hurun India Rich List 2024
The Hurun India Rich List 2024 has named Mumbai as Asia’s new billionaire capital, surpassing Beijing. Mumbai now
hosts 386 billionaires, up by 58, with a total wealth of $445 billion. This positions Mumbai third globally behind New
York (119) and London (97). Delhi ranks second in India with 217 billionaires, while Hyderabad (104) overtakes
Bengaluru (100).
Mumbai’s rise is attributed to its robust financial sector and entrepreneurial ecosystem. Beijing's decline, with 18
fewer billionaires, reflects broader economic challenges in China.
Switzerland Tops "Best Countries for 2024" Survey for Third Consecutive Year
In the “Best Countries for 2024” survey, Switzerland ranked as the best country for the third year in a row, out of 89
countries.
The survey, which assessed categories like adventure, entrepreneurship, and quality of life, saw European countries
dominate the top 25. Switzerland was followed by Japan, the USA, Canada, and Australia.
India ranked 33rd, dropping three spots from last year, performing best in the "movers" and heritage categories, but
scoring low in social purpose and adventure. The UAE and Qatar were the only Middle Eastern countries in the top 25.
192
o YouTube: 60.6 million
SPORTS
Vithya Ramraj Shatters P.T. Usha’s 39-Year-Old Record at the 63rd National Open Athletics Championships
At the National Open Athletics Championships in Bengaluru, Vithya Ramraj broke a nearly four-decade-old Indian
record in the women's 400m hurdles, previously held by the iconic P.T. Usha. Ramraj clocked in at 56.23 seconds,
besting Usha's 1985 record of 56.80 seconds.
This historic achievement underscores a significant moment in Indian athletics, spotlighting Ramraj’s rising
prominence. The event also witnessed standout performances across various disciplines, with Tamil Nadu's Nithin
breaking the men’s 200m meet record. Ancy Sojan earned the best woman athlete title with a remarkable long jump
of 6.71 metres.
Women's T20 World Cup 2024 Relocated from Bangladesh to UAE Due to Safety Concerns
The ICC has announced the relocation of the 2024 Women's T20 World Cup from Bangladesh to the UAE, set to take
place from October 3 to 20 in Dubai and Sharjah.
Political unrest in Bangladesh, following Prime Minister Sheikh Hasina’s resignation and ongoing turmoil, led to safety
concerns and travel advisories from several participating nations.
Jannik Sinner and Aryna Sabalenka Win First Cincinnati Open Titles
Top-ranked Jannik Sinner and women’s No. 2 Aryna Sabalenka claimed their first Cincinnati Open titles with straight-
set victories. Sabalenka defeated Jessica Pegula 6-3, 7-5, securing her 15th WTA title, while Sinner triumphed over
Frances Tiafoe 7-6 (4), 6-2, becoming the youngest Cincinnati champion since Andy Murray in 2008.
Sabalenka’s win reinforces her as a U.S. Open contender despite recent injuries, while Sinner looks forward to the U.S.
Open after his milestone victory. Tiafoe and Pegula faced challenges in their respective finals, with errors and fatigue
contributing to their defeats.
Neeraj Chopra Secures Second Place at Lausanne Diamond League with Season's Best Throw
Neeraj Chopra, India’s star javelin thrower, claimed a second-place finish at the Lausanne Diamond League,
showcasing his resilience.
Despite initial struggles, Chopra surged in the final rounds, delivering an 89.49m throw in the sixth round—his
season’s best, surpassing his Paris Olympics throw. Grenada’s Anderson Peters won gold with a 90.61m throw, while
Germany's Julian Weber secured bronze. Battling a groin injury, Chopra’s performance earned him 15 points in the
Diamond League standings, tying him for third place.
193
Diana Pundole Becomes First Indian Woman to Win National Car Racing Championship
Diana Pundole, a teacher and mother from Pune, made history by becoming the first Indian woman to win the national
championship in the saloon category at the MRF Indian National Car Racing Championship 2024 in Chennai.
Achieving this milestone on her birthday weekend, Diana overcame personal and societal challenges in a male-
dominated sport. Her racing career, fueled by a lifelong passion, includes competing internationally at prestigious
tracks like Dubai Autodrome and Hockenheimring.
Lando Norris Triumphs at Dutch Grand Prix, Secures Second Career Win
Lando Norris delivered an exceptional performance at the Dutch Grand Prix, claiming his second career victory with
a commanding lead over Max Verstappen. Despite losing the lead early, Norris executed a decisive overtake to regain
control, finishing over 20 seconds ahead of Verstappen, who had to settle for second on home turf. Ferrari’s Charles
Leclerc secured third place, marking a strong result for the team. McLaren enjoyed mixed fortunes with Oscar Piastri
finishing fourth, while Carlos Sainz earned fifth for Ferrari.
Siddhartha Aggarwal Becomes Oldest Indian to Swim Solo Across English Channel
On August 29, 49-year-old Siddhartha Aggarwal from Bengaluru became the oldest Indian to swim solo across the
English Channel, completing the 42 km swim in 15 hours and six minutes. His achievement surpasses the previous
record set by Srikaanth Viswanathan at age 46.
Aggarwal’s journey began in 2018 with a relay swim across the Channel, and after 15 months of rigorous training
under coach Satish Kumar, he accomplished this solo feat despite challenging tides and winds. His remarkable
dedication serves as an inspiration for swimmers of all ages.
India Announces Bid for 2030 Youth Olympics at OCA General Assembly
India's Union Minister of Youth Affairs & Sports, Dr. Mansukh Mandaviya, announced India's bid to host the 2030
Youth Olympics at the Olympic Council of Asia's (OCA) 44th General Assembly on September 8, 2024.
Competing against countries like Peru, Mexico, and Thailand, this bid aligns with India’s long-term goal of hosting the
2036 Olympics. Dr. Mandaviya highlighted India's sports initiatives, such as the ‘Khelo India’ scheme and the ‘Target
Olympic Podium Scheme (TOPS)’, which have significantly boosted the country’s sports infrastructure and athlete
development. The ASMITA program, promoting women's participation in sports, was also spotlighted.
Jannik Sinner Wins U.S. Open, Securing His Second Grand Slam Title
194
World No. 1 Jannik Sinner clinched his second Grand Slam title at the U.S. Open, defeating Taylor Fritz in a commanding
final. Despite being cleared of a doping controversy just before the tournament, Sinner overcame doubts and delivered
a dominant performance, showcasing his powerful baseline game.
His victory not only reaffirmed his top ranking but also ended a long Grand Slam drought for American men, with the
last U.S. winner being Andy Roddick in 2003. Fritz, playing in his first Grand Slam final, fell short but remains optimistic
about his future in the sport.
Saleema Imtiaz Becomes First Pakistani Woman Nominated to ICC Development Umpires Panel
Saleema Imtiaz has made history as the first Pakistani woman to be nominated to the ICC International Panel of
Development Umpires, as announced by the Pakistan Cricket Board (PCB). Imtiaz, who began her umpiring career
with the PCB women’s panel in 2008, has officiated in notable events like the 2022 Asia Cup and the 2023 ACC
Emerging Women’s Cup.
With 22 T20Is under her belt, her nomination allows her to officiate in women’s bilateral international matches and
ICC events. Imtiaz’s achievement symbolises a growing influence of women in cricket and the PCB's commitment to
gender development in the sport.
India Clinches Fifth Men’s Asian Champions Trophy Title with Jugraj Singh’s Winning Goal
India’s Men’s Hockey Team secured their fifth Asian Champions Trophy title in 2024 by defeating hosts China 1-0 in a
tense final. Jugraj Singh scored the decisive goal with an assist from Harmanpreet Singh, breaking the deadlock after
China’s strong defensive effort in the first half. India dominated possession but faced challenges from China’s
counterattacks, keeping the scoreboard at 0-0 until Jugraj’s second-half strike secured the win.
195
India and South Africa Sign Agreement to Boost Submarine Rescue Capabilities
On September 4, 2024, the Indian and South African navies formalised an agreement to enhance submarine rescue
support, ensuring South African submarine crew safety during emergencies.
The agreement grants South Africa access to India's Deep Submergence Rescue Vehicle (DSRV), capable of rescuing
submariners from depths of up to 650 metres. This partnership, initiated after a 2023 accident that claimed the lives
of three South African navy members, reflects a commitment to maritime safety.
India and Malaysia Sign MoU on Public Administration and Governance Reforms
The Department of Administrative Reforms and Public Grievances (DARPG) of India and the Public Service
Department of Malaysia signed a Memorandum of Understanding (MoU) on August 20, 2024, for cooperation in public
administration and governance reforms.
This five-year agreement focuses on citizen-centric services, e-governance, transparency, and public grievance
redress mechanisms. The MoU was exchanged during a ceremonial function attended by Indian Prime Minister
Narendra Modi and Malaysian Prime Minister Anwar Bin Ibrahim at Hyderabad House, New Delhi.
Tata Power Solar Systems Ltd Signs Financing Agreement with ICICI Bank
Tata Power Solar Systems Ltd (TPSSL) has partnered with ICICI Bank to offer loans for solar unit financing to
residential and corporate customers. Customers can avail loans up to Rs 90 lakh, with collateral-free options and
repayment tenures of up to 5 years.
Additionally, higher loan amounts with collaterals are available for up to 20-year tenures. This initiative, part of
TPSSL's efforts to expand solar energy adoption, is set to make solar panel purchases more accessible to a broader
customer base.
196
54th GST Council Meeting Proposes Tax Cuts on Cancer Drugs and Health Insurance, Considers Adjustments for
Other Goods
The 54th Goods and Services Tax (GST) Council meeting, chaired by Finance Minister Nirmala Sitharaman in New
Delhi, focused on reducing tax rates for essential items such as cancer drugs, health insurance, and savoury snacks.
Key cancer drugs like Trastuzumab Deruxtecan and Osimertinib may see GST reductions from 12% to 5%, easing
treatment costs for patients.
RBI Penalizes UCO Bank and Cent Bank Home Finance Ltd for Regulatory Non-Compliance
The Reserve Bank of India (RBI) has imposed penalties on two financial institutions for regulatory violations. UCO
Bank was fined Rs 2.68 crore for breaches related to current account openings, deposit interest rates, and fraud
classification.
Cent Bank Home Finance Ltd faces a Rs 2.1 lakh penalty for non-compliance with Know Your Customer (KYC)
regulations.
197
The Reserve Bank of India (RBI) has imposed penalties on HDFC Bank (₹1.91 crore) and Axis Bank (₹1 crore) for
violations related to deposit regulations, KYC norms, and agricultural loans.
HDFC Bank faced penalties for non-compliance in areas like deposit interest rates, recovery agent conduct, and
operational issues. Axis Bank was fined for issuing multiple customer identification codes, collateral violations on
agricultural loans, and subsidiary activities outside permitted regulations.
Union Bank of India Joins Partnership for Carbon Accounting Financials (PCAF)
Union Bank of India has become the first major Indian bank to join the Partnership for Carbon Accounting Financials
(PCAF), reflecting a global focus on managing climate risks. PCAF provides a standardised approach for financial
institutions to assess and disclose greenhouse gas (GHG) emissions tied to their loans and investments, often referred
to as Scope 3 emissions.
This move aligns with the RBI’s draft guidelines on climate risk disclosures, which emphasise governance, strategy,
and risk management for regulated entities.
LIC Selects Infosys for Next-Gen Digital Platform under DIVE Programme
The Life Insurance Corporation of India (LIC) has partnered with Infosys to build a next-generation digital platform
as part of its DIVE (Digital Innovation and Value Enhancement) programme.
This platform will enhance customer services, business lifecycle management, and streamline operations for sales
intermediaries. It marks a strategic shift for LIC from previous IT collaborations with Tata Consultancy Services (TCS)
and Tech Mahindra. The platform will be cloud-native, modular, and designed to integrate emerging technologies.
198
LEGALEDGE TEST SERIES
MOCK COMMON LAW ADMISSION TEST 2024-25
CLAT MOCK
TR ID.
(In Figures)
INSTRUCTIONS TO CANDIDATES
Duration of Test : 2 Hours (120 Minutes) Maximum Marks : 120
1. Separate carbonised Optical Mark Reader (OMR) 10. Use BLACK/BLUE BALL POINT PEN only for
Response Sheet is supplied along with this writing the roll No. and other details on OMR
Questions Booklet and the carbon copy has to be response Sheet.
detached and taken by the candidates. 11. Use BLACK/BLUE BALL POINT PEN for shading
2. In case of any discrepancy in the question booklet the circles. Indicate only the most appropriate
(QB), please request the invigilator for answer by shading from the options provided. The
replacement of a fresh packet of QB with OMR. Do answer circle should be shaded completely
not use the previous OMR response Sheet for a without leaving any space.
fresh booklet so obtained. 12. As the responses cannot be modified/corrected on
3. Candidates will not be given a second blank OMR the OMR Response Sheet, candidates have to take
response Sheet under any circumstance. Hence, necessary precautions before marking the
OMR response Sheet shall be handled carefully. appropriate circle.
4. Answer all questions. No clarification can be 13. The candidate should retain the Admit Card duly
sought on the Questions Paper Signed by the invigilator, as the same has to be
5. Possession of electronic devices in any form is produced at the time of Admission.
strictly prohibited in the examination Hall. 14. Handle the OMR response Sheet with care. Do not
6. The use of any unfair means by any candidate shall fold.
result in the cancellation of his/her examination. 15. Ensure that invigilator puts his/her signature in
7. Impersonation is an offense and the candidate, the space provided on the OMR response Sheet.
apart from disqualification, will be liable to be Candidate should sign in the space provided on the
prosecuted. OMR response Sheet.
8. The test Paper for Five Year integrated Law 16. The candidate should write Question Paper
Programme is for 120 marks containing 120 booklet No., and OMR response Sheet No., and sign
multiple Choice Questions. in the space/column provided in the attendance
9. There will be Negative marking for multiple choice sheet.
objective type questions. 0.25 marks will be 17. Return the Original Page of OMR response Sheet to
deducted for every wrong answer or where the invigilator after the examination.
candidates have marked more than one response. 18. The candidate shall not write anything on the OMR
response Sheet other than the details required and
in the spaces provided for
199
SECTION-A : ENGLISH LANGUAGE
Passage (Q.1-Q.6): Many of us want to be different. We may desire to be more extroverted, spontaneous, or productive.
On a more concrete level, we may need to exercise three days a week, consume less alcohol, or lose 20 pounds. One
scientific fact we know for sure is that when we want to change our personalities, we have both the capacity and the
ability. The quest for personal development and change does not imply malcontent but instead suggests the ubiquitous
human need to improve.
Improvement comes in many forms and may include reaching goals such as enhancing skills, becoming healthier, or
appearing more sociable. However, personal development has psychological costs that hold some people back. When we
contemplate change, we also acknowledge an existing deficit or weakness, a realization that some are ill-prepared to
confront. We also must devote considerable effort to the challenge. In many cases, change minimally means habit revision
and, in some cases, may result in a complete reinvention of our personalities, for better or worse.
The capacity to change starts with self-awareness and the creation of a targeted measurable goal. For lasting change to
occur a few things must happen. First, we must have a compelling reason to change. Desire alone, while critical, is
insufficient to make change happen. Often the need for change is suggested or obvious (such as smoking cessation) but
the person does not assess a pressing need and thus, compliance is temporary or sporadic. Second, we must understand
how to make the change. Not everyone recognizes which optimal strategies support the desired outcome. Third, social
support for change improves the likelihood of change effectiveness and stability. Unless these three factors are intact, we
may quickly revert back to our bad habits, subsequently feeling disappointed and frustrated about our failed attempt.
My dilemma was a result of twenty years of late-night activities. When I started graduate school, I worked or attended
class during the days and became accustomed to staying up late most nights to study. It would not be unusual for me to
be pounding the books until after midnight. I needed a few hours to unwind before sleep; my typical lights-out time would
be 2:00 a.m. Unfortunately, I was unable to break the unhealthy habit even though I finished school 16 years ago. The late
bedtime did not mean I would sleep all day, but it clearly meant that I was not getting enough rest for optimal thinking
and productivity.
1. Identify the statement(s) that is/are correct as per the 3. Which of the following can be inferred about the
information given in the passage. author’s perspective on personal development?
A. Improvement in personal development is solely (a) Personal development is an effortless journey that
dependent on an individual’s desire to change. leads to immediate results.
B. The capacity to change involves self-awareness (b) A lack of self-awareness can significantly hinder
and setting measurable goals. the process of personal change.
C. Social support is essential for maintaining effective (c) The author believes that everyone can change their
and stable change. habits without any external support.
D. The author successfully broke their habit of staying (d) The author feels that personal development is only
up late after finishing graduate school. about physical changes, such as weight loss.
E. Personal development can result in a complete
reinvention of one’s personality. 4. Which of the following is most likely to be the source of
(a) Only A and B are correct. the given passage?
(b) Only B, C, and E are correct. (a) Motivational novel (b) Scientific journal
(c) Only A, C, and D are correct. (c) Newspaper article (d) Sociology textbook
(d) Only B, D, and E are correct 5. What would be the underlying premise for the author's
2. Mr. X joins a well-reputed gymnasium, to help him in statement "We must devote considerable effort to the
his targeted and measurable weight-loss journey. challenge"?
However, even after spending considerable amount of (a) People don't invest appropriate amount of effort to
time, he isn't able to achieve the desired result for tackle a challenge.
himself. Based on the information given in the passage, (b) Effort and its optimum amount is indispensable in
what could be the most likely reason for non- addressing a challenge to achieve the desired
achievement of desired goals by Mr. X? result.
(a) Mr. X may not have a compelling reason to pursue (c) The degree of effort required to handle a challenge
weight loss. varies in accordance with the nature of challenge
(b) Mr. X is likely not receiving adequate social at hand.
support in his weight-loss journey. (d) Challenge can break people, for worse, if
(c) Mr. X lacks the desire to change his lifestyle habits. appropriate amount of effort is not given to
(d) Mr. X does not have any measurable goals set for address the same.
his weight-loss journey.
200
6. Which of the following best describes the relationship (b) Self-awareness serves as a prerequisite for
between self-awareness and personal change, establishing measurable goals.
according to the passage? (c) Self-awareness is irrelevant to the process of
(a) Self-awareness is a byproduct of successful personal development.
change. (d) Self-awareness often leads to an unrealistic
perception of personal deficits.
Passage (Q.7-Q.12): Using robotic surgery for major abdominal operations leads to quicker recovery and reduced time
in the hospital, according to a study by British researchers. Before visions of frightening science fiction start dancing
around your brain, you should know that humans are still at the controls in these procedures. Robotics is simply a more
precise way to minimize invasion, which makes for quicker healing. The researchers reported that robotic surgery, which
involves surgeons guiding minimally invasive instruments remotely, reduces the chance of re-admission to the hospital
by 52 percent. They also wrote that robotics reduced the chance of blood clots by 77 percent.
The researchers said their findings challenged the idea that traditional “open” surgery is the “gold standard” for major
operations. The study participants’ physical activity, which was tracked by a wearable smart sensor, showed stamina and
quality of life also increased. Experts note that less-invasive robotic procedures can mean less pain during recovery, which
means less pain medication. “The decreased pain means a decreased need for narcotics,” said Dr. Bethany Malone, a
private practice colon and rectal surgeon in Fort Worth, Texas, who specializes in robotic surgery. “For procedures like
inguinal hernia repair, I often send patients home on over-the-counter medications like Tylenol or ibuprofen,” Malone
told Healthline. “Given the recent opioid pandemic, this is a huge win for the healthcare system. The improved recovery
times also translate (____) a shorter length (____) stay.”
“After robotic colon surgery, patients often go home the first or second day after surgery. With open surgery, patients
would often stay 5 to 7 days in the hospital after surgery,” she added. “There are even centers that are performing
outpatient robotic colectomies where patients are going home the same day as their colon operation,” “The scarring after
robotic surgery is also minimal given the size of the trocars, so patients are able to wear bathing suits without having to
feel self-conscious or having to be reminded of whatever reason they had to require an operation,” Malone noted.
7. Before visions of frightening science fiction start 10. The passage is an answer to which of the following
dancing around your brain, you should know that question?
humans are still at the controls in these procedures. (a) Is robotics the future?
What concern does the author tries to ease through the (b) Does robotic surgery involve no human oversight
lines? during procedures?
(a) Robotic surgeons will be able to perform better (c) Are the surgeons equipped for robotic surgery?
surgeries as they will be devoid of human frailties. (d) Will the robotic surgery replace traditional surgery
(b) Robotic surgery does not mean an absence of in all ailments?
human surgeon while performing the surgery.
(c) Robotic surgery will dispel the need for surgeons 11. ‘The improved recovery times also translate _____ a
which will result in many surgeons becoming shorter length _____ stay.” Fill in the blanks with correct
redundant. set of the Prepositions.
(d) Robotics is simply a more precise way to minimize (a) in; at (b) to; of
invasion which makes for quicker healing. (c) in; of (d) to; at
8. Which of the following reflects the tone of the passage? 12. What is the primary purpose of Dr. Bethany Malone’s
(a) Ironical. (b) Incendiary. statement about prescribing over-the-counter
(c) Technical. (d) Mordacious. medications after robotic surgery?
(a) To highlight the overall effectiveness of robotic
9. All of the followings are the benefits of robotic surgery, surgery compared to traditional surgery.
except (b) To illustrate the potential reduction in healthcare
(a) Robotic surgery results in quicker recoveries and costs associated with robotic surgery.
fewer readmissions to the hospital. (c) To draw attention to the ongoing opioid crisis and
(b) Robotic surgery recovery leads to less pain and the need for alternative pain management
fewer pain medications. strategies.
(c) Robotic surgery is less invasive and more precise. (d) To emphasize the significance of patient
(d) The patients are embracing this brave new world satisfaction following robotic surgery.
of surgery.
201
Passage (Q.13-Q.18): Too many leaders assume that talking about ethics is something you do when there has been a
scandal, or as part of an organization's compliance program. Everyone gets their annual "ethics flu shot" in the mandatory
review of the compliance policy, and all is well for another year. Nick Eply, professor at the University of Chicago, in Four
Myths about Morality and Business, says, "It's a myth to think 'Everyone is different, and everything is relative.' You
actually have to teach people the relative value of principles relative to choices." Leaders have to infuse everyday activities
with ethical considerations and design policies and norms that keep ethics top of mind. Jonathan Haidt, Professor of
Business Ethics at NYU says, "It's important to talk about the positive examples of ethical behaviour, not just the bad ones.
Focusing on the positive reasons you are in business and reinforcing the good things people do strengthen ethical choices
as 'the norm' of the organization."
Leaders must accept they are held to higher standards than others. They must be extra vigilant about not just their
intentions, but how others might interpret their behaviour. While they cannot control every possible misinterpretation,
leaders who know their people well make careful choices in now they react to stressful situations, confront poor
performance, how political they are in the face of controversy, and how receptive they are to bad news. Above all, even in
what might be considered the smallest "white lie," ethical leaders are careful not to signal that hypocrisy is okay. As an
example, a leader may casually review an employee's presentation and provide feedback like, "I think we need to take
these two slides out - that data is inflammatory and we don't want to derail the ultimate outcome which is to convince the
budget committee to give us the resources we want." While the leader might presume he has acted in the best interest of
the group - going to bat for resources they need- the person building the presentation has just been told, "We can't tell the
entire truth because it could prevent us from getting what we want." Leaders must put themselves in the shoes of those
they lead to see what unintended messages they may be sending.
13. According to the passage, why is it important to discuss (b) The Prime Minster, while addressing a public
positive examples of ethical behaviour? meeting, compliments the departments for their
(a) To make the employees realise the necessity of contributions to the well-being of the public and
ethical behaviour. gently reminding them to keep in my mind nation’s
(b) To encourage people to make ethical choices in the health as the prime goal.
organisation. (c) Hard to prove, easy to join in on, and devastating in
(c) To attract the attention of other companies. impact, ostracization is a favourite tactic of
(d) To discourage people from making unrequired workplace aggressor.
ethical choices. (d) Focusing inward and focusing constructively on
others helps leaders cultivate the primary
14. Which of the following is the meaning of the word elements of emotional intelligence.
'compliance' as mentioned in the given passage?
(a) The act of disobeying the authority. 17. According to the passage, which of the following is the
(b) The act of obeying an order, rule or request. trait of an ethical leader?
(c) Strengthening something. (a) They are careful that hypocrisy does not become a
(d) Boasting one's abilities. trend in the organisation.
(b) They set their standards same as their employees.
15. With which of the following statement is the author (c) They talk about ethics only when it suits the
likely to agree? leaders.
(a) Role of a leader is slightly important in an (d) They keep in mind that data should not be
organisation. inflammatory.
(b) Ethical behaviour is important in an organisation.
(c) A leader needs to be apathetic towards its 18. Which of the following statements reflects a key
employees. concern highlighted in the passage about leadership
(d) Ethical behaviour is expected during times of communication?
scandal. (a) Leaders should communicate strictly factual
information to avoid ambiguity.
16. Which of the following statement is similar to the last (b) Leaders must be aware of how their feedback can
sentence of the given passage? unintentionally convey ethical messages.
(a) Police need to think like criminals as it helps to (c) Leaders should always prioritize their own
have the perspective of the criminal when intentions over the interpretations of others.
analysing possible escape routes, points of entry, (d) Leaders should only provide feedback when it is
and patterns of deception. positive to maintain morale.
202
Passage (Q.19-Q.24): Many people at times stop to consider questions such as: Who am I, what is my life’s purpose, and
what will bring me fulfilment? Are we simply the body that we come into this world with, that ages with time and withers
away after death? Are we the mind that grows astute with intellectual pursuits, only to evaporate at the time of death?
Are we the mind that grows astute with intellectual pursuits, only to evaporate at the time of death? Or is there much
more to our ‘selves’?
Spiritual awakening is the moment you begin asking these questions. It marks the start of your quest to find answers.
One way people reflect on these questions is by sitting in silence in meditation. This opens doors to help us find answers.
There is a source of wisdom within you that cannot be accessed through an internet search. It is awaiting deep within you.
Instead of logging on to the computer, you can log on to your own inner essence, soul, through meditation. The answers
are already within each and every one of us, waiting to be unlocked through the practice of meditation. This will lead you
to a source of wisdom that will enrich your life.
What is unlimited wisdom? It is not the intellectual knowledge that we learn from lectures or books; it is consciousness.
Our soul, being the same essence as God, is consciousness. It is a ‘state of knowing all that is to be known’. When we access
the divine wisdom, we reach a state of all-consciousness, one in which we know the answers to life’s mysteries and our
purpose in life. We see life as more than a string of meaningless events and find a lesson and message in all that occurs.
Those who have tapped into their soul look at life from a refreshing perspective. Rather than be tossed about on the sea
of life, dashed by every wave, they watch their life like a movie with subtitles, in which the words at the bottom of the
screen let them know what is going on at the spiritual level. Spirituality is the process of discovering our true self. We
normally think that perception is possible only through our bodily sense organs. However, when we become spiritually
aware, we recognise that we can perceive through the spirit.
19. According to the passage, what helps in getting the (b) Those who reach soul levels get the enjoy life with a
answers to the questions given in the passage? broader perspective and with multiple choices at
(a) Sitting in the presence of a guru. hand.
(b) Sitting in meditation. (c) Those who reach the inner self look at life differently
(c) To go on a journey to find the answers. and search for their role assigned to them by divinity.
(d) Researching on the internet to seek probable (d) By becoming aware of the surroundings, we find all
answers. the answers that lie in the events all around us.
20. According to the passage, what is unlimited wisdom? 22. What can be a suitable title to the above passage?
i) It is the intellectual knowledge that we learn from (a) Consciousness: an unlimited wisdom.
lectures or books. (b) Meditation and consciousness.
ii) Our consciousness, which is the soul, the essence of (c) Answers to the question.
being the same as God. (d) Life from a fresh perspective.
iii) A state of all-consciousness, one in which we know
the answers to life’s mysteries and our purpose in 23. Which of the following is not the synonym of the word
life. ‘astute’?
(a) Only I (b) Only ii (a) Sagacious. (b) Ingenious.
(c) ii & iii but not I (d) All of the above. (c) Perspicacious. (d) Imbecile.
21. “Those who have tapped into their soul look at life from 24. The passage implies that the source of wisdom people
a refreshing perspective. Rather than be tossed about seek is:
on the sea of life, dashed by every wave, they watch (a) Primarily found through academic studies and
their life like a movie with subtitles, in which the words knowledge acquisition.
at the bottom of the screen let them know what is going (b) Accessible through the practice of meditation and
on at the spiritual level.” What can be inferred from the self-reflection.
given sentences? (c) Best discovered through discussions with
(a) By becoming aware of our consciousness, we spiritual leaders.
become aware and play a more active and (d) Limited to the experiences gained from life
meaningful role in life rather than merely seeing life events.
go by.
203
SECTION-B : GENERAL KNOWLEDGE
204
33. Which of the following is/are the example/s of 34. What is the main purpose of CAR T-cell therapy?
approved CAR T-cell therapies? (a) To remove cancer cells from the body
(a) Tisagenlecleucel, also known as tisa-cel (Kymriah) (b) To alter T cells to attach to specific cancer cell
(b) Axicabtagene ciloleucel, also known as axi-cel antigens
(Yescarta) (c) To replace damaged T cells with healthy ones
(c) Brexucabtagene autoleucel, also known as brexu- (d) To prevent the development of cancer in the body
cel (Tecartus)
(d) All three example of CAR T-cell therapies are 35. Who out of the following is the Present Union Minister
approved. of Health and Family Welfare?
(a) Kiren Rijiju (b) Dr. Jitendra Singh
(c) JP Nadda (d) Bhupendra yadav
Passage (Q.36-Q.40): Bharat NCAP
The Tata Safari and Tata Harrier achieved a top star rating for adult and child occupants showing a stable structure and
robust restraint systems strategy which offered good protection in our tests. Both models showed full protection for child
occupants in the dynamic tests. Both are fitted with ISOFIX anchorages, and a passenger airbag disabling switch, making
them very versatile for transporting child occupants. The Safari and Harrier also meet requirements of UN127 and GTR9
for pedestrian protection as standard. The models also comply with Global NCAP’s ESC requirements, with seatbelt
reminders in all seating positions and offer optional ADAS technologies. Alejandro Furas, Global NCAP Secretary General
said, “Global NCAP congratulates Tata on the five star twin model rating for the Safari and Harrier. This very strong result,
the top adult and child safety score in our testing to date, confirms the manufacturer’s strong commitment to developing
safer vehicles which we warmly welcome and applaud.” David Ward, President of the Towards Zero Foundation said, “It
is very satisfying to see Tata’s continuing commitment to high levels of safety performance. The top scoring result for the
Safari and Harrier demonstrates an encouraging safety trend that we are confident will continue across the Indian
automotive industry as Global NCAP’s testing transitions to the work of Bharat NCAP.”
36. What is the speed at which the frontal crash test is (c) New Crash Avoidance Protocol
conducted under Bharat NCAP protocols as of 2023? (d) National Collision Avoidance Policy
(a) 40km/h (b) 50km/h
(c) 64km/h (d) 80km/h 39. In the Latin NCAP and Global NCAP car-to-car
campaign, what type of safety rating did the Peugeot
37. Who out of the following has launched the Bharat Partner Patagónica receive in 2023?
NCAP, India’s own crash safety rating system? (a) 4 stars (b) 3 stars
(a) Suresh Prabhu (b) Piyush goyal (c) 2 stars (d) 0 stars
(c) Dr. Harshavardhan (d) Nitin Gadkari
40. In which of the following year did Global NCAP initiate
38. In the context of global crash test protocols, what does its "#SaferCarsForAfrica" campaign to assess and
the term "NCAP" stand for? promote vehicle safety in African markets?
(a) New Car Assessment Program (a) 2022 (b) 2013
(b) Non-Collapsible Automobile Platform (c) 2017 (d) 2020
Passage (Q.41-Q.46): FCRA Part
The Fair Credit Reporting Act (FCRA), Public Law No. 91-508, was enacted in [X] to promote accuracy, fairness, and the
privacy of personal information assembled by Credit Reporting Agencies (CRAs).
CRAs assemble reports on individuals for businesses, including credit card companies, banks, employers, landlords, and
others. The FCRA provides important protections for credit reports, consumer investigatory reports, and employment
background checks. The FCRA is a complex statute that has been significantly altered since [X] by Congress and the courts.
The Act’s primary protection requires that CRAs follow “reasonable procedures” to protect the confidentiality, accuracy,
and relevance of credit information. To do so, the FCRA establishes a framework of Fair Information Practices for personal
information that include rights of data quality (right to access and correct), data security, use limitations, requirements
for data destruction, notice, user participation (consent), and accountability.
The Federal Trade Commission (FTC) issues commentaries on the statute, but does not engage in rulemaking for the FCRA.
The FCRA was passed to address a growing credit reporting industry in the United States that compiled “consumer credit
reports” and “investigative consumer reports” on individuals. The FCRA was the first federal law to regulate the use of
personal information by private businesses.
The first major credit reporting agency, Retail Credit Co, was started in [a]. Over the years, Retail Credit purchased
smaller CRAs and expanded its business into selling reports to insurers and employers.
205
41. What has been redacted by [X] in this passage? 44. Where is headquarter of the Federal Trade
(a) 1989 (b) 1977 Commission located?
(c) 1970 (d) 1969 (a) Paris (b) Washington, DC
(c) Geneva (d) New York
42. Consider the following statements about the FCRA:
I. The FCRA is a complex statute that has been 45. The first major credit reporting agency, Retail Credit
significantly altered since 1980 by Congress and Co, was started in which of the following year?
the courts. (a) 1892 (b) 1907
II. The Act’s primary protection requires that CRAs (c) 1899 (d) 1932
follow “reasonable procedures” to protect the
confidentiality, accuracy, and relevance of credit 46. How did the 2020 amendments of the FCRA affect
information. administrative spending by nonprofits?
(a) Only I is correct. (b) Both I & II are incorrect. (a) They allowed nonprofits to spend more on
(c) Only II is correct. (d) Both I & II are correct. administrative activities.
(b) They imposed no changes on administrative
43. The FCRA was passed to address a growing credit spending.
reporting industry in the ______that compiled (c) They set a stricter limit on the amount that can be
“consumer credit reports” and “investigative spent on administrative purposes.
consumer reports” on individuals. (d) They removed any restrictions on administrative
(a) United States (b) France spending.
(c) Russia (d) Ukraine
Passage (Q.47-Q.52): Sutlej - Yamuna Link Canal Dispute
"They must have sought directions from the AAP Govt before the hearing in which they conveyed that they were willing
to construct the SYL canal but the opposition and farmers didn't let them do the same." "The Punjab Congress will also
stage a peaceful protest in front of the residence of the Punjab CM. However, he has already disappointed the Punjabis in
the SC. Punjab CM has already sold his conscience," he added. Shiromani Akali Dal (SAD) chief Sukhbir Singh Badal on
Saturday appealed to Punjabis not to allow any central team wanting to conduct a survey for SYL canal land to enter the
state. Speaking on the issue, Haryana BJP MLA Ram Bilas Sharma said, “BJP has always supported the demand of farmers
of southern Haryana for the release of water. Several districts of Haryana such as Bhiwani, Mahendergarh,
Rewari and others have been devastated due to scarcity of water for irrigation purposes.” The SYL canal issue has been a
bone of contention between Punjab and Haryana for the past several years. The Congress workers also protested about
the issue on Monday, declaring they will not allow a single drop of water from Punjab to flow to any other state and that
they will also not allow the canal to be built.
47. Which of the following principle does Punjab invoke (c) 5 years (d) 10 years
to deny sharing water with Haryana?
(a) Riparian (b) Equitable 51. What constitutional provision authorizes Parliament
(c) Harmonious (d) Sustainable to address conflicts related to inter-State river waters,
leading to the enactment of the Inter-State River
48. Which out of the following treaty between India and Water Disputes Act, 1956?
Pakistan allowed India to use the waters of Ravi, Beas (a) Article 306(b) (b) Article 262(1)
and Sutlej? (c) Article 201 (d) Article 184(1)
(a) Beas Water Treaty (b) Indus Water Treaty
(c) Sutlej Water Treaty (d) Karachi Treaty 52. Consider the following statements about the Satluj-
Yamuna Link (SYL):
49. In which of the following year did the Punjab I. The act annulled all inter-state agreements for
Assembly pass the legislation that led to the sharing the waters of the Ravi and Beas rivers.
termination of water-sharing agreements, particularly II. The Supreme Court held the act
impacted the construction of the Sutlej-Yamuna Link "unconstitutional" in 2022, stating that the state
canal? could not unilaterally abrogate the water-sharing
(a) 1998 (b) 2001 pact.
(c) 2004 (d) 2010 (a) Both I & II are correct.
(b) Only I is correct.
50. What is the maximum time limit set for an Inter-State (c) Both I & II are incorrect.
River Water Disputes Tribunal to submit its report (d) Only II is correct.
after the reference is made?
(a) 1 year (b) 3 years
206
SECTION-C : LEGAL REASONING
Passage (Q.53-Q.57): The Supreme Court dismissed the appeal filed by Loop Telecom and Trading Limited assailing the
decision of TDSAT of dismissing their pleas seeking refund of Entry Fee of Rs 1454.94 crores paid for grant of Unified
Access Service Licenses.
The bench of Justices DY Chandrachud, Surya Kant and Vikram Nath observed that Loop Telecom was in pari delicto with
the Department of Telecom and the then officials of the Union government and that they were beneficiary of the ―First
Come First Serve policy which was intended to favour a group of private bidding entities at the cost of the public
exchequer.
On 2 February 2012, the Supreme Court Court by its judgment in Centre for Public Interest Litigation v. Union of
India declared that the policy of the Union government for allocation of 2G spectrum on a ―First Come First Serve basis
was illegal. The judgement resulted in quashing of the UASL's granted by the Union.
Referring to Section 65 of Indian Contract Act, 1872 court observed that in adjudicating a claim of restitution, the court
must determine the illegality which caused the contract to become void and the role the party claiming restitution has
played in it. If the party claiming restitution was equally or more responsible for the illegality (in comparison to the
defendant), there shall be no cause for restitution. (Para 52)
Indian Contract Act, 1872 - Section 56 - Doctrine of Frustration discussed - The applicability of Section 56 of the Indian
Contract Act is not limited to cases of physical impossibility. The doctrine of frustration states that frustration occurs
when an unforeseen event renders performance of a contract impossible or radically different from that originally
contemplated by the parties. No party is considered at fault.
If a contract is found to be frustrated, it is automatically terminated. All future obligations of the parties to the contract
are discharged.
53. The passage in Para 2 gives reference to an observation 55. Continuing with the same scenario as above, what if
made by the court in the present case. It was observed Shyamlal used to bribe the municipality for not
that Loop Telecom was in pari delicto with the examining his items on specific occasions under the
Department of Telecom. What is meant by the guise of tax? Can Shyamlal later allege that he was
expression “in pari delicto”? wrongfully charged an additional tax every time he
(a) A contract by which one party promises the other enters the province?
party to save from loss which may be caused either (a) No, he cannot claim so as he was in pari dalicto
by the conduct of the promissor or by the conduct with the municipality.
of any other person. (b) Yes, he can because he was charged an additional
(b) Both the parties being equally or more responsible tax for entering the province.
for making the contract frustrated. (c) No, he cannot claim that he was charged extra
(c) Loop telecom was equally responsible for the because he bribed the authorities under the
illegality. pretence of tax.
(d) Both parties were equally responsible for the (d) No, he cannot be charged extra because it is against
illegalities. the law.
54. Shyamlal used to deliver freshly baked bread 56. CodeALFA had agreed to construct tunnels for the rail
throughout the province of Hogwarts. He used to pay authority on the premise that works be done 24 hours
octroi, a tax levied by the municipality to enter their a day, seven days a week in order to complete the work
territory, at the entry. Shyamlal used to pay the entry on schedule. Residents' noise complaints resulted in an
tax every time he entered the province for which injunction from court that limited labour hours. The
receipt is given, until he discovered that municipalities rail authority refused to pay the increased fees,
were illegally charging shyamlal. Octroi is collected claiming that CodeALFA had not followed the terms of
only when a person enters the province with goods. the contract. Decide
Shyamlal files a claim for monetary reparation. Decide (a) The contract stands frustrated as the contract
(a) Shyamlal will succeed as he was illegally charged became impossible to perform.
extra tax. (b) The contract do not stands frustrated as it is not
(b) Shyamlal will succeed as in the present case the limited to the cases of physical impossibility.
party claiming restitution was not responsible for (c) The contract stands frustrated because of a
the illegality. foreseen event.
(c) Shyamlal will not succeed as he was in pari dalicto (d) The contract does not stand frustrated merely
with the municipality. because one party is not able to perform the
(d) Shyamlal will not succeed as he has no contract contract.
with the municipality.
207
57. Simran used trivago app to purchase an "All in one (a) The contract has been frustrated because the
travel and seating package" in order to attend a BTS common purpose of both parties has been
concert organized by trivago. This allows her to a one- destroyed.
day return train ticket from Bhopal to Delhi, as well as (b) The contract has been frustrated on grounds of
a front-row seat to the concert. Unfortunately, the common mistake.
event was cancelled due to the rise of a warlike (c) Warlike situation terminates the contract and no
situation between India and China. Simran claims that party is considered at fault.
her contract with trivago was frustrated. Which of the (d) The contract is not frustrated as Simran can still
following assertions is more likely to be adopted by travel to Delhi.
the court in determining the legal position
between Simran and Trivago?
Passage (Q.58-Q.62): While explaining that to prove an offence under Section 306, the Prosecution must satisfy the
ingredients of Section 107 first, Justice Sandeep N Bhatt of the Gujarat High Court refused to interfere with the impugned
judgement and quash the order of acquittal. While doing so, the Bench delved into the terms 'abetment' and 'instigation'
under Sections 306 and 107 of the IPC, at length.
Section 306 of the Indian Penal Code defines 'Abetment of suicide' as an offence punishable under this code. If any person
commits suicide, whoever abets the commission of such suicide, shall be punished with imprisonment of either
description for a term which may extend to ten years, and shall also be liable to fine.
S. 107 Abetment of a thing.—A person abets the doing of a thing, who—
(First) — Instigates any person to do that thing; or
(Secondly) —Engages with one or more other person or persons in any conspiracy for the doing of that thing, if an act or
illegal omission takes place in pursuance of that conspiracy, and in order to the doing of that thing; or
(Thirdly) — Intentionally aids, by any act or illegal omission, the doing of that thing. Explanation 1.—A person who, by
wilful misrepresentation, or by wilful concealment of a material fact which he is bound to disclose, voluntarily causes or
procures, or attempts to cause or procure, a thing to be done, is said to instigate the doing of that thing.
It cannot be said that the abusive language, which had been used by the appellant on 25.07.1998 drove the deceased to
commit suicide. Additionally, the Court referred to Ramesh Babulal Doshi vs. State of Gujarat (1996) 9 SCC 225) to affirm
that the Appellate Court cannot substitute its own view by reversing the acquittal into conviction and this is a cardinal
principal of criminal jurisprudence. (1996) 9 SCC 225) Ramesh Babulal Doshi V. State of Gujarat.
"When the trial Court renders its decision by acquitting the accused, presumption of innocence gathers strength before
the Appellate Court. As a consequence, the onus on the prosecution becomes more burdensome as there is a double
presumption of innocence."
Accordingly, the Bench dismissed the Appeal.
(SOURCE: Prosecution Must Satisfy Section 107 Requirements To Prove An Offence Under Section 306 Of IPC: Gujarat
High Court)
58. Chavi was well aware that Radha was madly in love 59. Sanju is a well-known Bollywood actor from the 1990s.
with him and will do everything he ask. They took a He met Jim at a private party, and there he introduced
short trip to Lonavala one time. On the way, Chavi Sanju to the world of drugs. Sanju was also going
asked Radha what crazy things she could do in love, and through a rough patch in his life, so he turned to drugs
Radha replied, "If you ask me to jump down a hill, I'll do for solace. Sanju was sent to rehab after his father
that for you." When Chavi reached the top, he became concerned that his son had become an addict.
inadvertently asked Radha to jump in a funny way. Doctors warned Sanju that if he didn't quit, he would
Radha took a leap. Is Chavi liable under Section 306 of die. Jim secretly went to the rehab Centre and gave
the IPC? Sanju the drug in the form of a brownie. Sanju died as a
(a) There was no intention on part of Chavi, hence, not result of a drug overdose. What crime will Jim be
liable. charged with?
(b) First condition of S. 107 attracts Chavi’s liability (a) Jim will be charged under Section 306 read in
under section 306. conjunction with Section 107 of the IPC.
(c) Chavi abetted Radha to jump down the hill, hence (b) Jim will be charged with instigating, aiding and
liable. abetting under Section 107 of the IPC.
(d) Chavi will be held liable if prosecution satisfies the (c) Jim will not face any charges because all he did was
ingredients of Section 107 first. start giving Sanju some drugs.
(d) Jim will not be held liable because he was unaware
of Sanju's critical condition.
208
60. Government of India launched a helpline number for Appellate Court. As a consequence, the onus on the
those who suffered from severe anxiety and depression prosecution becomes more burdensome as there is a
the term of COVID. Rahul feeling anxious and double presumption of innocence." Which other
depressed one day as he was living alone for a period presumption of innocence is referred to by the court?
of 8 month and had no one to take care called the (a) There shall be no such presumption of innocence
helpline. He asked for help as he was having suicide once a person is convicted by a trial court.
thoughts and needed to talk to someone. Talking to (b) An accused is innocent until proven guilty.
helpline assistant did not helped and even escalated (c) An accused is guilty until proven innocent.
the situation to worse by stating that you are of no use, (d) Presumption of innocence.
you should die. Rahul tried committing suicide by
drinking a bottle of whitener placed on his study table 62. Assertion: To prove an offence under Section 306, the
and died. Decide who will be held liable? ingredients of Section 306 and Section 107 must be
(a) Helpline assistant will be held liable for instigating considered because they are not mutually exclusive
rahul to take such decision. events.
(b) Rahul will be held liable if the attempt was not Reason: Prosecution Must Satisfy Section 107
successful. Requirements To Prove An Offence Under Section 306
(c) Can’t be determined. Of IPC.
(d) Government will held vicariously liable for the acts (a) Both A and R are true but R is not correct
of his servants in instigating rahul to attempt explanation of A.
suicide. (b) Both A and R are true and R is correct explanation
of A.
61. Author mentions in its last Para that “When the trial (c) A is true but R is false.
Court renders its decision by acquitting the accused, (d) A is false but R is true.
presumption of innocence gathers strength before the
Passage (Q.63.-Q.67): Many a times, especially in defamation cases the identity of an anonymous person posting a
comment as a blog, become a crucial question. In Independent Newspaper Inc. v. Brodie, the court of Maryland held that
the court can direct the disclosure of details about the anonymous person subject to the following conditions: i).
Reasonable notice has been given to the anonymous person that his identity was being requested by the plaintiff. ii). Such
person has been given sufficient time to oppose disclosure of details. iii). Alleged defamatory statement has been specified
in the notice. iv). There is a prima facie case; and v). It has been proved that balance of convenience lies in favour of
disclosure of his identity. In India, the Code of Criminal Procedure, 1973 empowers the police to call for records from
telephone companies and Internet service Providers (ISPs) to identify the person who has made a transaction; and obtain
his log information content contained in an e-mail/ chat account and IP address details. In civil cases, courts can issue
notices to such ISPs directing them to submit the required information. Normally, Internet Service Providers will not
entertain direct request for disclosure of identity of a person, and even if the complaint is made against a fake blog or
impersonating profile on website, an Internet Service Provider in whose favour balance of convenience lies will at most
block the subject page from access but do not reveal the person who created it.
(Source:https://www.bhu.ac.in/law/blj/Banaras%20Law%20Journal%20%202015%20Vol%2044%20No.%202.pdf)
63. Assertion: Courts have wrestled with producing an his “Virat Roop” from the “early 2000s” to “tame”
appropriate standard for revealing the identity of an Banerjee in Bengal, sparking anger among some
anonymous blogger who posts allegedly defamatory Twitter users, who asked the US-based platform to take
material on a message board or website. action. The referenced account has been suspended for
Reason: The standard created is far too sympathetic to repeated violations of Twitter Rules specifically our
anonymous bloggers and fails to address important Hateful Conduct policy and Abusive Behavior policy.
issues facing victims of defamation. Will the anonymous individual have any redress
(a) Both A and R are true but R is not correct against the internet service provider's arbitrary
explanation of A. action?
(b) Both A and R are true and R is correct explanation (a) No, since the internet service provider blocked the
of A. user's account after receiving a complaint.
(c) A is true but R is false. (b) Yes, because no notice was provided to the
(d) A is false but R is true. anonymous individual and sufficient time for them
to object to the disclosure of details.
64. An anonymous individual had tweeted on alleged (c) No, since there is a prima facie case of slander.
violence in the state following Banerjee-led All India (d) Yes, because the balance of convenience favors
Trinamool Congress’ assembly election victory. In the disclosing his identify.
tweet, the person urged Prime Minister Modi to show
209
65. A criminal complaint has been filed against an (b) The action taken against Meera is
anonymous commenter who has commented on every not inappropriate because the internet service
social media post by JJP leader Shri Modi. The provider has the right to directly block the user in
complainant considers the accused's statements such such cases.
as "chowkidaar chor hai," "choron ka sardar," and (c) The action taken against Meera is unjust since she
"Commander-in-Thief" to be defamatory to the whole was not served with the notice.
political party and has thus attempted to prosecute the (d) The action taken against Meera is inappropriate
anonymous commenter under Section 500 of the as an internet service provider cannot unilaterally
Indian Penal Code. Decide block a user's account.
(a) The court will order for disclosure of information
about the anonymous individual. 67. Chetna, is the winner of an MTV entertainment show,
(b) The Internet service provider must provide the India's next top model. Her reputation skyrocketed
necessary information on the anonymous user after she was seen in the city whilst handing food packs
who submitted defamatory comments. to the needy. Fans admired the actress's compassion
(c) The Internet service provider must give enough and gratitude towards other humans. Dheeraj, the
notice and opportunity to anonymous camera guy for Etv, a programme that provides
person defends his case before blocking his frequent updates on the entertainment industry, used
account. to write her current location in the comment section of
(d) The police may request the disclosure of her Instagram account from his fake account, so that
information about the anonymous individual. her followers could reach her. This became a major
issue for the actress after a while, since she is now
66. Meera writes a regular blog, most of which are political being followed by creeps and stalkers on a regular
in nature. The topic revolves around the lack of basis. The actress filed a criminal complaint against the
government responses to queer people, who are fake account. In the current situation what will be the
constantly denied basic rights. Who always have to next resource of law enforcement agencies?
battle for their basic rights as a form of equality. The (a) Dheeraj's bogus account might be directly blocked
gay community accused Meera of defamation, claiming by the Internet service provider.
that the material is targeted at laymen and that (b) The court will issue an order requiring the Internet
amongst them they are a disadvantaged community service provider to reveal the identification of the
that would remain such as neither the government nor accused.
the law-making agencies are interested in elevating the (c) The police will request that the Internet service
position of queer people in society. Meera's blog was provider supply the essential information on the
blocked and deleted without even notifying her. Decide offender so that the investigation may proceed.
(a) The action taken against Meera was unjust since (d) The police can immediately request information on
she was not given a fair chance to defend herself. the accused from the Internet service provider.
Passage (Q.68–Q.73): Coercion is committing, or threatening to commit, any act forbidden by the Indian Penal Code
(Section 45 of 1860), by the unlawful detaining, or threatening to detain, any property, to the prejudice of any person
whatever, with the intention of causing any person to enter into an agreement. The words in this segment are significantly
more extensive which existed amid the English authorities. This definition expresses that the coercion which refutes a
contract ought to not really go from a gathering to an agreement, or ought to be quick against a man whom it was proposed
to cause to go into a contract or anything which influences his property.
There are certain words in this section that needs to be defined:
Act forbidden by IPC- The word act forbidden by Indian Penal Code make it necessary for the court to decide in a civil
action, whether the alleged act of coercion is such as to amount to an offence. It is clear that coercion is committing
or threatening to commit any act which is contrary to law.
Unlawful Detaining of Property: A consent can be said to be caused by coercion, if it is caused because of unlawful
confining of a property, or a risk to do as such.
Prejudice- Mere sentimental prejudice is no ground for coercion rather there should be some legal injury in order to
be prejudiced.
Causing any person to enter into an agreement- In the case of Vibha Mehta v Hotel Marina at the point when coercion
is charged, the claim like extortion or distortion must be upheld by particulars. It is simply after entire particulars of
the affirmed coercion are given that the court can ask into it and choose whether it stands demonstrated or not.
210
68. Vidya wanted to purchase a vehicle from her friend (c) No, since the jewellery was given out of Gayatri’s
Sugandha for some time now. While the prices were own accord, there is no coercion and the claim will
being negotiated, Sugandha said that this is the best fail in the court of law.
price Vidya can get this vehicle for. On being inquired (d) Yes, theft is an offence punishable under IPC and
whether this vehicle is in a perfect condition Sugandha, therefore Gayatri’s claim will survive in the court
without any doubt said that this vehicle is as good as of law.
new. Also knowing that Vidya believed a lot on
astrology, Sugandha took benefit of this fact and stated 70. After a few days, Pooja came back and told Gayatri that
that this vehicle brings great prosperity in the house of she would not return her jewellery unless Gayatri
those who take care of it and brings turmoil to those agrees to give one of her jewellery to Pooja as a gift
who say anything bad about it. After heavy insistence (although she gave Gayatri the choice to select which
by Sugandha and on representations made by her, jewellery she wants to gift Pooja). Gayatri gives Pooja
Vidya agreed to purchase the vehicle for the price the cheapest jewellery and Pooja also returns the other
quoted by Sugandha. Next day, she met with an jewels to Gayatri. Gayatri later brings a claim for
accident while driving the vehicle and also found out coercion against Pooja to fetch the one jewellery that
that this vehicle had some latent issues which were not she gifted Pooja. Can she re-claim the gifted jewellery
disclosed by Sugandha at that time. Vidya brings a on the premise of coercion?
claim against Sugandha for coercion. Will her claim (a) No, the jewellery were given as a gift and therefore
succeed? there is no coercion committed by Pooja.
(a) Yes, there was heavy insistence by Sugandha on (b) No, despite the jewellery being given by initial
Vidya to purchase the vehicle and therefore Vidya’s threats, however, the choice as to which jewellery
claim will succeed. to be gifted vested with Gayatri.
(b) Yes, Sugandha took unfair advantage of the belief (c) No, there was no actual threat to life and limb of
of Vidya into astrology and misled her to purchase Gayatri or her acquaintances and Pooja cannot
the vehicle. reclaim the gifted jewellery.
(c) No, Vidya herself failed to conduct the requisite (d) Yes, there was detaining of jewellery by Pooja to
due diligence before purchasing the vehicle. compel Gayatri to gift her the jewellery and
(d) No, there was no actual committing or threatening therefore there is coercion.
to commit any act forbidden by law and thus no 71. Which of the following statements are correct?
coercion is involved in the whole transaction. I. Intention plays a vital role in determining whether
69. In Gayatri’s locality, the news of theft had been coercion has been committed or not.
becoming rampant. Her neighbour, Pooja came to her II. Coercion affects the validity of the contract as it
house one day and told her about the new locker she affects free consent.
bought which no thief can ever break and if Gayatri III. Act punishable by IPC should be based on a civil
wishes, she can also keep her jewellery in her locker wrong.
and can take it whenever she wants. Based on these (a) All I, II and III are correct.
representations and to secure her jewels, she gave (b) Only I and II are correct.
them to Pooja for safe-keeping. After a week, it came to (c) Only II and III are correct.
Gayatri’s knowledge that Pooja had fled with her (d) Only I and III are correct.
jewellery and now she wants to bring a claim against 72. Ritesh, a 10-year-old wanted to purchase a new bicycle
her family and to secure her jewels, she gave them to as all his peers were in possession of the new bicycles.
Pooja for safe-keeping. After a week, it came to One day, he threatened his parents to buy him a bicycle
Gayatri’s knowledge that Pooja had fled with her otherwise he will commit suicide. Frightened by the
jewellery and now she wants to bring a claim against threats of Ritesh his family agreed to buy the same.
her family for the coercion committed by Pooja (as However, they later refused once they caught hold of
theft is an offence punishable under IPC). Is her claim Ritesh. Has Ritesh exercised coercion?
likely to succeed in a court of law? (a) No, threat to commit suicide is not an offence
(a) No, the element of threat or actual committing of under IPC and therefore no coercion is committed
an offence or detaining in order to compel to enter by Ritesh.
into a contract is missing in this case and hence, no (b) Yes, there was threat to commit suicide and
coercion has been committed. therefore Ritesh is liable for coercion.
(b) Yes, there is detaining of property which led to (c) No, Ritesh is a minor and no minor can be made
wrongful loss to Gayatri and therefore coercion has liable for an offence under IPC.
been committed against Gayatri. (d) Yes, Ritesh used illegal means to enter into
contract and therefore committed coercion.
211
73. During the proceedings, it was highlighted that Ritesh (b) No, since parents firmly believed that there is a
was known to make false threats in order to get his likelihood of the commission of the offence
work done which everybody was aware including his therefore coercion can be invoked.
parents. However, the parents didn't want to negate (c) Yes, as crying wolf amounts to bad precedents for
the possibility of actual commission. Will this fact Ritesh and this would influence his veracity in the
influence the contract? court of law.
(a) Yes, since the parents were already aware that (d) No, since threatening to commit suicide is an
Ritesh only makes false claims there was no actual offence punishable under IPC, the act of compelling
threat and therefore no coercion was committed. to buy a bicycle will amount to coercion.
Passage (Q.74-Q.78): A contract is said to be discharged by performance when both the parties perform all the primary
obligations, both express and implied, which are set out under the contract. The obligation is considered performed only
if the performance complies with the standard of performance required. A failure to do so constitutes a breach. Various
High Courts while interpreting Section 37 held that the proviso to it maintains that the promise binds the representatives.
In the case of BasantiBai v. Prafulla Kumar Routra, Cuttack High Court held that if the contract is legal, and enforceable,
then even if one of the parties to the contract dies leaving no legal heir, the persons, who acquire interest over the subject
matter of the contract through that deceased party would be bound by the contract and specific performance can be
enforced against such persons. Additionally, when no time for performance is specified in the contract, it must be done
within a reasonable time. Unreasonably long delay cannot be regarded as reasonable. In a contract wherein the date of
performance is fixed, it must be performed during the usual business hours of the day and at the place where such a
performance ought to be performed. When a promise has to be performed within a certain time, it must be performed on
any day before the lapse of that time. If the promisor brings the goods after the business hours and they are not received,
it cannot be said that the promise has been performed. When a promise is to be performed and no place is fixed, it is the
duty of the promisor to apply to the promisee to appoint a reasonable place for the performance of the promise, and to
perform it at such a place. Therefore, the time and manner of performance of any promise can be sanctioned by the
promisee.
74. W was a wool clothing retailer. He took the decision to items after business hours and they are not
buy ready-made clothing from V, the manufacturer. A accepted.
twenty lakh rupee supply of woolen clothing was (b) When there is no deadline for completion in the
required under the deal. There was no mention of the contract, it must be completed within a reasonable
time or date. It was expected that it would be delivered amount of time. An excessively long delay cannot
in a timely manner. Although October had arrived, V be rationalized.
had not yet finished the shipment. W asked him to (c) When both parties execute all of the primary
provide at least half of the agreed-upon shipment, but obligations outlined in the contract, both express
in vain. In March, V supplied the finished goods. He was and implicit, the agreement is said to be discharged
sued by W for contract violation. Identify the veracity by performance.
of his assertion? (d) If the contract is valid and enforceable, then even if
(a) Since V delayed the delivery and the deadline one of the parties passes away without leaving a
window had already expired, he will be held liable legal heir, the people who gain through the
for contract violation. deceased party would be liable under the terms of
(b) V shall not be guilty for any violation since the the agreement, and specific performance could be
deadline for the shipment of the merchandise was obtained against them.
uncertain.
(c) Given that the delivery of the items was delayed by 76. For the supply of milk powder, D and F entered into an
an excessively extended period of time, V will be agreement. Within the last day of October, F was
accountable for breach of contract. scheduled to deliver the consignment. Five days prior
(d) V will not be made liable for the delay because, to the deadline, D reminded F, and F gave him the
despite being a little late, he had delivered the performance guarantee at that time. On November 1st
entire shipment to W. at 7am, the shipment arrived. D was annoyed since the
shipment was not delivered on time even though he
75. Which among the following statements, as applied to had reminded F several times to get it. D, as a result
the passage, is the most inappropriate? sued F in court. What advice would you give D if you
(a) Any day before the deadline must be preferred to were his legal counsel?
accomplish promises that need to be kept within a (a) F was only late by a few hours and shouldn't be
specific amount of time. The promise can be punished for such minor neglect. Therefore, D
deemed accomplished if the promisor delivers the shouldn't complain against him.
212
(b) As no time for performance is mentioned in the (c) Due to the fulfilment of the requirements under
contract, it must be completed within a reasonable section 37 of the Indian Contract Act, OP jewellers
amount of time, so D should hold F accountable for may urge B to carry out her end of the
breaching the agreement. arrangement.
(c) D cannot accuse F for any infringement since he did (d) The contract becomes voidable in the instance if
not have any deliberate intent to delay the delivery either party passes away; therefore OP jewellers
of the product. have no standing to demand that B perform.
(d) Owing to the shipment's delay, D has sufficient
grounds to consider F violated the terms of the 78. T and Y reached an agreement to sign a contract
contract, thus D should lodge a suit for breach of stipulating that T would provide Y with 2 kg of
contract. premium cucumbers every day of the week in exchange
for 280 rupees each week. The contract was diligently
77. B's legal guardian was G. He had an agreement with OP carried out and adhered to by both parties for a few
Jewellers to have two sets of gold bracelets and three weeks. After that, T began charging the same price for
sets of diamond necklaces customized specifically for cucumbers that weighed less than 2 kg. T was accused
B. Unexpectedly and tragically, G experienced a heart of fraud by Y because he felt that he was being duped
attack a few days following the deal. He was admitted by T. Tell us what you think about this specific instance.
to the top medical facility in the area, where he spent (a) T will be considered accountable for his failure to
his final days before dying. In order to complete the adhere to the prerequisites of a contract, as
deal, OP Jewellers asked his daughter B, for whom the specified in the passage.
jewellery was made exclusively. She refused to carry (b) Since it was Y's responsibility to ensure that the
out her end of the bargain. What say does OP Jewellers contract was being completed in accordance with
have in this matter? its terms, T shall not be held accountable for his
(a) OP jewellers have no say in the matter given that G acts.
finalized the deal, and as a result, B is under no (c) Since the facts in the question are irrelevant to the
obligation. details given in the passage, it is difficult to
(b) Given that G had designated B as his legal heir, OP ascertain whether T should be held accountable.
jewellers might require B to uphold her end of the (d) T will be held accountable for his actions since
deal. charged Y the false price with fraudulent intent.
Passage (Q.79-Q.84): The Supreme Court on Thursday enquired the whereabouts of former Mumbai Police
Commissioner Param Bir Singh, who was recently declared as an absconder by a Mumbai Magistrate after his non-
appearance in an extortion case. A bench comprising Justices Sanjay Kishan Kaul and MM Sundresh was considering the
petition filed by Param Bir Singh seeking protection in the cases of corruption and extortion filed against him. "Where are
you? Are you in the country? Outside the country", Justice Kaul asked Senior Advocate Puneet Bail who appeared for Singh,
as soon as the matter was taken. The senior counsel replied that the queries can be answered only by the filing counsel.
The Advocate-on-Record told the bench that the petitions have been filed by the power-of-attorney holder of Singh. "You
have not joined any investigation. If you are sitting abroad and approaching the Court...Our suspicion might be wrong...if
the Court gives a favourable order only then he'll come back. It might be so", Justice Kaul observed. The bench said that
no protection can be granted until his whereabouts are known. "No protection, no hearing till we have the answer to the
question - where are you?" Justice Kaul said. The counsel then sought for time till Monday to get instructions. The petition
was filed against the September 16 judgment of the Bombay High Court which dismissed as not maintainable the petitions
filed by him challenging the two enquiries orders issued by the State Home Ministry for allegedly violative service rules
and the second over allegations of corruption.
79. P was a police commissioner of the state. He was (d) The court was justified for deciding the same but
accused of case regarding extortion. However he was the police commissioner was not absconding.
absconding and no one knew his whereabouts.
However the court denied granting protection to P 80. D was a police commissioner who was stuck in a case
unless his whereabouts were known to the court. In the regarding the extortion. However he was running away
light of the above passage was a correct option. from the proceeding. However the court granted the
(a) The court was right in its position as according to protection to the police commissioner without
the passage it is evident that a person having no knowing his whereabouts. In the light of the above
clear whereabouts known to Court cannot be passage choose the correct option.
granted protection. (a) The court was justified for granting him the
(b) The court was justified in giving protection to the protection.
police commissioner. (b) The Court was not justified for granting him the
(c) The court was not justified for deciding the same. protection.
213
(c) As his whereabouts were not known it was court. In the light of the above passage choose the
important for the protection to be granted to him. correct option.
(d) According to the passage the court should not have (a) As he was a police commissioner he should have
granted protection to him. been granted the protection.
(b) As no whereabouts of him were known the Court
81. S was a police commissioner of the state who was was justified in denying him protection.
alleged for extortion in a case. His council pleaded for (c) The court was wrong in its decision.
protection for the police commissioner. However the (d) The court was absolutely right in its decision.
court observed that the court did not know his
whereabouts and the court denied him giving 83. In the light of the passage choose the correct option.
protection at any cost unless his whereabouts are (a) Whereabouts of a person is one of the important
known. In the light of the above passage choose the factors taken into account before being granted
correct option. protection.
(a) The court was right in its approach. (b) Whereabouts of the person is not an important
(b) As it was necessary for the court to know his factor for giving protection.
whereabouts the court was justified for denying (c) The court has no power to deny protection to a
the same. police commissioner.
(c) Ashe was not absconding so the court was not (d) Every person has a right to protection and the
justified. court was not justified in the passage.
(d) As he was a police commissioner he should be
granted protection and the court was not justified. 84. In the light of the passage choose the misinterpreted
option.
82. P was a police commissioner who was alleged to be (a) Whereabouts of a person is not an important factor
involved in extortion and the proceeding was to be held for being granted protection.
against him. However any whereabouts of him were (b) Whereabouts of a person is required by the court
not known. As he was absconding, the counsel pleaded to grant him protection.
for the police commissioner infront of the court that he (c) Court has the right to deny granting protection to
should be granted protection so that he can take part in the accused person.
the proceeding. However the court did not grant the (d) Whereabouts of a police commissioner is also
protection until his whereabouts are known to the important for granting him the protection by the
court.
214
SECTION-D : LOGICAL REASONING
Passage (Q.85- Q.89): The President of India has sent condolences to Vandana Mishra’s family, after her death in Kanpur
late Friday night, when the car carrying her to hospital was stopped for the presidential convoy to pass. But while the
President’s response is in keeping with obligations of his high office, the Kanpur tragedy is by no means unique. VVIP
convoys have become yet another manifestation of a political and administrative culture where public servant’s self-
esteem depends on the distance they can maintain from the public. Police are overzealous because they respond to a
system where public inconvenience is extremely low priority.
VVIP convoys in Indian cities are particularly unwelcome. Most cities suffer terrible traffic congestion. In Mumbai it is
estimated that a trip takes about 53%more time than it should, in Bangalore 51% and in New Delhi 47%. On top of this
there are few green corridors for ambulances. So, road blockades set up to smoothen VVIP movement worsen the situation
painfully. There was hope following the 2017 Union Cabinet decision to end the lal batti raj. But long snaking VVIP convoys
aren’t part of this change. And MLA’s from Haryana to Himachal Pradesh have tried to retain traffic privileges with flags
atop their vehicles.
By contrast, in America the idea of holding up citizens for hours for the convenience of politicians would be laughter. In
New Zealand, when the PM’s motorcade was caught over the speed limit, it was fined for dangerous driving. In
Scandinavian countries, far from reserving a faster lane for themselves, netas take public transport shoulder to shoulder
with citizens. This is the direction our democracy must take too. For VVIP protection, the state must deploy smarter
security arrangements, and nix those convoys. When public servants are driven, let the public not be driven to despair or
worse.
85. Which among the following is the most significant (a) The traffic situation in Indian and Western
message conveyed by the author of the passage? countries is broadly similar.
(a) The VVIP system is not suitable for India since it (b) All countries being mentioned are Europeans.
causes inconvenience to common people. (c) Time is the essence of life.
(b) A public servant’s self-esteem carries a lot more (d) Politicians should be held responsible for their
significance than that of a common man. actions.
(c) It is time that the state does away with convoys
and brings up a smarter security system for 89. ‘In Scandinavian countries, far from reserving a faster
VVIPs. lane for themselves, netas (political leaders) take
(d) The President should be held responsible for the public transport shoulder to shoulder with citizens.’
death of Vandana Mishra. Which of the following statements, from the options,
would weaken the argument?
86. Which of the following courses of action should India (a) The political leaders have been put on a pedestal
take from the various countries mentioned in the by the people who wish to be in awe of their
passage? leaders.
I. Hold up citizens for long hours for politicians. (b) The political leaders are known for their
II. Levying a penalty on the vehicles of the politicians indifference towards development and people may
for violating traffic rules. vent out their anger on the leaders, if the leaders
III. Oblige politicians to use public transport. were to mingle with people.
(a) Only I (b) Only II (c) The leaders being the representative of people face
(c) Both II and III (d) All I, II and III constant death threats and are under the radar of
those who want to sabotage the development,
87. Which of the following assumptions must be true? cannot be treated as commoners as they hold
(a) The tragedy that happened due to the convoy of position of relevance.
the President is one of the last such tragedies. (d) The leaders are representative of people and
(b) At least one of the political and administrative conducting themselves with humility will only
VVIP manifestations should be done away in strengthen their support amongst their people.
India.
(c) Deploying smarter security systems for VVIP 90. Given the concerns raised in the passage about VVIP
convoys is a one-stop solution for the perils of convoys and their impact on public inconvenience,
democracy. which of the following actions should the government
(d) VVIP convoys should not be done away with in a prioritize to address this issue effectively?
democracy such as India. (a) Implement a strict penalty system for VVIP
convoys that cause traffic disruptions, similar to
88. While giving examples of other countries, which among the fine imposed on the PM's motorcade in New
the following is the central flaw made by the author? Zealand, to discourage such practices.
215
(b) Increase the number of VVIP convoys and and their role in governance to foster public
enhance their security to ensure that public understanding.
servants can perform their duties without (d) Invest in infrastructure improvements to create
interruptions. dedicated lanes for VVIP convoys, thereby
(c) Conduct a public awareness campaign minimizing their impact on regular traffic flow.
highlighting the importance of VVIP movements
Passage(Q.91-Q.96): In its most recent meeting, the monetary policy committee of the RBI chose to maintain the policy
status quo, keeping the benchmark repo rate at 4 percent and continuing with its accommodative stance, presumably as
long as is necessary to revive growth. But the decision was not unanimous. One of MPC’s members voted not only against
continuing with the accommodative stance, but also argued in favour of raising the reverse repo rate. Concerns over how
long the MPC can continue with its policy stance appear to be gaining traction. Even though the other members stuck to
their earlier positions, there appears to be a shift in the tone of the commentary, with another member arguing that “over-
stimulus as after the global financial crisis, with delay resulting in sharp adjustment, has to be avoided.” So far, the MPC
has continued to attach primacy to growth considerations. In their most recent comments, most members highlighted the
uneven nature of the recovery — the scarring of small business and the informal sector. According to another MPC
member, of the 404 industries for which data is available, 63.4 percent are operating below 2018-19 levels. The RBI
Governor also argued that the “informal sector is likely to take even longer to recoup as the impact of the second wave on
this sector was relatively more pronounced.” But the space for continuing with its current stance appears to be receding.
On the inflation front, the MPC members were cognisant of the risks, as, even though headline inflation has dipped — CPI
fell to 4.35 per cent in September, down from 5.3 per cent in August — core inflation remains elevated. As the member
who voted against continuing with the accommodative stance noted, “inflationary pressures are beginning to show signs
of greater persistence than anticipated earlier”. Economists expect inflation to inch back upwards. According to the RBI’s
projection, it is expected to dip to 4.5 per cent in the third quarter, rising thereafter to 5.8 per cent in the fourth quarter.
The withdrawal of policy support is likely to be gradual with the process playing out at multiple levels — beginning with
normalisation of liquidity, which has begun, followed by a hike in the reverse repo, a shift in the stance from
accommodative to neutral, culminating in a repo rate hike. However, the duration over which this plays out is uncertain.
Perhaps a few more months of data will provide greater clarity over the durability of the recovery, allowing the MPC to
firm up its exit plans.
[Extracted with edits and revisions from The Indian Express]
91. Which of the following is true according to the passage? 93. Which of the following can be inferred from the given
(a) There is a clear plan over the duration of the passage?
withdrawal of policy support. (a) Disagreement between members of an
(b) The RBI chose to continue with the present repo organization is a sign of its bleak future.
rate with the consent of all its members. (b) The RBI is a body which aggravates the already
(c) The decision of the RBI to maintain the status quo fragile situation of the weaker sectors.
met with some resistance from the members of the (c) The RBI’s stance on growth considerations is likely
MPC. to change in future.
(d) The fall of CPI refers to an increase in the inflation (d) The RBI will implement drastic policy changes in
rate, according to the Reserve Bank of India. the coming months to immediately address
inflation concerns.
92. Out of the following, which one is the best
representation of the main message of the author in 94. For which of the following claims has the author
the passage? presented some pieces of evidence?
(a) It will need greater clarity over durability of (a) Till now, the stance of the Monetary Policy
recovery for RBI to firm up its moves towards Committee on growth considerations has not
policy normalisation. changed.
(b) Policy normalisation is a distant goal for the RBI, (b) The decision of the Monetary Policy Committee to
firstly it needs to bring the members of the MPC to maintain the status quo was not unanimous.
an agreement. (c) The withdrawal of policy support by the Monetary
(c) The informal sector was one of the hardest hit Policy Committee is likely to be abrupt.
sectors due to the second wave and all efforts must (d) Both a and b
be made to revive it.
(d) The difference in opinion of the members of the 95. ‘In its most recent meeting, the monetary policy
MPC over a subject of particular concern is committee of the RBI chose to maintain the policy
disturbing, which might hamper its efficiency. status quo, keeping the benchmark repo rate at 4
percent and continuing with its accommodative stance,
216
presumably as long as is necessary to revive growth. (a) A significant number of economists have begun to
But the decision was not unanimous. One of MPC’s express concern that prolonged low-interest rates
members voted not only against continuing with the could lead to asset bubbles in the real estate and
accommodative stance, but also argued in favour of stock markets, which could destabilize the
raising the reverse repo rate.’ Which of the following economy.
reflects the assumption(s) behind the passage? (b) Recent reports show that inflation expectations
I. There are more than one committee that governs among consumers and businesses have risen,
the monetary policies of the RBI. suggesting a shift in sentiment that could influence
II. The decision to maintain the policy status quo with actual inflation rates moving forward.
regard to repo rate was unanimous. (c) The data indicate that the recovery of the informal
(a) Only assumption I is implied. sector has been slower than anticipated, leading to
(b) Only assumption II is implied. increasing financial strain on small businesses,
(c) Both assumptions I and II are implied. which necessitates continued support.
(d) Neither assumptions I nor II is implied. (d) An analysis of the global economic outlook reveals
that major economies are poised for growth,
96. Which of the following statements, if true, would most reducing the likelihood of a synchronized global
strengthen the case for the RBI to gradually transition economic downturn.
from its current accommodative stance to a more
neutral or restrictive monetary policy?
Passage (Q.97-Q.102): While the pandemic has forced many in the confines of their homes, the worst hit are the elderly
who are devoid of any human contact and without any contact in the real world. Among others, health care, medical
services and care for the elderly is something that is becoming hard with times like these. So, what is it that can bring a
drastic change in the life of elderly; how can ageing be better with smarter technology? Let us discover.
Big data and artificial intelligence (AI). With the amalgamation of these two technological advanced tools, it is possible
now to usher in personalized medicine, treatments that are tailor made for individual, and all this brings us to
revolutionize the health care system. It brings together all the aspects including information on the lifestyle of the
individual, biological aspect and clinical aspect all these help in creating a unique blueprint of the individual or the patient.
It becomes easier to begin with target therapies for achieving the best outcome whether it is about prevention or to
manage the disease. Early detection of disease becomes significantly easier that provides the medical practitioner with
the power to easily diagnose and provide accurate treatment for the same. Life threatening diseases then can be handled
in a better manner like the onset of cancer and Alzheimer’s.
Monitoring the home of the elderly, providing timely medical supplies and help, smart heating devices or temperature
control and other such things makes it easier for the ageing people to lead a comfortable life.
Helping the aged people deal with the isolation is yet another factor that is essential to be considered as they are more at
risk than the rest of the population.
The time elderly people were active socially that was pre-pandemic time their longevity was positively impacted. They
felt healthier and happier that kept them from falling ill frequently, but with the onset of pandemic things changed
dramatically.
Digital technology is providing a breakthrough in relation to wellbeing for the elderly. Connecting people with elderly
population and running their errands, maintaining contact with them and taking on tasks for them, checking upon them
is all made possible. Facilitating real-time interactions helps in reducing social isolation.
97. According to the author, big data and artificial 98. ‘Monitoring the home of the elderly, providing timely
intelligence will help the elderly medical supplies and help, smart heating devices or
(a) By bringing in treatments that are tailor made for temperature control and other such things makes it
individuals. easier for the ageing people to lead a comfortable life.’
(b) By including information on the lifestyle of the What is the assumption behind the argument?
individual, biological aspect and clinical aspect all (a) The aging people require monitoring on the basic
these help in creating a unique blueprint of the functioning of life.
individual. (b) The aging people are unable to manage the
(c) By early detection of disease becomes significantly simplest of tasks.
easier that provides the medical practitioner with (c) The aging people live alone without anyone
the power to easily diagnose and provide accurate monitoring.
treatment for the same. (d) The aging people are unable to perform the basic
(d) All of the above. functioning.
217
99. Which of the following will undermine the contention (d) With the help of artificial intelligence, facilitating
presented in the passage? real-time interactions will help in reducing social
(a) With the Pandemic out of the system, life will isolation.
resume back to normal, even for the elderly people. 101. Which of the following reflects the main idea of the
(b) The aging people need support of the NGOs and passage in brief?
only NGOs can make the life better for the aging (a) Ageing reverse with smarter technology.
people. (b) Ageing better with smarter technology.
(c) The aging people are technology savvy and are able (c) Aging made more comfortable.
to figure out artificial intelligence systems. (d) Aging post Pandemic
(d) The ageing people with their weak faculty and
102. Which of the following identifies a flaw in the argument
fugacious memory are unable to perform a task as
that integrating big data and artificial intelligence (AI)
simple as dialing a number on their mobile.
can significantly improve the quality of life for the
100. If the arguments in the passage are true, how will elderly during the pandemic?
artificial intelligence help in making the ageing people’s (a) The passage assumes that all elderly individuals
life comfortable? have access to technology and the internet, which
(a) Using AI, the elderly can be made to learn multiple may not be the case for everyone.
tasking, thereby easing their lives and keeping (b) The passage relies on anecdotal evidence about the
them busy so that they do not feel isolated. social lives of elderly individuals before the
(b) Using AI with human values, data related to each pandemic without considering the impact of other
elderly fed into the system used for analysis, factors on their well-being.
medical health care and other services available (c) The passage does not acknowledge that
through smart devices is where technology will advancements in technology can also lead to
make a viable impact. increased stress and anxiety for some elderly
(c) Data related to each elderly will be fed into the individuals.
system that can help in assisting medical (d) The passage emphasizes the benefits of
emergencies and hospital care. personalized medicine while failing to discuss the
potential costs associated with implementing such
technologies.
Passage (Q.103-Q.108): The main task of education and research is to train people to perform in future economic and
technological environments with many unknowns. Workers prepare to tackle unknown problems using instruments yet
to be developed. In truth, we know embarrassingly little about tomorrow’s jobs. Interdisciplinary and holistic education
and research are indispensable as the workplace combines cognitive skills with teamwork and debate with focus on
adaptability, replacing silo thinking with a flexible approach that applies knowledge from multiple sectors that at first
appearances may not seem relevant.
Education and research are increasingly out of touch with demand for skills. The tendency to focus on cognitive skills,
including the STEM topics of science, technology, engineering, and mathematics, cannot preclude the productivity benefit
of soft skills – applying knowledge and finding opportunities offered by technology. In recent decades most countries have
fallen into the trap of overextending cost-benefit analyses while training students to solve yesterday’s problems.
Governments enforce short-term fiscal planning on education programs, yet measuring social skills is not as easy as
calculating STEM competences.
Much attention is devoted to high-calibre education and research and for good reason, but demand trends suggest that
human factors may be more essential than normally assumed. For example, health, nonstop improving of skills and
entertainment may prove to be the growth sectors of the future – and main job providers. Human maintenance will grow
almost exponentially steered by demographics and a growing proportion of elderly citizens. Human improvement, the
ability to try new technologies should not be overlooked as higher productivity embedded in new technology only
blossoms if humans have the skill to manage technology. Entertainment follows from a shorter work life and longer
retirements. Elderly people are more active than previous generations and demand health care, entertainment, social
networks and communication. They need coaches, presenting huge job openings.
103. What has the author conveyed regarding the current (c) It has somehow lost touch with the need for soft
pattern of Education and Research? skills.
(a) Current education and research patterns have no (d) It nowhere relates to applying knowledge and
productivity benefits. finding opportunities offered by technology.
(b) It is less inclined towards providing STEM
competences.
218
104. Which of the following can be deduced from the above (d) Enforcing more short-term fiscal planning on
passage? education programs.
(a) There is a need to identify the evolving job trends
107. In the above passage, what does the author mean when
and focus on many in-demand cognitive skills.
he says “Much attention is devoted to high-calibre
(b) Acquiring academic knowledge and mapping the
education and research and for good reason, but
set standards should be the ultimate goal.
demand trends suggest that human factors may be
(c) Overcoming silo thinking and gaining flexibility at
more essential than normally assumed.” Which of the
work must be a priority.
following reflects the central flaw in the reasoning
(d) The current education and research patterns need
above?
to focus more on enhancing soft skills to help tap
(a) The government’s focus is more on high-calibre
the growing sectors of the future.
education and research than focusing on human
105. As per the passage, which of the following does not factors.
correctly represent the author’s view regarding (b) The author assumes the high-calibre education
education and research? and research does not take into account human
(a) Education and Research must focus on skills that factors.
involve conscious intellectual effort. (c) High-calibre education and research and human
(b) Education and Research trains people to enable factors are intra-connected.
them to perform in future economic and (d) The demand for the human factors in the future
technological environments. will diminish the high-calibre education and
(c) One cannot do away with human improvements research.
that relates to the ability to try new technologies.
108. Which of the following statements is the author least
(d) The new cognitive working environment demands
likely to agree with regarding the future of education
extracting the students out of the archaic solving of
and research?
yesterday’s problems.
(a) Emphasizing STEM education alone will
106. Which of the following steps must be taken to help adequately prepare students for the uncertainties
education and research become Interdisciplinary and of future job markets.
holistic? (b) Education systems need to integrate soft skills to
(a) Preparing workers to tackle unknown problems of ensure adaptability and innovation in the
the future. workplace.
(b) Condemning the system that lays much (c) A strong focus on human factors in education will
importance to cognitive skills and STEM be critical to meet the needs of an aging
competences. population.
(c) Realising the value of social skills and inculcating (d) None of the above
human improvement along with the existing focus
on cognitive skills and STEM knowledge.
219
SECTION-E : QUANTITATIVE TECHNIQUES
Direction (Q.1-Q.6): The volumes of three vessels are in the ratio 1: 2 : 4. The volume of the second vessel is 40 L. The
first vessel is empty, the second is filled with milk and the third is filled with water. 50% of the volume from the second
vessel is transferred to the first and that is replaced with the liquid from the third vessel. The contents of the first vessel
is now emptied into the third vessel. This operation is done once more.
1. What is the ratio of milk to water in the first vessel at
the end of the second operation, just before it is 4. What is the total volume of liquid in the third vessel
emptied into the third vessel? after the second operation?
(a) 1:4 (b) 1:3 (a) 60 (b) 70 L
(c) 1:1 (d) 3:1 (c) 80 (d) 90 L
2. What is the quantity of milk in the third vessel after 5. What is the ratio of the total liquid in the third vessel to
second operation? the second vessel after both operations?
(a) 20 L (b) 25 L (a) 3:1 (b) 2:1
(c) 30 L (d) 35 L (c) 1:2 (d) 1:3
3. What fraction of the third container comprises milk 6. After the second operation, what is the ratio of milk to
finally? water in the third vessel?
(a) 5:16 (b) 5:11 (a) 2:3 (b) 5:11
(c) 16:5 (d) 11:5 (c) 5:7 (d) 3:5
Direction (Q.7 – Q.12): In a class, there are 200 students who took the exam in three subjects Maths, Physics and
Chemistry. It is known that the number of students who passed in at least one subject is thrice that of the students who
passed in exactly one subject. The number of students who, passed in exactly one subject is thrice the number of students
who did not pass in any subject. The number of students who passed in all the three subjects is twice the number students
who failed in all the three subjects.
7. How many students passed in exactly two subjects? (a) 140 (b) 120
(a) 60 (b) 100 (c) 160 (d) 100
(c) 80 (d) 40
11. What is the ratio of students who passed in exactly
8. How many students passed in exactly one subject? one subject to those who passed in all three subjects?
(a) 40 (b) 80 (a) 3:2 (b) 4:1
(c) 100 (d) 60 (c) 3:1 (d) 6:1
9. How many students who did not pass in any subject? 12. What percentage of the students passed in at least one
(a) 20 (b) 10 subject?
(c) 15 (d) 25 (a) 75% (b) 80%
10. In the class, what is the total number of students who (c) 85% (d) 90%
passed in exactly one or two subjects?
220
ANSWER KEY AND EXPLANATIONS
SECTION-A : ENGLISH LANGUAGE (c)- There is no chronology of events being discussed in the
1. (a) Option (B) is the correct answer. Statement A is said passage, so, a newspaper article is unlikely to be the
incorrect because the passage explicitly states that while source of the passage.
desire is important, it is not sufficient for facilitating (d)- The passage talks about individual, in particular and
change; a compelling reason and additional factors are suggested approach towards goal or change attainment;
necessary. Statement B is correct, as the passage it doesn't delve into the social aspects or impact of
highlights that the capacity for change begins with self- society in the said process or journey, just a passing
awareness and the establishment of measurable goals. reference while discussing importance of social
Statement C is also correct; the text emphasizes the role support). (a)- The passage talks about step-by-step
of social support in enhancing the effectiveness and procedure to achieve the desired outcome or change. At
stability of personal change efforts. Conversely, the start, it sets the agenda for the passage of self-
Statement D is incorrect because the author confesses improvement and ways to achieve the same. Therefore,
their inability to break the late-night habit even after a motivational novel is most likely to be the source of the
completing graduate school, indicating that they did not given passage. Hence, (a) is the right answer.
succeed in making that change. Finally, Statement E is 5. (b) (b)- The usage of 'must' indicates or hints towards the
correct, as the passage supports the idea that personal indispensable nature of effort and its optimum degree,
development can lead to a complete reinvention of one’s which forms the underlying premise for the author to
personality. make such a statement. Hence, (b) is the right answer.
2. (a) The passage underscores the importance of having a (a)- It's a blanket statement and cannot be the underlying
compelling reason for change, suggesting that desire premise for the statement as it might not hold true for all
alone is insufficient to ensure commitment and success. (usage of 'usually', 'often' would have made the sentence
Without a strong motivation driving his efforts, Mr. X a suitable premise).
may struggle to maintain consistency and dedication in (c)- It might be true, in general, but can't be the premise on
his weight-loss journey. While option B, regarding social which the said statement is based upon as there is no talk
support, is also a valid consideration, the passage about variation in challenge.
emphasizes that a compelling reason is paramount for (d)- The main statement doesn't talk about the consequence
making lasting changes. Options C and D are incorrect; of a situation where inappropriate effort is not given to
the passage does not indicate that Mr. X lacks the desire address the challenge; so, it can be the underlying
to change, nor does it suggest that he does not have premise.
measurable goals, as it explicitly states he is engaged in 6. (b) Option B is correct because the passage explicitly states
a targeted and measurable weight-loss journey. that the capacity to change starts with self-awareness,
3. (B) Option B is correct because the passage emphasizes that highlighting its role as a foundational element for
self-awareness is essential for initiating personal change establishing targeted and measurable goals. This
and setting measurable goals. It implies that without emphasizes that self-awareness is essential for initiating
understanding one’s strengths and weaknesses, personal development. In contrast, Option A
individuals may struggle to identify the need for change misrepresents the sequence of events, suggesting self-
or the strategies required to achieve it. In contrast, awareness is a result of change rather than a precursor.
Option A is incorrect as the passage suggests that Option C incorrectly asserts that self-awareness is
personal development requires considerable effort irrelevant, while the passage clearly indicates its
rather than being effortless. Option C misrepresents the importance. Lastly, Option D suggests that self-
passage's message, as it does not imply that change can awareness leads to unrealistic perceptions of personal
occur without any external support; rather, it deficits, which is not supported by the text; instead, the
emphasizes the importance of social support in passage suggests that recognizing deficits is necessary
enhancing the likelihood of successful change. Option D for meaningful change.
incorrectly narrows personal development to physical 7. (b) Through the statement, the author implies that the very
changes, while the passage discusses a broader range of mention of a robotic surgery brings to mind the fact that
improvements, including psychological and habitual surgeries will be performed by robots, and hence the
changes. author dispels the concern that, rest assured, it is the
4. (a) (a)- The passage talks about step-by-step procedure to surgeon who will be at the helm of a surgery, largely,
achieve the desired outcome or change. At the start, it non-invasive surgeries with the help of technology.
sets the agenda for the passage of self-improvement and Therefore, option (b) correctly interprets author’s
ways to achieve the same. Therefore, a motivational stance. Option (d) might appear to be the answer, but on
novel is most likely to be the source of the given passage. a closer look it is more of an extension of the main
Hence, (a) is the right answer. statement. It is not the interpretation or the concern that
(b)- The science behind humans and their goal attainment the author eases. The author in option (d), which is also
isn't part of the passage; rather, the association or a direct extract from the passage, provides an
dissociation between the two has been dealt from a explanation of how the robotic surgery will take place.
human perspective; therefore, scientific journal can't be Options (a) and (c) are fer-fetched interpretations, not
the source of the passage. even remotely suggested by the author throughout the
passage.
221
8. (c) The passage belongs to the area of science. The trend in the organisation. Hence, the answer is option b).
terminologies discussed in the passage (decreased need Option (a) is not the intended objective behind
for narcotics, inguinal hernia repair, Tylenol or discussing positive examples of ethical behaviour.
ibuprofen, outpatient robotic colectomies etc) are very Options (c) and (d) are irrelevant to the main objective.
specific; hence, option (c) is the correct answer. 14. (b) The word 'compliance' means 'the action or fact of
The tone is not deliberating, as nowhere in the passage complying with a wish or command'. Hence, the answer
is there a reflection on the robotic surgery. The tone is is option b). Option (a) is contrary to the meaning of the
not incendiary, as the author has not used inflammable passage. Option (c) and (d) are not the meanings of the
language to suggest so. Mordacious means biting or word ‘compliance’.
pungent, which is far from the tone used in the passage. 15. (b) The whole passage talks about ethical behaviour, its
9. (d) Option (d) cannot be cited as an example from the given importance and how the behaviour and role of a leader
passage; therefore, is the correct answer. The rest of the direct the course of action of its employees. So, the
options have been mentioned in the passage. For option importance of ethical behaviour is highlighted by the
(a), refer to the lines, ‘the researchers reported that author. Hence, the answer is option b). The author would
robotic surgery, which involves surgeons guiding be likely to disagree with other options presented here.
minimally invasive instruments, remotely reduces the (a}- the author finds the role of the leader to be much
chance of re-admission to the hospital by 52 percent.’ more important. (c)- a leader needs to be enthusiastic
And, ‘Experts note that less-invasive robotic procedures towards its employees. (d)- ethical behaviour is
can mean less pain during recovery, which means less expected during all times, as suggested by the author
pain medication.’ The aforementioned lines take care of pointing out the flaw of ethical behaviour discussion
option (a) and (b) as well. Option (c) can be referred only during the times of scandals in the first few
from the lines, ‘Robotics is simply a more precise way to sentences of the passage.
minimize invasion, which makes for quicker healing.’ 16. (d) This option relates to leaders being aware of their actions
10. (a) Option (a) is the correct answer for the given question. and how those actions affect others, similar to the
The author writes about the benefits of the robotics in passage's emphasis on leaders considering the impact of
the abdominal procedures and goes onto write about the their behaviour on their team. Option (a) does not align
different records and findings. In a way, the author with the passage as it focuses on police tactics rather
indicates the robotic surgery as the future (also note the than leader empathy and understanding team dynamics.
comparison with the traditional surgery). Option (b) is Option (b), while discussing public acknowledgment,
incorrect because the passage explicitly states that lacks the emphasis on self-awareness and the impact of
"humans are still at the controls in these procedures." It leaders’ actions on their team. Option (c) addresses
emphasizes that while robotic surgery utilizes advanced workplace aggression but is unrelated to the positive
technology, surgeons are actively guiding the qualities of leadership or the importance of
instruments and making decisions during the surgery.. understanding unintended messages.
Options (c) and (d) can only be true, if something on the 17. (a) Option a) is correct as per the fourth sentence of the
similar lines had been mentioned in the passage, for second paragraph of the given passage, which says that
these two options to be the questions. Since no answer an ethical leader remains cautious about signalling that
can be derived from the passage, regarding the hypocrisy is alright in the organisation. Other options
information that the questions seek, these two options are incorrect as per the given passage. Hence, the answer
are irrelevant. is option a).
11. (b) The verb translate will take the preposition ‘to’, while’ 18. (b) Option (B) is correct because the passage emphasizes the
length’ and ‘stay’ being nouns will have the preposition importance of leaders being mindful of how their
‘of’. Therefore, the sentence reads, ‘The improved feedback and actions can be perceived by others,
recovery times also translate to a shorter length of stay.’ particularly regarding ethical implications. The text
‘In’ and ‘at’ as prepositions for the respective blanks, are illustrates this with the example of a leader downplaying
unsuited contextually. critical data in a presentation, which inadvertently
12. (c) Option (C) is correct because Dr. Bethany Malone teaches employees that it's acceptable to hide the truth
specifically links the reduced need for narcotic pain for desired outcomes. In contrast, option (A) is incorrect
medications after robotic surgery to the ongoing opioid because it suggests leaders should avoid ambiguity by
crisis, emphasizing the importance of finding safer sticking strictly to factual information, which doesn't
alternatives for pain management. In contrast, option address the ethical implications of communication.
(A) is too general as it focuses on the overall Option (C) misrepresents the passage's focus, as it
effectiveness of robotic surgery rather than addressing highlights the need for leaders to consider the
pain management; option (B) inaccurately suggests a interpretations of others rather than prioritizing their
primary concern about healthcare costs, which is not own intentions. Lastly, option (D) is also incorrect
mentioned in the passage; and option (D) incorrectly because it implies that leaders should only provide
emphasizes patient satisfaction without Dr. Malone positive feedback to maintain morale, which overlooks
directly discussing it. the necessity of addressing ethical issues and fostering
13. (b) Option b) is correct as per the last sentence of the first honest communication within the organization.
paragraph of the given passage, which says that 19. (b) Refer to the lines: “One way people reflect on these
"reinforcing the good things people do strengthens questions is by sitting in silence in meditation.”. Through
ethical choices as 'the norm' of the organisation." i.e., to these lines, it is clear that option B is the answer. Option
encourage people to make ethical choices and make it a A is ruled out as it is nowhere mentioned in the passage.
222
Option C is also ruled out as it finds no support in the derived from life experiences, overlooking the deeper
passage. Option D is contrary to what has been said in consciousness that meditation reveals.
the passage, where the author says that the answers to
the questions pertaining to life-purpose can not be SECTION-B : GENERAL KNOWLEDGE
sought on the internet. 25. (d) The Constitution (One Hundred and Twenty-Eighth
20. (c) Refer to the line: “It is not the intellectual knowledge that Amendment) Bill, 2023, also known as the Women's
we learn from lectures or books….” This lines clearly is a Reservation Bill or the Nari Shakti Vandan Adhiniyam,
contradiction to option A, which makes the option A aims to reserve one-third of seats for women in the Lok
incorrect. Now refer to the lines: “Our soul, being the Sabha, state legislative assemblies, and the Delhi
same essence as God, is consciousness. It is a ‘state of Assembly.
knowing all that is to be known”. And “When we access • The bill was introduced in the Lok Sabha on
the divine wisdom, we reach a state of all-consciousness, September 19, 2023.
one in which we know the answers to life’s mysteries • The bill will be effective after the census following
and our purpose in life.”, it is clear form these lines that the bill's commencement, and will be in place for
Option B and C are correct with regard to the question 15 years.
above. • The bill will also extend the reservation to seats
21. (a) The gist of the lines are that as we reach our inner reserved for SCs and STs in the Lok Sabha and
consciousness and gain deeper wisdom within ourselves, state legislative assemblies.
we look at ourself and the world differently and play a • The bill will rotate seats reserved for women after
more active rather than just watch the events affecting us, each delimitation.
without seeking to change them or find some answers. • The bill does not include a quota for women from
Based on the gist, option A is closest in interpretation. the Other Backward Classes (OBCs).
Option B is a rather tweaked version of the given lines. 26. (a) The Women Reservation Bill 2023, formally known as
Option C though partly thru, deviates from the core issue the 128th Constitutional Amendment Bill or the Nari
as the phrase searching for their role assigned to them is Shakti Vandan Adhiniyam, has recently taken center
not the gist. Option D is contrary to what is given in the stage in Indian politics. This landmark legislation aims to
sentences. reserve one-third of seats in the Lok Sabha and State
22. (a) A title carries the basic idea of the author’s thought Assemblies for women.
process. It should be connected to the over all essence of a 27. (c) The reservation for women, as per the Bill, is intended to
passage. Based on the above information, option A is best last for an initial period of 15 years. However, it may
suited as it carries the essence of the passage. The last continue till a later date as determined by a law made by
paragraph which is also the basic idea that the author Parliament.
communicates through the passage is best represented as 28. (c) Article 330A stipulates the reservation of 47 seats in the
the title given in option A. Option B is close but meditation Lok Sabha for Scheduled Tribes. This constitutional
is not the main idea here, it is tool to reach the inner provision is aimed at ensuring proportional
wisdom. Also, the title is disjointed as it seems that representation and political empowerment for the
meditation and consciousness are two different entities. Scheduled Tribes, acknowledging their historical socio-
Option C is vague; hence, is incorrect and option D is close economic disadvantages.
but if one had to choose between option A and D, option A 29. (b) The Delimitation Commission has been constituted four
is better in terms of clarity. times in India— in 1952, 1963, 1973, and 2002. Each
23. (d) Astute means clever, shrewd or one with keen instance was aimed at readjusting constituency
discernment. One who is perceptive. boundaries based on changing demographic factors and
Sagacious means having or showing mental keenness. political considerations.
Shrewd; this is clearly on of the synonyms. 30. (c) As from the date of commencement of the Constitution
Ingenious is characterised by cleverness or originality of (Sixty-ninth Amendment) Act, 1991, the Union territory
invention; cleverly inventive or resourceful. This is a of Delhi shall be called the National Capital Territory of
synonym of ‘astute’. Delhi (hereafter in this Part referred to as the National
Perspicacious means having keen mental perception and Capital Territory) and the administrator thereof
understanding; acute and shrewd. This is very close to the appointed under article 239 shall be designated as the
meaning of ‘astute’. Lieutenant Governor.
Imbecile means a dunce, a blockhead. A simple-minded. Article 239AA grants the Legislative Assembly of the
This is opposite in meaning to the word ‘astute’. Clearly, NCT the power to make laws for the whole or any part
option D is the correct answer. of the NCT with respect to any matters in the State List
24. (b) Option B is the correct answer because the passage or Concurrent List, insofar as such matters are
specifically highlights that true wisdom resides within applicable to Union territories. However, it explicitly
each person and can be accessed through meditation, excludes the subjects of police, public order, and land
which opens doors to self-discovery and inner from the Legislative Assembly's purview.
understanding. In contrast, A) is incorrect as it limits 31. (c) CAR T-cell therapies are approved by the US Food and
wisdom to academic knowledge, while C) suggests Drug Administration (FDA) to treat some kinds of
relying on external discussions with spiritual leaders, lymphomas and leukemias, as well as multiple myeloma.
which the passage does not emphasize. D) is also CAR T-cell therapy is typically used after other types of
incorrect because it implies that wisdom is solely treatment have been tried.
223
Examples of CAR T-cell therapies currently approved 37. (d) The Union Minister of Road Transport and Highways
include: Nitin Gadkari has launched the much-awaited Bharat
• Tisagenlecleucel, also known as tisa-cel (Kymriah) NCAP, India’s own crash safety rating system. The
• Axicabtagene ciloleucel, also known as axi-cel assessment programme will be effective come October 1,
(Yescarta) 2023. The Ministry, during the launch event held in the
• Brexucabtagene autoleucel, also known as brexu- capital, said that the programme will be voluntary and
cel (Tecartus) the cars will be tested as per the Automotive Industry
• Lisocabtagene maraleucel, also known as liso-cel Standard (AIS) 197.
(Breyanzi) 38. (a) NCAP stands for New Car Assessment Program, which is
• Idecabtagene vicleucel, also known as ide-cel an internationally recognized system for rating the
(Abecma) safety of motor vehicles. Various countries have their
• Ciltacabtegene autoleucel, also known as cilta-cel own NCAP programs to evaluate and inform consumers
(Carvykti) about the safety of vehicles.
32. (c) In CAR T-cell therapies, T cells are taken from the 39. (d) The Peugeot Partner Patagónica received a 'zero star'
patient's blood and are changed in the lab by adding a safety rating in the Latin NCAP and Global NCAP car-to-
gene for a receptor (called a chimeric antigen receptor car campaign, indicating very poor safety performance.
or CAR), which helps the T cells attach to a specific 40. (c) Global NCAP initiated its "#SaferCarsForAfrica"
cancer cell antigen. The CAR T cells are then given back campaign in 2017. This campaign aims to assess and
to the patient. promote vehicle safety standards in African markets,
Since different cancers have different antigens, each focusing on improving safety for consumers.
CAR is made for a specific cancer's antigen. For 41. (c) The Fair Credit Reporting Act (FCRA), Public Law No. 91-
example, in certain kinds of leukemia or lymphoma, the 508, was enacted in 1970 to promote accuracy, fairness,
cancer cells have an antigen called CD19. The CAR T- and the privacy of personal information assembled by
cell therapies to treat these cancers are made to attach Credit Reporting Agencies (CRAs).
to the CD19 antigen and will not work for a cancer that The year the Fair Credit Reporting Act (FCRA), Public
does not have the CD19 antigen. Law No. 91-508, was passed by the U.S. Congress to
33. (d) CAR T-cell therapies are approved by the US Food and promote the accuracy, fairness, and privacy of personal
Drug Administration (FDA) to treat some kinds of information collected in credit reports. It has been
lymphomas and leukemias, as well as multiple myeloma. amended a number of times in the years since.
CAR T-cell therapy is typically used after other types of 42. (c) CRAs assemble reports on individuals for businesses,
treatment have been tried. including credit card companies, banks, employers,
Examples of CAR T-cell therapies currently approved landlords, and others. The FCRA provides important
include: protections for credit reports, consumer investigatory
1. Tisagenlecleucel, also known as tisa-cel (Kymriah) reports, and employment background checks. The FCRA
2. Axicabtagene ciloleucel, also known as axi-cel is a complex statute that has been significantly altered
(Yescarta) since 1970 by Congress and the courts. The Act’s
3. Brexucabtagene autoleucel, also known as brexu- primary protection requires that CRAs follow
cel (Tecartus) “reasonable procedures” to protect the confidentiality,
4. Lisocabtagene maraleucel, also known as liso-cel accuracy, and relevance of credit information. To do so,
(Breyanzi) the FCRA establishes a framework of Fair Information
5. Idecabtagene vicleucel, also known as ide-cel Practices for personal information that include rights of
(Abecma) data quality (right to access and correct), data security,
6. Ciltacabtegene autoleucel, also known as cilta-cel use limitations, requirements for data destruction,
(Carvykti) notice, user participation (consent), and accountability.
34. (b) To alter T cells to attach to specific cancer cell antigens 43. (a) The FCRA was passed to address a growing credit
They are made by collecting T cells from the patient and reporting industry in the United States that compiled
re-engineering them in the laboratory to produce “consumer credit reports” and “investigative consumer
proteins on their surface called chimeric antigen reports” on individuals. The FCRA was the first federal
receptors, or CARs. The CARs recognize and bind to law to regulate the use of personal information by
specific proteins, or antigens, on the surface of cancer private businesses.
cells. The first major credit reporting agency, Retail Credit Co,
35. (c) Union Minister of Health and Family Welfare, Shri Jagat was started in 1899. Over the years, Retail Credit
Prakash Nadda, inaugurated the second edition of the purchased smaller CRAs and expanded its business into
Global Food Regulators Summit 2024 in the presence of selling reports to insurers and employers. By the 1960s,
Shri Pralhad Joshi, Union Minister of Consumer Affairs, significant controversy surrounded the CRAs because
Food and Public Distribution, and Minister of New and their reports were sometimes used to deny services
Renewable Energy at Bharat Mandapam. and opportunities, and individuals had no right to see
36. (c) The frontal crash test under Bharat NCAP protocols is what was in their file.
conducted at a speed of 64km/h, as indicated in the 44. (b) The Federal Trade Commission (FTC) issues
provided information. This specific speed is critical for commentaries on the statute, but does not engage in
assessing the safety and crashworthiness of vehicles rulemaking for the FCRA.
during frontal impacts. CRAs may also be referred to as “credit bureaus” or
“consumer reporting agencies.”
224
Washington, DC, the U.S. capital, is a compact city on the but Punjab argued that these states were neither
Potomac River, bordering the states of Maryland and riparian nor basin states.
Virginia. It’s defined by imposing neoclassical
monuments and buildings – including the iconic ones SECTION-C : LEGAL REASONING
that house the federal government’s 3 branches: the 53. (c) Option A is not correct as passage does not states about
Capitol, White House and Supreme Court. It's also home such situation. Option B is correct because of in pari
to iconic museums and performing-arts venues such as delicto: a finding that they were equally at fault in
the Kennedy Center. causing the contract's breach. Thus, option C is correct.
45. (c) The first major credit reporting agency, Retail Credit Co, Option B is not considered as the Frustration applies
was started in 1899. Over the years, Retail Credit where an unforeseen event makes performance of the
purchased smaller CRAs and expanded its business into contract impossible. Thus, no such context can be drawn
selling reports to insurers and employers. By the 1960s, from the expression given. Hence not correct.
significant controversy surrounded the CRAs because Option D is though correct but the reference taken from
their reports were sometimes used to deny services and the passage, author indicated that Loop Telecom was in
opportunities, and individuals had no right to see what pari delicto with the Department of Telecom. As a result,
was in their file. option B is preferred above option D as Loop telecom
46. (c) The FCRA was originally enacted in 1976, and was equally responsible for the illegality.
significantly revised in 2010. It creates registration 54. (b) The question asked in the factual matrix is that shyamlal
requirements and spending restrictions on Indian claims for restitution of damages. Thus, according to the
nonprofit organizations receiving foreign donations. The information given in Para 4, in adjudicating a claim of
most recent 2020 amendments of the FCRA brought in restitution, the court must determine the illegality which
additional restrictions, banning subgranting among caused the contract to become void and the role the
FCRA-registered organizations, setting a severe cap on party claiming restitution has played in it. Since
administrative spending, and further centralizing shyamlal had not caused any irregularity or illegality on
control of FCRA funding with the State Bank of India and his part, as he always paid tax on entering the province.
the Ministry of Home Affairs. Thus, option B is correct. Though option A is valid, option
47. (a) Punjab refused to share waters with Haryana stating it B presents reasoning that is more in accordance with the
was against the riparian principle which dictates that the context of the passage.
water of a river belongs only to the State and country or Option C is incorrect because shyamlal was not equally
States and countries through which the river in question liable for the municipality's illegality. As a result,
flows. incorrect.
48. (b) Under the Indus Waters Treaty signed between India Option D is incorrect because the facts do not mention
and Pakistan in 1960, all the waters of three rivers, any agreement between the parties. As a result, its
namely Ravi, Sutlej and Beas ( Eastern Rivers)averaging absence cannot be asserted.
around 33 million acre feet ( MAF) were allocated to 55. (a) Option A is correct as according to the passage, it was
India for exclusive use. observed by the court that when parties are in state of
49. (c) The Punjab Assembly passed the Punjab Termination of pari dalicto. Also, If the party claiming restitution was
Agreements Act in 2004, leading to the termination of equally or more responsible for the illegality (in
water-sharing agreements and creating complications comparison to the defendant), there shall be no cause for
for the construction of the Sutlej-Yamuna Link canal in restitution. Since shyamlal was paying bribe, hence he
Punjab. This legislative move was a significant knew the law. As a result, option D is inaccurate. Option
development in the state's water management policies, C is not considered because option A is more in
sparking legal and inter-state disputes. accordance with what is said in the passage.
50. (a) According to the Inter-State Water Disputes Act, once a Option B is incorrect since he paid bribes to the officers
reference is made to the tribunal, it is required to submit and thus bears equal responsibility for the illegality. As a
its report within 1 year. This provision is designed to result, incorrect.
expedite the resolution of water disputes and prevent 56. (a) Option A is correct as the passage states that, the
prolonged conflicts. doctrine of frustration states that frustration occurs
51. (b) Article 262(1) of the Constitution lays down that when an unforeseen event renders performance of a
“Parliament may by law provide for the adjudication of contract impossible or radically different from that
any dispute or complaint with respect to the use, originally contemplated by the parties. No party is
distribution or control of the waters of, or in, any inter- considered at fault. Since the present set of facts
State river, or river valley”. Parliament has enacted the suggests that codeALFA failed to complete the
Inter-State River Water Disputes Act, 1956. construction on time due to an injunction order, which
52. (b) The Satluj-Yamuna Link (SYL) canal dispute between was unforeseen. Option B is incorrect since the passage
Punjab and Haryana has brought up references to the does not specify what constitutes a physical
Punjab Termination of Agreement Act passed in 2004. impossibility. As a result, cannot be considered. Option C
The act annulled all inter-state agreements for sharing is incorrect because a contract is frustrated when an
the waters of the Ravi and Beas rivers. However, the unforeseen event makes contract performance
Supreme Court held the act "unconstitutional" in 2016, impossible. As a result, incorrect. Option D is incorrect
stating that the state could not unilaterally abrogate the since no such inference can be drawn from the passage.
water-sharing pact. The act had allocated water to As a result, inaccurate.
Haryana and Rajasthan from the Ravi and Beas rivers,
225
57. (a) Option A is correct as the purpose of the contract was for Option B is incorrect because there is no mention of such
Simran to go to Delhi and attend the concert. According a liability in the passage. As a result, incorrect.
to the passage, frustration occurs when an unforeseen Option C is incorrect because it does not suggest an
event renders performance of a contract impossible or answer based on the facts or the passage. As a result,
radically different from that originally contemplated by eliminated.
the parties. No party is considered at fault. Considering Option D is incorrect because there is no mention of
war as an unforeseen event, occurrence of which vicarious liability in the passage.
rendered the performance of contract impossible. 61. (b) Option A is incorrect as it is already mentioned in the
Option B is incorrect because the statement has no excerpt that, the trial Court renders its decision by
relevance to either the facts or the passage. As a result, acquitting the accused. Thus, not correct.
cannot be considered. Option B is correct as innocent until proven guilty
Option C is incorrect because, option A specify the more beyond reasonable doubt is the general legal principle
exact answer that common purpose is destroyed so it adopted by courts in India. Thus, option C is incorrect as
becomes frustrated. Warlike situation terminate the it states the contrary.
contract is not specific cancellation of concert is the main Option D is incorrect as it only suggests what has been
issue. As a result, inaccurate. asked in the question but does not elaborate it. Thus, not
Option D is incorrect because Simran purchased a "all- correct.
in-one ticket" that included both travel and a seat 62. (b) The assertion is correct because, according to paragraph
reservation at the show. As a result, if one becomes 1 of the passage, the offence of attempting suicide cannot
impossible, the other will eventually become impossible be established unless there exists abetment or
as well. As a result, option A is right. instigation. As a result, the court ruled that the
58. (b) Option A is not correct as the passage do not prosecution must meet Section 107 requirements in
expressively determines “intention” as an essential order to prove an offence under Section 306 of the IPC.
ingredient to prove an offence under section 306. Thus, Furthermore, the given statement implies that these are
not correct. not mutually exclusive events. As a result, they can be
According to Para 1 of the passage, the court ruled that said to be related and will occur concurrently. As a result,
to prove an offence under Section 306, the Prosecution correct.
must satisfy the ingredients of Section 107 first. Therefore, option B is correct.
According to the facts, Chavi was aware, albeit 63. (c) Assertion is correct as according to the text of passage,
inadvertently, that Radha would do whatever he the court in the case of Independent Newspaper Inc. v.
requested. As a result, the first condition of section 107 Brodie, held that the court can direct the disclosure of
is met when Chavi initiates Radha to jump. Hence, First details about the anonymous person subject to the some
condition of S. 107 attracts Chavi’s liability under section conditions. Hence, the court tried to give an appropriate
306. As a result, the first condition of Section 107 opportunity to the anonymous blogger who posts
exposes Chavi to liability under Section 306. Option B is allegedly defamatory material on a message board or
the correct answer. website.
Option C is incorrect because, in order to prove an Reason is false as no such statement can be inferred from
offence under Section 306, the ingredients of Section 107 the context of the passage. The statement given presents
must be proven. As a result, mere abetment will not a speculative suggestion bearing no relevancy with the
result in liability unless it is combined with an act of passage given. Hence, not correct.
instigation. As a result, incorrect. Therefore, Option C is correct.
Option D is incorrect because the current set of facts 64. (a) Option A is correct as according to the last Para of the
does not explicitly mention the role of a prosecution. As passage, if the complaint is made against a fake blog or
a result, such an option is unacceptable. impersonating profile on website, an Internet Service
59. (b) Option A is incorrect because Sanju did not commit Provider will at most block the subject page from access
suicide. He died as a result of an overdose. As a result, but do not reveal the person who created it has been
incorrect. mentioned in the last line of the passage. Since
Option B is correct because Jim attempted to overdose according to the facts the user’s filed a complaint against
on Sanju while in rehab. As a result, he is liable for the anonymous user, the ISP can block the subject page
abetment under Section 107 of the IPC. and it will not be deemed an arbitrary action.
Option C is incorrect because the statement lacks any All of the other options, B, C, and D, are rejected since
logical argument in light of the law presented in the they directly pertain to the situation when there is a case
passage. As a result, incorrect. of defamation (Refer context of para 1). Since no
Option D is incorrect because, once again, no legal reference to defamation can be found in the factual
principle is inferred from the passage that intention has matrix, none of the offered statements are true.
to be there in the given statement. Hence not correct. 65. (d) Option A is not correct as present case is a criminal case
60. (a) Option A is correct as the facts suggest that the helpline while court has power to issue direction only in civil
assistant didn’t help and escalated the situation to worse case. Thus, not correct. According to the passage, In
by stating that you are of no use, you should die. Thus, he India, the Code of Criminal Procedure, 1973 empowers
will be held liable for instigating rahul to take such the police to call for records from telephone companies
decision. As according Para 4 of the passages condition and Internet service Providers (ISPs) to identify the
first of section 107 states that when someone Instigates person who has made a transaction; and obtain his log
any person to do that thing. Thus, he will be held liable. information content contained in an e-mail/ chat
226
account and IP address details. Thus, option D is correct as failure of Vidya to conduct due diligence will not
and the police may request the disclosure of information influence whether coercion is committed or not.
about the anonymous individual. 69. (a) Option (a) is correct. As the element of threat or actual
Option B is not correct as the statement lacks committing of an offence in order to compel to enter into
information as to whom the necessary information a contract is missing in this case and hence, no coercion
should be disclosed to. Thus, not correct. Option C is has been committed. As per the passage, the purpose of
correct but this procedure will be followed on the order detaining of the property or commission of the act
of police officer thus, option A is better answer should be for the purpose of making the person enter
compared to option D. into the agreement. In the present case, despite the fact
66. (d) Option A is incorrect since the opportunity to defend the act committed is an offence under IPC (theft), the
oneself is available in the case of the anonymous same was not committed in order to compel Gayatri to
person's disclosure. The argument cannot be evaluated enter into an agreement but is a mere case of only theft.
since the stated factual matrix does not state that Meera Therefore, this claim of coercion will fail in the court of
writes her blog anonymously. Option C is likewise law making (d) also as incorrect. (c) is incorrect also
discarded on similar grounds. because the purpose was safekeeping and returning
Option B is incorrect because the provided rationale is when requested. Beyond this period, the safe-keeping is
incorrect; the paragraph does indicate in the context that without any accord of Gayatri. (b) is incorrect as, as
the internet service provider has the "right to block" mentioned, merely detaining property is not sufficient to
user accounts but only when balance of convenience lies claim coercion but there has to be some element of
as states in last 5th line of the passage that Normally, compelling to enter into an agreement.
Internet Service Providers will not entertain direct 70. (d) Option (d) is correct. As there was detaining of jewellery
request for disclosure of identity of a person, and even if by Pooja to compel Gayatri to gift her the jewellery and
the complaint is made against a fake blog or therefore there is coercion. Regardless of the fact
impersonating profile on website, an Internet Service whether the jewellery was selected by Gayatri herself,
Provider in whose favour balance of convenience lies the fact that the same was detained by Pooja and
will at most block the subject page from access but do threatened to be not released unless Gayatri dispenses
not reveal the person who created it. In present case one of them to her is apparent. Therefore, Pooja is liable
prima facie no balance of convenience lie in favour of under coercion. (a) is incorrect as despite it being called
complainant. a gift, the fact that the same was obtained through
The context of the sentence suggests that they normally threats and detaining, cannot be ignored. (b) is incorrect
blocks the user upon the complaint. They cannot restrict as it fails to establish a logical reasoning between the
Meera's blog since, according to the facts, the complaint passage and the factual matrix. (c) is incorrect as there
has not yet been received by the internet service needn’t be only actual threat to life and limb but as the
provider. As a result, option D is right. passage provides that detaining of property can also be
67. (d) Option A is incorrect since the factual question clearly covered under coercion, the statement itself is incorrect.
requests the next course of law enforcement authorities. 71. (b) Option (b) is correct. Statement I is correct as the passage
Since the current statement describes the action done by provides that the intention of committing the act should
the internet service provider. As a result, inaccurate. be compelling the person to enter into the contract, thus
Option B is not correct as the factual matrix states that making Statement I as correct. Statement II is correct as
there is a criminal complaint filed against the accused. passage clearly provides that such agreements can be
Thus, according to the passage, the Code of Criminal refuted (i.e. are voidable). Statement III is incorrect as
Procedure, 1973 empowers the police to call for records the passage nowhere provides that the act punishable by
from telephone companies and Internet service IPC should be civil wrong (but rather covers act of
Providers (ISPs) to identify the person who has made a threatening or committing any act), thus making
transaction; and obtain his log information content Statement III as incorrect.
contained in an e-mail/ chat account and IP address 72. (b) Option (b) is correct. As there was a threat to commit
details. As a result, Option D is the right answer. Option suicide by Ritesh and therefore Ritesh employed threat
C is ruled out since the investigation's point of view is to make his parents agree into buying the bicycle. One
not discussed in the paragraph. As a result, option C is no essential of Coercion is either committing or threatening
longer a possibility. to commit an act punishable under IPC and the passage
68. (d) Option (d) is correct. As there was no actual committing states that coercion is committing or threatening to
or threatening to commit any act forbidden by law and commit any act which is contrary to law accompanied
thus no coercion is involved in the whole transaction. In with compelling a person to enter into a contract. A fair
order for an act to qualify as coercion, there should be understanding of contract implies that contract needn’t
some element of forcing an individual to enter into an be written but can be oral as in the present factual
agreement which such person would not otherwise matrix, Ritesh’s parents were compelled to agree to buy
enter into. Therefore, in the present factual matrix there a bicycle due to his threat thus, amounting to coercion.
is no such forcing of Gayatri to enter into the agreement (a) is incorrect as passage provides that coercion can be
and it is merely maiking false representations. (a) and committing any act contrary to law. (c) and (d) incorrect
(b) are incorrect as heavy insistence on something is not as there is no relation between the factual matrix and the
sufficient to qualify as coercion but should satisfy the passage and fail to establish a sound reasoning.
conditions as enumerated in the passage. (c) is incorrect 73. (d) Option (b) is correct. Since threatening to commit suicide
is an offence punishable under IPC, the act of compelling
227
to buy a bicycle will amount to coercion. The major even if one of the parties dies without leaving a legal heir,
influencing factor in determining whether coercion has the people who gain interest over the contract's subject
been committed or not is whether the act/ threat to matter through that deceased party would be bound by
commit the act is an offence punishable under law/ the agreement and specific performance could be
contrary to law. Having established that threat to derived against them. Here, it is clear that the jewellery
commit suicide is contrary to law, the act of compelling was made especially for B and that she had an interest in
to buy the bicycle on this premise will amount to the deal's intended outcome. She must therefore carry
coercion. It is also to be understood that the factual out the remaining provisions of the contract. For the
matrix remains silent on the role of belief on the same reason, option A is incorrect. B is not the right
likelihood of the actual commission of the act, the fact answer because it is nowhere given in the question that
that there is an act contrary to law is sufficient, therefore G had chosen B as his legal heir. D is not the right answer
making (a) and (b) as incorrect. (c) is incorrect as it fails because the question makes no mention of anything of
to provide a sound reasoning based on the factual matrix the sort. It draws on outside knowledge.
and the passage. 78. (a) The correct answer is A since the passage states that a
74. (c) The correct answer is C since the passage states that contract is discharged by performance when both
when no timeframe for performance is given in the parties fulfill all primary responsibilities outlined in the
contract, it must be completed within a reasonable contract. Only when the conduct meets the required
amount of time. Unreasonably long delay cannot be standard of performance, the obligation is deemed to
considered as acceptable. The shipment would be have been fulfilled. A breach would result from failure to
worthless if it came after the winter season had passed comply. T shall be held accountable for breach because
since it was obvious that W had made a contract for he failed to carry out the contract in accordance with the
woollen clothing and was, as a result, planning to sell it terms established. C is not the correct answer since the
during the winter month. It was reasonable to suppose facts are in consonance with the passage. B is not the
that the clothing were necessary before the advent of right answer because it was not Y's sole responsibility to
winter. Since there was no mention of a deadline in the review the contract's conditions. D is incorrect because
contract, option C is incorrect. Even though the deadline A provides a clearer line of argument which is in
was not defined, since they were woollen clothing, it was consonance with the passage.
rational to anticipate that they would arrive before 79. (a) Option (a) is correct because the court was justified in its
winter, so option B is incorrect. D is incorrect because if position because according to the passage it is clarified
there is an unreasonable delay, the entire purpose of the that a person having no clear whereabouts known to
contract is defeated. Court cannot be granted protection. Option (b) is
75. (a) The correct answer is A since the passage states that a incorrect because the court did not give protection to the
promise that must be honoured within a specified period police commissioner. Option (c) is incorrect because the
of time must be performed on any day prior to the court was justified for denying him protection. Option
expiration of that period. If the products are supplied (d) is incorrect because the court was justified for
after business hours and are not accepted, the promise deciding the same and the police commissioner was
would be deemed unfulfilled. The passage states that indeed absconding.
when no time for performance is given in the contract, it 80. (d) Option (d) is correct because according to the passage
must be completed within a reasonable amount of time; the court should not have granted the protection to the
hence option B is not the right answer. C is not the right police commissioner as he was absconding. Option (a) is
answer because a contract is considered to be incorrect as per the passage, the Court must know his
discharged by performance when all of the primary whereabouts before granting him protection. Option (b)
obligations, both stated and implied, that are outlined in is incorrect because it does not provide a proper
the contract are met by both parties. D is not the correct explanation as per the passage for the same. Option (c)
answer because if the contract is valid and enforceable, is incorrect because his whereabouts were not known
even if one of the parties passes away without leaving a thus as per the passage the protection should not have
legal heir, those who gain ownership of the contract's been granted.
subject matter through that deceased party would be 81. (b) Option (b) is correct because it was important for the
subject to its terms, and the contract could be enforced court to know his whereabouts and the court was
against them. justified in denying him protection. Option (a) is
76. (d) The correct answer is D since the passage states that incorrect because the court was right in its approach but
when a promise has to be delivered within a specified the Option does not give an explanation for the same.
period of time, it must be performed on any day before Option (c) is incorrect because whether he was
the end of that period. Since the contract called for the absconding or not and the court must know his
milk powder to be delivered by the end of October, and whereabouts before granting him protection. Option (d)
F shipped the product in November, he can be held in is incorrect because irrespective of him being a police
breach of contract. It doesn't matter if there was a delay commissioner he should not be granted protection if his
of one hour or a day. A is not the correct answer for the whereabouts were not known.
same reason. Since the question mentioned a specific 82. (b) Option (b) is correct because there were no
deadline, option B is incorrect. Option C is incorrect whereabouts of the police commissioner so the Court
because, in the given situation, intention is irrelevant. was justified in denying him protection. Option (a) is
77. (c) The correct answer is C because, according to the incorrect because irrespective of being a police
passage, if the contract is valid and enforceable, then commissioner he would not be granted any protection.
228
Option (c) is incorrect because the court was right in its not the correct assumption. Option D is not correct, as
decision. Option (d) is incorrect because even though the VVIP convoys should be away within a democracy such
court was right in its decision, this Option does not give as India; otherwise, it will differentiate the citizens of
an explanation as to why it was right. India.
83. (a) Option (a) is correct because the statement is true as 88. (a) A central flaw is that the argument mistakes a condition
whereabouts of a person is an important factor for being sufficient for bringing about a result for a condition
granted protection. Option (b) is incorrect owing to the necessary for doing so. Option A is the correct answer as
same logic. Option (c) is incorrect because it makes a the author while putting forth the argument assumes the
contradictory assertion as the court has the power to traffic situation in India and Western countries to be the
deny the protection of a police commissioner. Option (b) same (a condition sufficient to bring about the changes.
is incorrect because the passage does not talk about the Assuming similar situations and overlooking other
right to protection. factors in reaching an outcome). Option B is the incorrect
84. (a) Option (a) is correct because whereabouts of a person is answer as it is not irrelevant because all the countries
an important factor for being granted protection and the mentioned in the passage are Europeans. Option C is not
statement is false. Option (b) is incorrect because this the correct one, as this is the truth under all
statement is true as the whereabouts of a person is circumstances and is not a central flaw. Option D is not
required by the court to grant him protection. Option (c) valid as the situations, and the treatment for the public
is incorrect because again it makes a correct statement servants in India and the Western countries are not the
by saying that the court has the right to deny granting same. Also, this is the conclusion one can draw, but
protection to the accused persons as seen in the passage. cannot be said to be the central flaw.
Option (d) is incorrect because the statement is true as 89. (c) The argument above talks of the conduct of the political
whereabouts of a police commissioner is also important leader whereby they do not take advantage of their
for granting him the protection by the court. position, but are true representatives of their people.
The main argument is to be weakened. To weaken the
SECTION-D : LOGICAL REASONING argument is to present a counter argument that
85. (c) Option C is the correct answer because the author tries completely invalidates the reasoning. Option (c)
to put forth through the passage to remove the VVIP weakens the argument completely. If the leaders, who
convoy system and enable a more intelligent security are the representatives of the people and have important
system that will benefit the public. Option A is not works of national interest are under threats, they cannot
correct as it is not about only India but for any country. be exposed in the public. Hence, they need protection,
It will provide a hurdle and inconvenience to the for which they have to maintain distance and cannot be
ordinary person and pose a threat to them. Option B is seen in public transports. Option (b) may appear close
not correct, as it is not the message that the author wants by way of their indifference and people may vent out
to pass on. Option D is not valid as only the President their anger is to assume that all leaders are indifferent,
cannot be held liable for the death of Vandana Mishra as which is an extreme case. Option (a) and (d) strengthen
the VVIP convoy system was not implemented by him. the argument. In option (a), if people are in awe
86. (b) Statement I is contrary to the essence of the passage. (marvel), they do not want their leaders to conduct
Statement II is correct as it is the desirable course of themselves as commoners, but would like to know them
action, as per the author. Refer to the lines, ‘In New from afar and they are fine with the distance. Option (d)
Zealand, when the PM’s motorcade was caught over the is actually the inference of the main argument.
speed limit, it was fined for dangerous driving.’ 90. (a) This option aligns with the author’s critique of VVIP
Statement III is not the correct course of action for India, convoys causing public inconvenience and traffic
given the size of the population and a smooth working of congestion. By implementing penalties, the government
the functionary. A course of action should be practical would deter the overzealous practices associated with
and feasible, keeping in mind the place and the factors VVIP movements, promoting accountability among
involved. One country’s asset can be other country’s public servants. The reference to New Zealand’s
liability. Option B is the correct answer as Article 14 of approach indicates that the author advocates for a more
the Indian Constitution states that the law holds every citizen-friendly and responsible governance model.
person to be the same before it, and no one acquires a Option B is unsuitable because it contradicts the
higher position in the eyes of the law. So a politician passage's argument by proposing to increase VVIP
doesn’t have any right to violate traffic rules, just like an convoys, which would exacerbate traffic congestion and
ordinary man. Option A is incorrect as statement I is not public inconvenience. Option C, suggesting a public
the right course of action. Option C is incorrect as awareness campaign, fails to address the core issue of
Statement III is incorrect and Option D is incorrect as traffic disruptions caused by these convoys, leaving
both Statements I and III are wrong courses of action. citizens to continue facing delays without any real
87. (b) Option B is the correct answer as if any one of the solution. Similarly, Option D, which advocates for
political and administrative VVIP manifestations is not dedicated lanes for VVIPs, prioritizes the needs of public
done away, the people of India will continue to suffer, officials over the general public, reinforcing the culture
and their rights for equality will be infringed. Option A is of privilege criticized by the author.
not correct as it is not relevant information in the 91. (c) The correct answer is C. There were some members of
passage, and, therefore, not the right assumption. Option the MPC who voted against maintaining the status quo.
C is not valid, as bringing more innovative security into Therefore, this option is true according to the passage. A
action will not end the perils of democracy; therefore, is incorrect since the author has clearly stated that there
229
is some uncertainty over the duration of the withdrawal tightening monetary policy. Option D provides context
of the policy support. B is incorrect since this is in clear about global growth but does not directly impact the
contrast with Option C. D is incorrect since the author RBI's immediate policy decisions regarding domestic
has indicated that the fall in CPI indicates fall in the inflation and economic recovery.
inflation rate. 97. (d) Option (D) is the correct answer as these are mentioned
92. (a) The correct answer is A. This is the best representation in the second paragraph of the passage. Refer to the
of the main idea of the passage. The RBI is uncertain over lines, ‘it is possible now to usher in personalized
its plans to withdraw the policy support and it needs medicine, treatments that are tailor made for individual,
clarity over the durability of recovery to make a firm makes option (A) correct. Refer to the line, ‘It brings
move on the policy support that it provides. B is together all the aspects including information on the
incorrect since the second part of the option is not of any lifestyle of the individual, biological aspect and clinical
concern of the passage. C is incorrect since this is not the aspect all these help in creating a unique blueprint of the
main concern of the passage but only a general piece of individual or the patient’, which makes option (B)
information. D is incorrect as well, for the same reason correct. Refer to the line, ‘Early detection of disease
as that of Option B’s second part. becomes significantly easier that provides the medical
93. (c) The correct answer is C. The author has indicated that practitioner with the power to easily diagnose and
the RBI’s stance on growth considerations is likely to provide accurate treatment for the same.’ This makes
change with time as there is more clarity over the option (C) correct. Therefore, option (D) is the correct
durability. Refer to the lines, ‘The withdrawal of policy answer.
support is likely to be gradual with the process playing 98. (a) An assumption is the basis of an argument on which the
out at multiple levels — beginning with normalisation of argument is built upon. It is the core idea behind the
liquidity, which has begun, followed by a hike in the argument left unsaid or taken for granted by the author.
reverse repo, a shift in the stance from accommodative The above argument relies on the fact that the aging
to neutral, culminating in a repo rate hike. However, the people require monitoring on basic functioning. Look at
duration over which this plays out is uncertain. Perhaps the statement…providing timely medical supplies, smart
a few more months of data will provide greater clarity heating devices or temperature control, etc. Therefore,
over the durability of the recovery, allowing the MPC to option (A) is the answer. Option (B)is far-fetched even
firm up its exit plans.’ A is incorrect since this is not of for an assumption. This cannot be the assumption, as it
any concern in the passage. B is in clear contrast with is an extreme case, which cannot be the basis for the
what the pieces of evidence in the passage suggest. argument. Option (C)is not the correct assumption
Option (d) is incorrect because the passage discusses a because the passage does not explicitly state or imply
gradual withdrawal of policy support rather than that all elderly people live alone. The argument focuses
immediate or drastic changes. on the need for monitoring and assistance to enhance
94. (d) The correct answer is D. The author has presented comfort, which could apply regardless of whether
evidence for option A in the subsequent sentence of “So elderly people live alone or with others.. Option (D) may
far, the MPC has continued to attach primacy to growth be confusing, but again on closer look, one can see that is
considerations.” The author has presented some pieces it an absolute assumption. The assumption is that all the
of evidence for option B in the subsequent sentence of aging people are unable to perform the basic
“But the decision was not unanimous.” Option C is a functioning.
claim which is inconsistent with the author’s claims. 99. (d) To undermine an argument is to weaken it. To weaken an
95. (d) Neither of the assumptions are inbuilt in the passage. argument, one has to negate the contention of the
Assumption I, though partly true in terms of that there author. In the passage, the author makes the point that
are more than one committee of the RBI, since the post pandemic, life has been difficult for the elderly
passage mentions the monetary policy committee of the people and AI is the way to make life comfortable. To
RBI, it is based on the assumption that the author must negate, one has to attack the fact that AI will not help.
put up the name of the committee in order to avoid Option (D) is the most suitable answer. If the elderly are
confusion, but the latter part of the statement cannot be not able to figure out a simple function of a mobile due
supposed apart from the monetary policy committee, the to weak faculty and poor memory, how would they
other committees also look into the monetary aspect. figure out multiple AI functions. Option (A) is rule out, as
Therefore, assumption I is incorrect. Assumption II is one is not sure of the post-pandemic situation. Pandemic
incorrect as it nullifies the argument completely. The has only highlighted the challenges of the aging people.
argument clearly states that one member was not in This does not negate the passage. Option (B) is far-
unison to the decision. Therefore, assumption II is fetched as it is absolute, and the fact the NGOs will help
incorrect, which makes option (d) as the answer. the elderly people. Option (C) strengthens the argument.
96. (b) Option B is the correct answer. Option B strengthens 100. (b) Option (B) covers most of the points that would make life
the case for shifting to a more neutral or restrictive comfortable for the elderly people. Option (A) is absurd,
policy by indicating that rising inflation expectations as teaching multi-tasking is not the solution; in fact, it is
can lead to actual inflation, reinforcing the need for the not making life comfortable. Option (C) is ruled out, as it
RBI to act preemptively. Option A presents a concern is one of the aspects and does not give a complete
about asset bubbles, but does not directly relate to the picture. Option (D), too, is one of the aspects, and does
immediate inflationary pressures that the RBI needs to not cover the entire gamut.
address. Option C emphasizes the need for continued 101. (b) The passage is about how artificial technology can help
support for small businesses, which could argue against make the lives of the elderly people comfortable. Option
230
(B) best reflects the main point in brief. Option (A) is out wants to say that much attention is already being given
of scope, as there is no mention of reversing of age. to cognitive skills, but one needs to realise the true value
Option (C) is close, but does not cover the essential point of soft skills too. Options B, C and D are true hence cannot
of technology. Option (D) is incomplete in terms of being be the correct answer here. Option B can be deduced
the main point. from these lines, ‘The main task of education and
102. (a) Option A identifies a critical flaw in the argument by research is to train people to perform in future economic
pointing out that the assumption of universal access to and technological environments with many unknowns.’
technology and the internet among the elderly is Option C can be derived from, ‘Human improvement, the
problematic. Without this access, the proposed benefits ability to try new technologies should not be overlooked
of big data and AI cannot be realized for many as higher productivity embedded in new technology only
individuals. Option B is less relevant because while blossoms if humans have the skill to manage technology.’
anecdotal evidence may be used, the focus is primarily And option D is derived from, ‘In recent decades most
on technology's impact, not solely on social lives. Option countries have fallen into the trap of overextending cost-
C raises a valid concern about the potential negative benefit analyses while training students to solve
effects of technology but does not directly address a flaw yesterday’s problems.’
in the primary argument. Option D is relevant but does 106. (c) Note that the author talks about what the current
not highlight a logical flaw in the argument itself; rather, education and research patterns include and focus on.
it points out an oversight regarding costs. For it to become holistic and ensure all round
103. (c) Option C is the correct answer as it is mentioned in the development, it must include soft skills that more closely
passage that education and research are increasingly out relate to interpersonal skills so that human
of touch with demand for skills. Refer to the first line of improvement aspect can be covered and one can easily
the second paragraph. Option A is incorrect as it adapt to the required skillset for the plethora of job
mentions that the education and research offer no opportunities knocking the door in the domains of
productivity benefits, while the passage mentions, ‘Much health and entertainment. So, option C is the correct
attention is devoted to high-caliber education and answer. Option A does not carry enough weightage to be
research and for good reason’. Nowhere it undermines the answer, as it is incomplete. Option B is far-fetched
productivity, but yes, it does talk about adding value to it and does not find support in the passage. Option D is
by realizing the importance of human factors and soft contrary to what the author says. In fact, he laments the
skills, making option C correct. Option B: This is also short-term planning on education programmes. Refer to
untrue as the passage mentions ‘tendency to focus on the line, ‘Governments enforce short-term fiscal
cognitive skills, including the STEM topics of science, planning on education programs, yet measuring social
technology’, which means that STEM competency is skills is not as easy as calculating STEM competences.’
being paid equal attention, not that it is being paid less 107. (b) The correct answer is option B. The central flaw is a fault
attention. Option D is contradictory as the author makes in the core idea of the author’s argument; basically, the
the opposite contention in the passage. Refer to the lines, assumption is flawed. The author argues that while more
‘Education and research are increasingly out of touch attention is given high-calibre education and research
with demand for skills. The tendency to focus on and for good reason, there is a trend in demand for
cognitive skills, including the STEM topics of science, human factors, which means that he considers that the
technology, engineering, and mathematics, cannot high-calibre education and research and human factors
preclude the productivity benefit of soft skills – applying are two separate entities and that there is no
knowledge and finding opportunities offered by interconnectedness between them. He fails in making
technology.’ the logical point that as the trend changes with more
104. (d) The correct answer is option D. Note that the passage emphasis on the human factors, the high-calibre
through out talks about how not only cognitive skills are education and research will be moulded to suit the
necessary, but there is a dire need to focus on inculcating demand. Option B reflects the flaw in the reasoning.
soft skills that involve human factors. Owing to this, Option (a) is the main argument of the author. Option (c)
option D is the only correct choice here. Option A cannot be the central flaw to be the flaw as the author
mentions only cognitive skills which is incorrect. Option does argue that the two are separate entities. Therefore,
B The passage talks about holistic education and this statement cannot be the central flaw. Option (d) is
learning new technologies rather than only far-fetched as a flaw. It is not suggested in the argument;
concentrating on academics. So this option is also therefore, cannot represent the central flaw.
incorrect. Option C is a fact stated in the passage, it is not 108. (a) The author argues that while cognitive skills,
just the main theme. particularly in STEM fields, are important, they cannot
105. (a) The correct answer is A owing to these lines of the solely prepare students for the unknowns of future
passage, ‘The tendency to focus on cognitive skills, employment, particularly without the integration of soft
including the STEM topics of science, technology, skills. Therefore, the assertion that STEM education
engineering, and mathematics, cannot preclude the alone is sufficient is contrary to the author's perspective.
productivity benefit of soft skills – applying knowledge Options B and C are not the correct answers because
and finding opportunities offered by technology.’ Note they align with the author's arguments. The author
that cognitive skill relates to conscious intellectual emphasizes the importance of integrating soft skills into
effort, such as thinking, reasoning, or remembering. ... education to ensure adaptability and innovation in the
Noncognitive or “soft skills” are related to motivation, workplace, making option B a statement the author
integrity, and interpersonal interaction. The author would likely agree with. Similarly, the author highlights
231
the need for education to focus on human factors to 3n + 4n + 2n + n = 200
address the needs of an aging population, which is ⇒ 10 n=200
exactly what option C states. n=20
Students who passed in exactly two subjects = d+ e + f =
SECTION-E : QUANTITATIVE TECHNIQUES 4n = 4×20 = 80.
109. (c) is the right answer 116. (d) is the right answer
- V1 V2 V3
After operation 0 20 L M + 20 L 60 L w + 20 L
1: W milk
After operation 0 15 L M + 25 L 55 L w + 25 L
2: W milk
110. (b) is the right answer It is given that, none = n,
- V1 V2 V3 g = 2n
Initial volume: 0L 40 L milk 80 L W water (a + b+c+d +e+f + g) = (a + b+ c)3
After operation 0 20 L M + 20 L 60 L w + 20 L a + b + c = 3n
1: W milk d+e+f=4n
After operation 0 15 L M + 25 L 55 L w + 25 L g=2n
2: W milk (a + b + c + d + e + f + g + n) = 200
111. (a) is the right answer 3n + 4n + 2n + n = 200
- V1 V2 V3 ⇒ 10 n=200
n=20
Initial volume: 0L 40 L milk 80 L W water
Students who passed in exactly one subject = a + b + c
After operation 1: 0 20 L M + 20 60 L w + 20 L
=3n
LW milk
3×20=60 Choice(D)
After operation 2: 0 15 L M + 25 55 L w + 25 L
117. (a) is the right answer
LW milk
Check from the above data.
112. (c) 80 L
113. (b) 2:1
114. (b) Is the correct answer.
115. (c) is the right answer
232